Anda di halaman 1dari 176

Faculty of Actuaries

Institute of Actuaries

EXAMINATION
6 April 2005 (am)

Subject CT1
Financial Mathematics
Core Technical
Time allowed: Three hours
INSTRUCTIONS TO THE CANDIDATE
1.

Enter all the candidate and examination details as requested on the front of your answer
booklet.

2.

You must not start writing your answers in the booklet until instructed to do so by the
supervisor.

3.

Mark allocations are shown in brackets.

4.

Attempt all 11 questions, beginning your answer to each question on a separate sheet.

5.

Candidates should show calculations where this is appropriate.

Graph paper is not required for this paper.

AT THE END OF THE EXAMINATION


Hand in BOTH your answer booklet, with any additional sheets firmly attached, and this
question paper.
In addition to this paper you should have available the 2002 edition of the
Formulae and Tables and your own electronic calculator.

CT1 A2005

Faculty of Actuaries
Institute of Actuaries

A bond is priced at 95 per 100 nominal, has a coupon rate of 5% per annum
payable half-yearly, and has an outstanding term of five years.
An investor holds a short position in a forward contract on 1 million nominal of this
bond, with a delivery price of 98 per 100 nominal and maturity in exactly one year,
immediately following the coupon payment then due.
The continuously compounded risk-free rates of interest for terms of six months and
one year are 4.6% per annum and 5.2% per annum, respectively.
Calculate the value of this forward contract to the investor assuming no arbitrage. [5]

An investment fund had a market value of 2.2 million on 31 December 2001 and
4.2 million on 31 December 2004. It had received a net cashflow of 1.44 million
on 31 December 2003.
The money weighted rate of return and the time weighted rate of return for the period
from 31 December 2001 to 31 December 2004 are equal (to two decimal places).
Calculate the market value of the fund immediately before the net cashflow on
31 December 2003.

[7]

A computer manufacturer is to develop a new chip to be produced from 1 January


2008 until 31 December 2020. Development begins on 1 January 2006. The cost of
development comprises 9 million payable on 1 January 2006 and 12 million
payable continuously during 2007.
From 1 January 2008 the chip will be ready for production and it is assumed that
income will be received half yearly in arrear at a rate of 5 million per annum.
(i)

Calculate the discounted payback period at an effective rate of interest of 9%


per annum.
[6]

(ii)

Without doing any further calculations, explain whether the discounted


payback period would be greater than, less than or equal to that given in part
(i) if the effective interest rate were substantially greater than 9% per annum.
[2]
[Total 8]

CT1 A2005

The force of interest, (t ) , is a function of time and at any time t (measured in years)
is given by
(t )

0.07 0.005t

for t

0.06

for t

(i)

Calculate the accumulation at time t = 10 of 500 invested at time t = 0.

[3]

(ii)

Calculate the present value at time t = 0 of a continuous payment stream at the


rate of 200e0.1t paid from t = 10 to t = 18.
[5]
[Total 8]

A university student receives a 3-year sponsorship grant. The payments under the
grant are as follows:
Year 1
Year 2
Year 3

5,000 per annum paid continuously.


5,000 per annum paid monthly in advance.
5,000 per annum paid half yearly in advance.

Calculate the total present value of these payments at the beginning of the first year
using a rate of interest of 8% per annum convertible quarterly.
[8]

At time t = 0 an investor purchased an annuity-certain which paid her 10,000 per


annum annually in arrear for three years. The purchase price paid by the investor was
25,000.
The value of the retail price index at various times was as shown in the table below:
Time t (years):
Retail price index:
(i)

t=0
170.7

t=1
183.3

t=2
191.0

t=3
200.9

Calculate, to the nearest 0.1%, the following effective rates of return per
annum achieved by the investor from her investment in the annuity:
(a)
(b)

the real rate of return; and


the money rate of return
[7]

(ii)

CT1 A2005

By considering the average rate of inflation over the three-year period, explain
the relationship between your answers in (a) and (b) of (i).
[2]
[Total 9]

PLEASE TURN OVER

A loan of nominal amount 100,000 is to be issued bearing coupons payable quarterly


in arrear at a rate of 5% per annum. Capital is to be redeemed at 103 on a single
coupon date between 15 and 20 years after the date of issue, inclusive. The date of
redemption is at the option of the borrower.
An investor who is liable to income tax at 20% and capital gains tax of 25% wishes to
purchase the entire loan at the date of issue. Calculate the price which the investor
should pay to ensure a net effective yield of at least 4% per annum.
[9]

A small insurance fund has liabilities of 4 million due in 19 years time and 6
million in 21 years time. The manager of the fund has sold the assets previously held
and is creating a new portfolio by investing in the zero-coupon bond market. The
manager is able to buy zero-coupon bonds for whatever term he requires and has
adequate monies at his disposal.
(i)

Explain whether it is possible for the manager to immunise the fund against
small changes in the rate of interest by purchasing a single zero-coupon bond.
[2]

(ii)

In fact, the manager purchases two zero-coupon bonds, one paying 3.43
million in 15 years time and the other paying 7.12 million in 25 years time.
The current interest rate is 7% per annum effective.
Investigate whether the insurance fund satisfies the necessary conditions to be
immunised against small changes in the rate of interest.
[8]
[Total 10]

The one-year forward rate of interest at time t = 1 year is 5% per annum effective.
The gross redemption yield of a two-year fixed interest stock issued at time t = 0
which pays coupons of 3% per annum annually in arrear and is redeemed at 102 is
5.5% per annum effective.
The issue price at time t = 0 of a three-year fixed interest stock bearing coupons of
10% per annum payable annually in arrear and redeemed at par is 108.9 per 100
nominal.
(i)

Calculate the one-year spot rate per annum effective at time t = 0.

[4]

(ii)

Calculate the one-year forward rate per annum effective at time t = 2 years.
[3]

(iii)

CT1 A2005

Calculate the two-year par yield at time t = 0.

[3]
[Total 10]

10

(i)

In any year, the interest rate per annum effective on monies invested with a
given bank has mean value j and standard deviation s and is independent of the
interest rates in all previous years.
Let Sn be the accumulated amount after n years of a single investment of 1 at
time t = 0.
j )n .

(a)

Show that E[ Sn ] = (1

(b)

Show that Var [Sn ] = (1 2 j

j2

s 2 ) n (1 j ) 2 n .
[5]

(ii)

The interest rate per annum effective in (i), in any year, is equally likely to be
i1 or i2 (i1 i2 ) . No other values are possible.
(a)

Derive expressions for j and s2 in terms of i1 and i2.

(b)

The accumulated value at time t = 25 years of 1 million invested with


the bank at time t = 0 has expected value 5.5 million and standard
deviation 0.5 million.
Calculate the values of i1 and i2.
[8]
[Total 13]

CT1 A2005

PLEASE TURN OVER

11

(i)

A loan is repayable over 20 years by level instalments of 1,000 per annum


made annually in arrear. Interest is charged at the rate of 5% per annum
effective for the first 10 years, increasing to 7% per annum effective for the
remaining term.
Show that the amount of the original loan is 12,033.56. (Minor discrepancies
due to rounding will not be penalised).
[2]

(ii)

The following are the details from the loan schedule for year x, i.e. the year
running from exact duration x 1 years to exact duration x years.
Instalment paid at the end of the year

Year x

Loan outstanding at the


beginning of the year

Interest

Capital

8,790.48

439.52

560.48

Determine the value of x.


(iii)

At the beginning of year 11, it is agreed that the increase in the rate of interest
will not take place, so that the rate remains at 5% per annum effective for the
remainder of the loan. The annual instalment will continue to be payable at
the same level so that there may be a reduced term and a reduced final
instalment.
(a)

Calculate by how many years, if any, the repayment schedule is


shortened.

(b)

Calculate the amount of the reduced final instalment.

(c)

Calculate the reduction in the total interest paid during the existence of
the loan as a result of the interest rate not increasing.
[7]
[Total 13]

END OF PAPER

CT1 A2005

[4]

Faculty of Actuaries

Institute of Actuaries

EXAMINATION
April 2005

Subject CT1
Financial Mathematics
Core Technical
EXAMINERS REPORT

Introduction
The attached subject report has been written by the Principal Examiner with
the aim of helping candidates. The questions and comments are based around
Core Reading as the interpretation of the syllabus to which the examiners are
working. They have however given credit for any alternative approach or
interpretation which they consider to be reasonable.

M Flaherty
Chairman of the Board of Examiners
15 June 2005

Faculty of Actuaries
Institute of Actuaries

Subject CT1 (Financial Mathematics Core Technical)

Ke

April 2005

Examiners Report

r T t

where:
t
T
r

is the present time


is the time of maturity of the forward contract
is the continuously compounded risk-free rate of interest for the interval from t
to T
is the spot price of the security at time t
is the present value, at the risk-free interest rate, of the income generated by the
security during the interval from t to T
is the delivery price of the forward contract
is the value of a long position in the forward contract

S
I
K
f

Here, working with 100 nominal,


S = 95, K = 98, T t =1, r = 0.052
I

0.046 0.5

2.5 e
f

0.052 1

95 4.81648 98e

0.052

4.81648
2.85071

The value of the investor s short position in a forward contract on 1 million is


therefore

1, 000, 000
100

10, 000 2.85071


= 28,507

MWRR: 2.2 1 i

1.44 1 i

Estimate i

6% , LHS

4.1466

7% , LHS

4.2359

0.06

4.2

4.2 4.1466
0.01
4.2359 4.1466

= 6.60% p.a. to two decimal places


Let F = Fund value before net cashflow on 31 December 2003

Page 2

Subject CT1 (Financial Mathematics Core Technical)

April 2005

Examiners Report

Then,
TWRR = 6.60% p.a. means that
1.0663

F
2.2

0.63452

4.2
F 1.44

F
F 1.44

0.63452 F 0.63452 x 1.44

F =2.5m

(i)

Work in millions:
9 12v a1 at 9%

PV of liabilities

i
9 12v. v
9 12 0.917432 1.044354
= 19.54811
The assets up to k 2 years from 1 January 2006 have:

PV

5v 2 a

5v 2
i

ak

5 0.84168 1.022015 ak
4.301048 ak

With k

6, PV

4.301048 4.4859

= 19.2941
The next payment of 2.5 million at k = 6.5 is made at time
8.5 and has present value = 2.5 v8.5 1.2018

Page 3

Subject CT1 (Financial Mathematics Core Technical)

April 2005

Examiners Report

This would make PV of assets (20.5m) > PV of liabilities (19.5m)


Discounted payback period = 8.5 years.
(ii)

The income of the development is received later than the costs are incurred.
Hence an increase in the rate of interest will reduce the present value of the
income more than the present value of the outgo. Hence the DPP will increase.

(i)

Accumulation = 500 e 0

10

s ds

8
0

= 500 e

0.07 0.005 s ds

0.07 S

= 500 e
= 500e0.40

10
0.06 ds
8

0.005 2
S
2

0.06 S
0

0.12

= 841.01
(ii)

PV

18
10

200e0.1t .e

18
10

18
10

200e

0.1t

s ds

8
0.07
0

.e

200e0.1t . e

200e0.08

t
0

0.40

18 0.04t
e
10

200e0.08 0.04t
e
0.04

dt
0.005 s ) ds

. e0.48

dt

dt

18
10

5000 e0.08 e0.72 e0.40

Page 4

0.06t

t
0.06 ds
8

3047.33

10
8

Subject CT1 (Financial Mathematics Core Technical)

12
1

Present Value = 5000 a1


where

1 i

a1

.v

1.02

v 2 .a

v.a

April 2005

Examiners Report

at i %

i 8.24322% p.a. effective

0.0824322
1
.
Ln 1.0824322 1.0824322
0.9614201

12
1

and a

1
12

1.0824322

1 v

12

where
1.0824322 = 1
12
1

and a

So PV

12

12

12

12

0.0794725

0.9645970
1

1.0824322 2 .

1 v
i

where 1.0824322
a

0.0808000

= 0.9805844
5000 0.9614201 v 0.9645970 v 2 0.9805844

13, 447.39

Examiners Comment: There are other valid methods for obtaining the required
answer which also received full credit.

Page 5

Subject CT1 (Financial Mathematics Core Technical)

(i)

(a)

April 2005

Examiners Report

Work in t = 0 monetary values

170.7 2 170.7 3 170.7


v
v
183.3
191.0
200.9

25000 = 10000 v

where v

with i = real rate of return

1 i

Try 4% RHS = 24770.94


3% RHS = 25241.25

0.03

25241.25 25000
0.01
25241.25 24770.94

= 0.0351
(b)

i.e. 3.5%

25000 = 10000 a3 at i % p.a.

2.5

From tables, a

2.5313 at 9%

= 2.4869 at 10%

0.09

2.5313 2.5
0.01
2.5313 2.4869

= 0.097
i.e. 9.7% p.a.
(ii)

We should find that

1 i
1 i

1 e

where e = average annual rate of inflation over the period.


Hence

1 i
1

1 i

1.097
1.06
1.035

which implies 6% p.a. inflation over the period

Page 6

Subject CT1 (Financial Mathematics Core Technical)

April 2005

Examiners Report

The actual average inflation rate was:


3

1 e

200.9
170.7

e 5.6% p.a.

The inflation rate would not be expected to be exactly 6% p.a. since the Retail
Price Index is not increasing by a constant amount each year.

g 1 t1
i

1.04

0.039414

0.05
0.80 0.038835
1.03
1 t1 g

Capital gain on contract


Assume redeemed as late as possible (ie: after 20 years) to obtain minimum yield.
Price of stock, P:
P

4
20

100000 0.05 0.80 a

103000 0.25 103000 P v 20 at 4%


4
20

4000 a
P

77250v 20

1 0.25v 20

4000 1.014877 13.5903 77250 0.45639


1 0.25 0.45639
= 102,072.25

Page 7

Subject CT1 (Financial Mathematics Core Technical)

April 2005

Examiners Report

(i)

No, because the spread (convexity) of the liabilities would always be greater
than the spread (convexity) of the assets
3rd Redington condition would
never be satisfied.

(ii)

Conditions required:

(a)

VA VL

(b)

VA' VL'.

(c)

VA" VL"

where differentiation can be in respect of delta or i. In this solution, it is in


respect of delta.
(a)

3.43v15 7.12v 25 @ 7%

VA

= 2.5550
4v19 6v 21

VL

= 2.5551
VA VL (ignoring rounding)
(b)

V 'A

3.43 15v15 7.12 25v 25

= 51.444
V 'L

4 19v19 6 21v 21

= 51.445
V ' A V ' L (ignoring rounding)
(c)

V "A

3.43 152 v15 7.12 252 v 25

= 1099.627
V "L

4 192 v19 6 212 v 21

1038.322
V " A V "L
all 3 conditions are satisfied.

Page 8

Subject CT1 (Financial Mathematics Core Technical)

April 2005

Examiners Report

Examiners Comment: There are other valid methods for obtaining the
required answer which also received full credit.

(i)

From two year stock information:


102v 2 at 5.5%

Price = 3a2

= 3 1.84632 + 102 0.89845


= 97.1811
Therefore, from one-year forward rate information,

97.1811

3 102
1 i1 1 f1,1

1 i1

where i1 =one-year spot rate

f1,1 = one-year forward rate from t = 1


97.1811

97.1811
i1
(ii)

3
1 i1

105
1 i1 1.05

103
1 i1

5.9877% p.a.

From three-year stock information:


108.9 =

10
1 i1

10
1 ii 1.05

110
1 i1 1.05 1 f 2,1

where f 2,1 =one-year forward rate from t = 2


Hence

108.9

10
10
110
1.059877 1.059877 1.05 1.059877 1.05

108.9 9.4351 8.9858

1 f 2,1

110
1.11287 1 f 2,1

Page 9

Subject CT1 (Financial Mathematics Core Technical)

f 2,1

(iii)

Let y2 % p.a. be the two-year par yield

100 = y2

100

y2

(i)

(a)

1
1 i1 i

1 i1

f1,1

1.05

100
1.059877 1.05

89.8577

5.506% p.a.

Let it be the (random) rate of interest in year t . Let Sn be the


accumulation of a single investment of 1 unit after n years:

E Sn

E 1 i1 1 i2

1 in

E Sn

E 1 i1 E 1 i2

E 1 in as it are independent

E Sn

(b)

100
1 i1 1 f1,1

1
1
1.059877 1.059877

y2 1.84208

E it

E S n2

1 i1 1 i2

E 1 i1

1 in

E 1 i2

1 2 j s2

j2

E 1 in

E 1 2i1 i12 E 1 2i2 i22

Page 10

Examiners Report

9.245% p.a.

100 = y2

10

April 2005

(using independence)

E 1 2in in2

Subject CT1 (Financial Mathematics Core Technical)

as E ii2
Var Sn

(ii)

(a)

s2

1 2 j s2

j2

V it

E Interest

E it

s2

E S25

2n

1 2 2
i1 i2
4

25

E Interest

1
i1 i2
2

1
i1.i2
2
2

1
i1 i2
2

(b)

j2

E Interest 2

1 2 2
i1 i2
2

Examiners Report

1
i1 i2
2

Var Interest

April 2005

5.5

0.0705686

Var S 25

j2

s2

25

1 2 0.0705686 0.07056862

s2

s2

1 2j

50

25

0.000377389

1
i1 i2
4

1.0705686

50

0.25

i1 i2

0.0388530

i1 i2

2 0.07056862 = 0.1411372

i1

0.000377389

Hence, s 2

2i1

0.5

(taking positive root since i1

i2 )

0.0388530 0.1411372
0.089995

8.9995%p.a.

Page 11

Subject CT1 (Financial Mathematics Core Technical)

and i2

11

(i)

0.051142

April 2005

Examiners Report

5.1142%p.a.

5%
7%
1000 a10
v10
5% a10

Loan

= 1000 7.7217 0.61391 7.0236


= 12033.56
(ii)

Note

439.52
8790.48

0.05

x 10

5%
8790.48 1000 a11
x

1 v11
8.79048

v11

0.05

8.79048 20

v11

7.0236

20 7.0236 v11

11.20952
12.9764

x 7%
v11
5% a10

0.86384 at 5%

x 8

(iii)

Let Y = reduced final payment


n = new total term of loan
7%
Loan outstanding after 10 years = 1000 a10
= 7,023.60

After change is made:


7023.60 = 1000 an

11

Yv n

10

at 5%

try n = 20 (i.e., keep to original term)


RHS = 1000 7.1078 Y 0.61391
Y

137.15

doesn t work
try n = 19

Page 12

Subject CT1 (Financial Mathematics Core Technical)

April 2005

Examiners Report

RHS = 1000 6.4632 Y 0.64461


Y

869.36

Hence:
(a)

Term shortened by 1 year

(b)

Final instalment = 869.36

(c)

Under original terms, total interest paid is:


20 1000 12033.56 7966.44

Under changed terms, total interest paid is:


18 1000 869.36 12033.56 6835.80
difference = 1,130.64

END OF EXAMINERS REPORT

Page 13

Faculty of Actuaries

Institute of Actuaries

EXAMINATION
7 September 2005 (am)

Subject CT1
Financial Mathematics
Core Technical
Time allowed: Three hours
INSTRUCTIONS TO THE CANDIDATE
1.

Enter all the candidate and examination details as requested on the front of your answer
booklet.

2.

You must not start writing your answers in the booklet until instructed to do so by the
supervisor.

3.

Mark allocations are shown in brackets.

4.

Attempt all 11 questions, beginning your answer to each question on a separate sheet.

5.

Candidates should show calculations where this is appropriate.

Graph paper is not required for this paper.

AT THE END OF THE EXAMINATION


Hand in BOTH your answer booklet, with any additional sheets firmly attached, and this
question paper.
In addition to this paper you should have available the 2002 edition of the
Formulae and Tables and your own electronic calculator.

CT1 S2005

Faculty of Actuaries
Institute of Actuaries

Describe how cashflows are exchanged in an interest rate swap .

An investor has earned a money rate of return from a portfolio of bonds in a particular
country of 1% per annum effective over a period of ten years. The country has
experienced deflation (negative inflation) of 2% per annum effective during the
period.

[2]

Calculate the real rate of return per annum over the ten years.

[2]

Calculate the time in days for 1,500 to accumulate to 1,550 at:


(a)
(b)

a simple rate of interest of 5% per annum


a force of interest of 5% per annum
[4]

The force of interest (t) at time t is a + bt2 where a and b are constants. An amount of
200 invested at time t = 0 accumulates to 210 at time t = 5 and 230 at time t = 10.
Determine a and b.

(i)

[5]

Calculate the present value of 100 over ten years at the following rates of
interest/discount:
(a)
(b)
(c)

a rate of interest of 5% per annum convertible monthly


a rate of discount of 5% per annum convertible monthly
a force of interest of 5% per annum
[4]

(ii)

A 91-day treasury bill is bought for $98.91 and is redeemed at $100.


Calculate the annual effective rate of interest obtained from the bill.

[3]
[Total 7]

(i)

State the features of a eurobond.

(ii)

An investor purchases a eurobond on the date of issue at a price of 97 per


100 nominal. Coupons are paid annually in arrear. The bond will be
redeemed at par twenty years from the issue date. The rate of return from the
bond is 5% per annum effective.
(a)
(b)

[3]

Calculate the annual rate of coupon paid by the bond.


Calculate the duration of the bond.
[6]
[Total 9]

CT1 S2005

A bank makes a loan to be repaid in instalments annually in arrear. The first


instalment is 50, the second 48 and so on with the payments reducing by 2 per annum
until the end of the 15th year after which there are no further payments. The rate of
interest charged by the lender is 6% per annum effective.
(i)

Calculate the amount of the loan.

[6]

(ii)

Calculate the interest and capital components of the second payment.

[3]

(iii)

Calculate the amount of capital repaid in the instalment at the end of the
fourteenth year.
[3]
[Total 12]

An insurance company has just written contracts that require it to make payments to
policyholders of 1,000,000 in five years time. The total premiums paid by
policyholders amounted to 850,000. The insurance company is to invest half the
premium income in fixed interest securities that provide a return of 3% per annum
effective. The other half of the premium income is to be invested in assets that have
an uncertain return. The return from these assets in year t, it, has a mean value of
3.5% per annum effective and a standard deviation of 3% per annum effective. (1 + it)
is independently and lognormally distributed.
(i)

Deriving all necessary formulae, calculate the mean and standard deviation of
the accumulation of the premiums over the five-year period.
[9]

(ii)

A director of the company suggests that investing all the premiums in the
assets with an uncertain return would be preferable because the expected
accumulation of the premiums would be greater than the payments due to the
policyholders.
Explain why this still may be a more risky investment policy.

CT1 S2005

[2]
[Total 11]

PLEASE TURN OVER

(i)

Explain what is meant by the expectations theory for the shape of the yield
curve.
[2]

(ii)

Short-term, one-year annual effective interest rates are currently 8%; they are
expected to be 7% in one years time, 6% in two years time and 5% in three
years time.
(a)

Calculate the gross redemption yields (spot rates of interest) from


1-year, 2-year, 3-year and 4-year zero coupon bonds assuming the
expectations theory explanation of the yield curve holds.

(b)

The price of a coupon paying bond is calculated by discounting


individual payments from the bond at the zero-coupon bond yields
in (a).
Calculate the gross redemption yield of a bond that is redeemed at par
in exactly four years and pays a coupon of 5 per annum annually in
arrear.

(c)

A two-year forward contract has just been issued on a share with a


price of 400p. A dividend of 4p is expected in exactly one year.
Calculate the forward price using the above spot rates of interest,
assuming no arbitrage.
[12]
[Total 14]

10

An investor purchased a bond with exactly 15 years to redemption. The bond,


redeemable at par, has a gross redemption yield of 5% per annum effective. It pays
coupons of 4% per annum, half yearly in arrear. The investor pays tax at 25% on the
coupons only.
(i)

Calculate the price paid for the bond.

(ii)

After exactly eight years, immediately after the payment of the coupon then
due, this investor sells the bond to another investor who pays income tax at a
rate of 25% and capital gains tax at a rate of 40%. The bond is purchased by
the second investor to provide a net return of 6% per annum effective.

CT1 S2005

[3]

(a)

Calculate the price paid by the second investor.

(b)

Calculate, to one decimal place, the annual effective rate of return


earned by the first investor during the period for which the bond was
held.
[10]
[Total 13]

11

(i)

Explain what is meant by the following terms:


(a)
(b)

equation of value
discounted payback period from an investment project
[4]

(ii)

An insurance company is considering setting up a branch in a country in


which it has previously not operated. The company is aware that access to
capital may become difficult in twelve years time. It therefore has two
decision criteria. The cashflows from the project must provide an internal rate
of return greater than 9% per annum effective and the discounted payback
period at a rate of interest of 7% per annum effective must be less than twelve
years.
The following cashflows are generated in the development and operation of
the branch.
Cash Outflows
Between the present time and the opening of the branch in three years time the
insurance company will spend 1.5m per annum on research, development and
the marketing of products. This outlay is assumed to be a constant continuous
payment stream. The rent on the branch building will be 0.3m per annum
paid quarterly in advance for twelve years starting in three years time. Staff
costs are assumed to be 1m in the first year, 1.05m in the second year, rising
by 5% per annum each year thereafter. Staff costs are assumed to be incurred
at the beginning of each year starting in three years time and assumed to be
incurred for 12 years.
Cash Inflows
The company expects the sale of products to produce a net income at a rate of
1m per annum for the first three years after the branch opens rising to 1.9m
per annum in the next three years and to 2.5m for the following six years.
This net income is assumed to be received continuously throughout each year.
The company expects to be able to sell the branch operation 15 years from the
present time for 8m.
Determine which, if any, of the decision criteria the project fulfils.
[17]
[Total 21]

END OF PAPER

CT1 S2005

Faculty of Actuaries

Institute of Actuaries

EXAMINATION
September 2005

Subject CT1 Financial Mathematics


Core Technical
EXAMINERS REPORT

Faculty of Actuaries
Institute of Actuaries

Subject CT1 (Financial Mathematics Core Technical)

September 2005

Examiners Report

As is in some recent diets, the questions requiring descriptions of concepts, definitions or


verbal reasoning (such as Q1, Q8(ii) and Q9(i)) tended not to be well answered with
candidates producing vague statements which did not demonstrate that they understood the
relevant points. It is important that candidates understand the subject well enough to express
important topics and issues in their own words as well as in mathematical language. In show
that questions or questions where students are asked to derive formulae (such as Q8 part (i))
candidates are required to show detailed steps in deriving the results required in order to
obtain full marks.
Please note that differing answers may be obtained to those shown in these solutions
depending on whether figures obtained from tables or from calculators are used in the
calculations but candidates were not penalised for this. However, candidates were penalised
where excessive rounding had been used or where insufficient working had been shown.

Page 2

Subject CT1 (Financial Mathematics Core Technical)

September 2005

Examiners Report

One party agrees to pay to the other a regular series of fixed amounts for a certain
term. In exchange the second party agrees to pay a series of variable amounts based
on the level of a short term interest rate.

If f = the rate of inflation; j = the real rate of return and i = the money rate of return,
then j = (i f)/(1 + f). In this case, f = 2%, i= 1% and therefore j = 3.061%.

(a)

Let the answer be t days


1,500(1 + 0.05

t/365) = 1,550

t = 243.333 days
(b)

Let the answer be t days


1,500e0.05(t/365) = 1,550
0.05 (t/365) = ln (1,550/1500)
t = 239.366 days

210 200 exp

a bt 2 dt

200 exp at

5
1 bt 3
3
0

a bt 2 dt

200 exp at

10
1 bt 3
3
0

0
10

230 200 exp


0

200 5a 41.667b

200 10a 333.333b

ln(1.05) 5a 41.667b
ln(1.15) 10a 333.333b
The second expression less twice the first expression gives:

ln(1.15) 2 ln(1.05) 250b b 0.0001687


ln(1.15) 333.333 0.0001687
a
0.0083520
10

Page 3

Subject CT1 (Financial Mathematics Core Technical)

100
100
100

Examiners Report

(1 + 0.05/12) 12 10 = 60.716
(1 0.05/12)12 10 = 60.590
e 10 = 60.6531

(i)

(a)
(b)
(c)

(ii)

98.91 = 100(1 + i)
ln(1 + i) = (

September 2005

91/365

65/91)

ln(98.91/100) = 0.04396

therefore i = 0.04494

(i)
Used for medium or long-term borrowing
Unsecured
Regular annual coupon payments
Generally repayable at par
Generally issued by large companies and on behalf of governments
Yields depend on risk and marketability
Generally innovative market designed to attract different types of investor
Issued internationally (normally by a syndicate of banks)
Can be issued in any currency (not necessarily the domestic currency of
the borrower)

(ii)

(a)

97 = ga20 + 100v20 at 5% per annum effective


a20 = 12.4622; v20 = 0.37689 therefore 97 = 12.4622g + 100

0.37689
g = (97 37.689)/12.4622 = 4.75927
(b)

Duration = Ct tvt/ Ctvt where Ct is the amount of the cash flow at


time t
( Ia )20 = tvt Therefore duration of the eurobond is:

(4.75927 ( Ia )20 + 100

20v20)/(4.75927 a20 + 100v20)

( Ia )20 = 110.9506 all other values have been used in (a) above

therefore duration is:


(4.75927 110.9506 + 100 20 0.37689)/(4.75927
12.4622 + 100 0.37689) =1281.8239/97 = 13.2147

Page 4

Subject CT1 (Financial Mathematics Core Technical)

(i)

September 2005

Examiners Report

Value of loan = 50v + 48v2 + 46v3 + 44v4 + + 22v15


= 52(v +v2 +v3 + + v14 + v15)

2(v + 2v2 +4v3 +

+ 28v14+30 v15)

= 52 a15 - 2 ( Ia )15
( Ia )15 = 67.2668

a15 = 9.7122
Therefore amount of the loan is 52

9.7122 - 2

67.2668 = 370.501

Candidates who derived an appropriate formula for a decreasing annuity directly or


who calculated the value of the loan by summing the individual terms received full
credit.

(ii)

Interest component in first year is 0.06 370.504 = 22.23024; therefore


capital component is 50 22.23024 = 27.76976.
Capital remaining after first instalment is 370.504 27.76976 = 342.73424.
Interest paid in second instalment is 0.06 342.73424 = 20.56405
Capital in second instalment is 48

(iii)

20.56405 = 27.43595.

At the end of the thirteenth year, the capital outstanding is:


24v + 22v2 = 24

0.94340 + 22

0.89000 = 42.2216

The interest due in the fourteenth instalment 0.06

42.2216 = 2.53330

The capital payment is therefore 24 2.53330 = 21.46670

Page 5

Subject CT1 (Financial Mathematics Core Technical)

(i)

September 2005

Examiners Report

Let it be the (random) rate of interest in year t . Let S5 be the accumulation of


a single investment of 1 unit after 5 years:
5

E S5

1 it
t 1

E 1 it
t 1

as it are independent

E S5

E 1 it

1 E it = 1.035

E 1 it
E S5

1.035

1.187686

E S52

1 it
t 1

E 1 it

E S52

2 5

it2

E 1 2it

Var it
E S5

1 2 E it
E it

(using independence)

t 1

1 2 E it
Var S5

E 1 it

E it

1 2 E it

E it2

2 5

Var it

E it

2 5

E 1 it

10

0.035

Var it

0.032

Var S5

1 2 0.035 0.032 0.0352

1.035

10

1.416534 1.410598
0.0059356

Mean value of the accumulation of premiums is:


425000 E S5 425000(1.03)5 425000 1.187686

425000 1.15927

997458

Standard deviation is 425000 SD S5

425000

0.0059356

32743.21

Candidates who obtained slightly different answers by first deriving the parameters of
the lognormal distribution received full credit.

Page 6

Subject CT1 (Financial Mathematics Core Technical)

September 2005

Examiners Report

(ii)

Investing all premiums in the risky assets is likely to be more risky because,
although there may be a higher probability of the assets accumulating to more
than 1 million, the standard deviation would be twice as high so the
probability of a large loss would be greater.

(i)

Bond yields are determined by investors expectations of future short-term


interest rates, so that returns from longer-term bonds reflect the returns from
making an equivalent series of short-term investments

(ii)

(a)

Let it be the spot yield over t years:


One year: yield is 8% therefore i1 = 0.08
two years: (1 + i2)2 = 1.08 1.07 therefore i2 = 0.074988
three years: (1 + i3)3 = 1.08 1.07 1.06 therefore i3 = 0.06997
four years: (1 + i4)4 = 1.08 1.07 1.06 1.05 therefore i4 = 0.06494

(b)

Price of the bond is 5[(1.08) 1 + (1.074988) 2 + (1.06997) 3]


+ 105 (1.06494) 4 = 13.03822 + 81.6373 = 94.67552
Find gross redemption yield from
94.67552 = 5 a4 + 100v4
try 7%; a4 = 3.3872; v4 = 0.76290
gives RHS = 93.226
GRY must be lower, try 6%; a4 = 3.4651; v4 = 0.79209
gives RHS = 96.5345
interpolate between 6% and 7%.
i = 0.07 0.01 (94.67552 93.226)/(96.5345
i = 0.07 0.0043812 = 0.06562

(c)

93.226)

Present value of the dividend is 4v calculated at 8% per annum


effective = 3.70370.
Therefore forward price is
F = (400 3.70370) 1.08

1.07 = 457.9600

Page 7

Subject CT1 (Financial Mathematics Core Technical)

10

(i)

September 2005

Examiners Report

Price paid by first investor is P1


4a (2)

P1
i
i

100v15

15 5%

(2)

1.012348

v15

0.48102

a15

10.3797

P1

4 1.012348 10.3797

100 0.48102

42.0315 48.1020 90.1335

(ii)

(a)

1.06

g 1 t1
i

0.059126

0.04 0.75 0.03


1 t1 g

Capital gain on contract

Price paid by second investor is P2


P2

0.75 4a (2)

7 6%

7
P2 1 0.4v6%

i
i

(2)

0.66506

a7

5.5824

7 6%

7
0.6 100v6%

0.75 4 1.014782 5.5824

77.5207

Page 8

0.75 4a (2)

1.014782

v7

P2

7
7
100v6%
0.4 100 P2 v6%

1 0.4 0.66506

60 0.66506

Subject CT1 (Financial Mathematics Core Technical)

(b)

September 2005

Examiners Report

Rate of return earned by the first investor is the solution to:

90.1335 0.75 4a (2) 77.5207v8


8

2%
i

(2)

1.004975

v8

0.85349

a8

7.3255

RHS

88.2490

i 1.5%
i
i

(2)

1.003736

v8

0.88771

a8

7.4859

RHS
i

11

(i)

91.3575
90.1335 88.2490
0.02
91.3575 88.2490

0.005 1.697% 1.7%

(a)

An equation of value expresses the equality of the present value of


positive and negative (or incoming and outgoing) cash flows that are
connected with an investment project, investment transaction etc.

(b)

The discounted payback period from an investment project is the first


time at which the net present value of the cash flows from the project is
positive.

Page 9

Subject CT1 (Financial Mathematics Core Technical)

(ii)

September 2005

Examiners Report

Consider first the NPV at 9% per annum effective. Working in million.


Present value of cash outflows:
3
3
4
5
1.5a3 9% 0.3a (4) v9%
v9%
1.05v9%
1.052 v9%
12 9%

1.0511 v14
9%

1.5 1.044354 2.5313 0.3 1.055644 7.1607 0.77218


0.77218

1 1.0512 v12
1 1.05v

5.71647 7.60679 13.32326

Present value of cash inflows:


a6 9% a3 9%
2.5a15

1.9 a9 9% a6 9%

0.6a9

0.9a6

a3

8v15
9%

2.5 a15 9% a9 9%

8v15

1.044354 2.5 8.0607 0.6 5.9952 0.9 4.4859 2.5313

8 0.27454

12.6253

Hence NPV of project @ 9% = 12.6253


so the IRR is less than 9% p.a. effective

13.3233 = 0.698 million

To find whether the discounted payback period is less than 12 years at 7% per
annum effective, we need to find the NPV @ 7% of first twelve years
cashflows
Present value of cash outflows:
3
3
4
5
1.5a3 7% 0.3a (4) v7%
v7%
1.05v7%
1.052 v7%
9 7%

1.058 v11
7%

1.5 1.034605 2.6243 0.3 1.043380 6.5152 0.81630


0.81630

Page 10

1 1.059 v9
1 1.05v

5.73739 6.82096 12.55835

Subject CT1 (Financial Mathematics Core Technical)

September 2005

Examiners Report

Present value of cash inflows:

a6 7% a3 7%
2.5a12

0.6a9

1.9 a9 7% a6 7%
0.9a6

2.5 a12 7% a9 7%

a3

1.034605 2.5 7.9427 0.6 6.5152 0.9 4.7665 2.6243


9.3461

NPV is negative so the discounted payback period is more than 12 years.


Project fulfils neither the discounted payback period criterion nor the internal
rate of return criterion.

END OF EXAMINERS REPORT

Page 11

Faculty of Actuaries

Institute of Actuaries

EXAMINATION
4 April 2006 (am)

Subject CT1
Financial Mathematics
Core Technical
Time allowed: Three hours
INSTRUCTIONS TO THE CANDIDATE
1.

Enter all the candidate and examination details as requested on the front of your answer
booklet.

2.

You must not start writing your answers in the booklet until instructed to do so by the
supervisor.

3.

Mark allocations are shown in brackets.

4.

Attempt all 12 questions, beginning your answer to each question on a separate sheet.

5.

Candidates should show calculations where this is appropriate.

Graph paper is not required for this paper.

AT THE END OF THE EXAMINATION


Hand in BOTH your answer booklet, with any additional sheets firmly attached, and this
question paper.
In addition to this paper you should have available the 2002 edition of the
Formulae and Tables and your own electronic calculator.

CT1 A2006

Faculty of Actuaries
Institute of Actuaries

An investment is discounted for 28 days at a simple rate of discount of 4.5% per


annum. Calculate the annual effective rate of interest.

An annuity certain with payments of 150 at the end of each quarter is to be replaced
by an annuity with the same term and present value, but with payments at the
beginning of each month instead.
Calculate the revised payments, assuming an annual force of interest of 10%.

[3]

[3]

At time t = 0 the n-year spot rate of interest is equal to (2.25 + 0.25n)% per annum
effective (1 n 5).
(a)

Calculate the 2-year forward rate of interest from time t = 3 expressed as an


annual effective rate of interest.

(b)

Calculate the 4-year par yield.

(c)

Without performing any further calculations, explain how you would expect
the gross redemption yield of a 4-year bond paying annual coupons of 3.5% to
compare with the par yield calculated in (b).
[7]

An investor, who is liable to income tax at 20% but is not liable to capital gains tax,
wishes to earn a net effective rate of return of 5% per annum. A bond bearing
coupons payable half-yearly in arrear at a rate 6.25% per annum is available. The
bond will be redeemed at par on a coupon date between 10 and 15 years after the date
of issue, inclusive. The date of redemption is at the option of the borrower.
Calculate the maximum price that the investor is willing to pay for the bond.

[5]

A share currently trades at 10 and will pay a dividend of 50p in one month s time. A
six-month forward contract is available on the share for 9.70. Show that an investor
can make a risk-free profit if the risk-free force of interest is 3% per annum.
[4]

An actuarial student has created an interest rate model under which the annual
effective rate of interest is assumed to be fixed over the whole of the next ten years.
The annual effective rate is assumed to be 2%, 4% and 7% with probabilities 0.25,
0.55 and 0.2 respectively.
(a)

Calculate the expected accumulated value of an annuity of 800 per annum


payable annually in advance over the next ten years.

(b)

Calculate the probability that the accumulated value will be greater than
10,000.
[4]

CT1 A2006 2

A company has entered into an interest rate swap. Under the terms of the swap the
company makes fixed annual payments equal to 6% of the principal of the swap. In
return, the company receives annual interest payments on the principal based on the
prevailing variable short-term interest rate which currently stands at 5.5% per annum.
(a)

Describe briefly the risks faced by a counterparty to an interest rate swap.

(b)

Explain which of the risks described in (a) are faced by the company.

[4]

An ordinary share pays annual dividends. A dividend of 25p per share has just been
paid. Dividends are expected to grow by 2% next year and by 4% the following year.
Thereafter, dividends are expected to grow at 6% per annum compound in perpetuity.
(i)

State the main characteristics of ordinary shares.

[4]

(ii)

Calculate the present value of the dividend stream described above at a rate of
interest of 9% per annum effective from a holding of 100 ordinary shares. [4]

(iii)

An investor buys 100 shares in (ii) for 8.20 each. He holds them for two
years and receives the dividends payable. He then sells them for 9
immediately after the second dividend is paid.
Calculate the investor s real rate of return if the inflation index increases by
3% during the first year and by 3.5% during the second year assuming
dividends grow as expected.
[4]
[Total 12]

The force of interest (t ) is a function of time and at any time t, measured in years, is
given by the formula:

0.04
(t )

0 t

0.008t
0.005t 0.0003t

5 t 10
10 t

(i)

Calculate the present value of a unit sum of money due at time t = 12.

[5]

(ii)

Calculate the effective annual rate of interest over the 12 years.

[2]

(iii)

Calculate the present value at time t = 0 of a continuous payment stream


that is paid at the rate of e 0.05t per unit time between time t = 2 and time
t = 5.
[3]
[Total 10]

CT1 A2006 3

PLEASE TURN OVER

10

A piece of land is available for sale for 5,000,000. A property developer, who can
lend and borrow money at a rate of 15% per annum, believes that she can build
housing on the land and sell it for a profit. The total cost of development would be
7,000,000 which would be incurred continuously over the first two years after
purchase of the land. The development would then be complete.
The developer has three possible project strategies. She believes that she can sell the
completed housing:
in three years time for 16,500,000
in four years time for 18,000,000
in five years time for 20,500,000
The developer also believes that she can obtain a rental income from the housing
between the time that the development is completed and the time of sale. The rental
income is payable quarterly in advance and is expected to be 500,000 in the first year
of payment. Thereafter, the rental income is expected to increase by 50,000 per
annum at the beginning of each year that the income is paid.
(i)

Determine the optimum strategy if this is based upon using net present value
as the decision criterion.
[9]

(ii)

Determine which strategy would be optimal if the discounted payback period


were to be used as the decision criterion.
[2]

(iii)

If the housing is sold in six years time, the developer believes that she can
obtain an internal rate of return on the project of 17.5% per annum. Calculate
the sale price that the developer believes that she can receive.
[6]

(iv)

Suggest reasons why the developer may not achieve an internal rate of return
of 17.5% per annum even if she sells the housing for the sale price calculated
in (iii).
[2]
[Total 19]

CT1 A2006 4

11

An actuarial student has taken out two loans.


Loan A: a five-year car loan for 10,000 repayable by equal monthly instalments of
capital and interest in arrear with a flat rate of interest of 10.715% per
annum.
Loan B: a five-year bank loan of 15,000 repayable by equal monthly instalments of
capital and interest in arrear with an effective annual interest rate of 12% for
the first two years and 10% thereafter.
The student has a monthly disposable income of 600 to pay the loan interest after all
other living expenses have been paid.
Freeloans is a company which offer loans at a constant effective interest rate for all
terms between three years and ten years. After two years, the student is approached
by a representative of Freeloans who offers the student a 10-year loan on the capital
outstanding which is repayable by equal monthly instalments of capital and interest in
arrear. This new loan is used to pay off the original loans and will have repayments
equal to half the original repayments.
(i)

Calculate the final disposable income (surplus or deficit) each month after the
loan payments have been made.
[5]

(ii)

Calculate the capital repaid in the first month of the third year assuming that
the student carries on with the original arrangements.
[5]

(iii)

Estimate the capital repaid in the first month of the third year assuming that
the student has taken out the new loan.
[5]

(iv)

Suggest, with reasons, a more appropriate strategy for the student.

CT1 A2006 5

[2]
[Total 17]

PLEASE TURN OVER

12

A pension fund has liabilities of 3 million due in 3 years time, 5 million due in 5
years time, 9 million due in 9 years time, and 11 million due in 11 years time.
The fund holds two investments, X and Y. Investment X provides income of 1
million payable at the end of each year for the next five years with no capital
repayment. Investment Y is a zero coupon bond which pays a lump sum of R at the
end of n years (where n is not necessarily an integer). The interest rate is 8% per
annum effective.
(i)

Investigate whether values of R and n can be found which ensure that the
fund is immunised against small changes in the interest rate.
5

t 2 vt

You are given that

40.275 at 8%.

[8]

t 1

(ii)

(a)

The interest rate immediately changes to 3% per annum effective.


Calculate the revised present values of the assets and liabilities of the
fund.

(b)

Explain your answer to (ii)(a).

END OF PAPER

CT1 A2006 6

[4]
[Total 12]

Faculty of Actuaries

Institute of Actuaries

EXAMINATION
April 2006

Subject CT1
Financial Mathematics
Core Technical
EXAMINERS REPORT
Introduction
The attached subject report has been written by the Principal Examiner with the aim of
helping candidates. The questions and comments are based around Core Reading as the
interpretation of the syllabus to which the examiners are working. They have however given
credit for any alternative approach or interpretation which they consider to be reasonable.

M Flaherty
Chairman of the Board of Examiners
June 2006

Comments
Individual comments are shown after each question.

General comments
As is in some recent diets, the questions requiring verbal reasoning (such as Q3(c), Q7(b),
Q10(iv) and Q11(iv)) tended not to be well answered with candidates producing vague
statements which did not demonstrate that they understood the relevant points.
Please note that different answers may be obtained to those shown in these solutions
depending on whether figures obtained from tables or from calculators are used in the
calculations but candidates are not penalised for this.
However, candidates may be penalised where excessive rounding has been used or where
insufficient working is shown.

Faculty of Actuaries
Institute of Actuaries

Subject CT1 (Financial Mathematics Core Technical)

28d
Annual rate of interest is i where 1
365

This gives i

April 2006

1 i

Examiners Report

28 / 365

365 / 28

28 0.045
365

1 4.611%

Comments on question 1: This was generally well answered.

We require X where:

600a (4)
n

d (12) 12 1

i (4)

a (4)

12 Xa (12)
n

1 d

4 1 i

Hence X

1/ 4

50

1/12

n
(12)
a
n

50

12 1 e

4 e

d (12)
i (4)
0.099584

12

0.101260

49.1724 or 49.17

Comments on question 2: Candidates were not penalised for assuming that the annuities
were for a specific term even though this was not needed for the calculations.

(a)

(b)

f3,2

1 y5
1 y3

yc4

Page 2

1.035
1.03

0.12009
3.71785

1.0275

f3,2

Par yield is yc4 where yc4 v y1


Thus yc4 1.025

(c)

v 2y2

1.03

v3y3

4.255%

v 4y4

1.0325

v 4y4
4

1.0325

3.230%

The par yield is equal to the gross redemption yield for a par yield bond.
Coupons for the 3.5% bond are higher than for the par yield bond. Thus a
lower proportion of the total proceeds are included within the redemption
payment which is when spot yields/discount rates are highest. The present
value of the proceeds of the 3.5% bond will be higher and so the gross
redemption yield will be lower than that of the par yield bond and thus less
than the par yield.

Subject CT1 (Financial Mathematics Core Technical)

April 2006

Examiners Report

Comments on question 3: Part (a) was answered well but some candidates struggled with
the calculation of the par yield in part (b). In part (c) the marks were awarded for a clear
explanation. Many candidates, who just stated their conclusion, were unable to explain their
reasoning clearly and so failed to score full marks on this part.

0.049390

g 1 t1

0.0625 0.80 0.05

1 t1 g

Capital loss on contract


Assume redeemed as early as possible (i.e.: after 10 years) to obtain minimum
yield.
Price of stock per 100 nominal, P:
P

2
10

100 0.0625 0.80 a


P

2
10

5a

100v10at 5%

100v10

5 1.012348 7.7217

100 0.61391

39.0852 61.3910 100.4762

Comments on question 4: Well answered although some candidates who recognised that the
investor faced a capital loss did not recognise that this meant that the minimum yield would
be obtained if the bond was redeemed at the earliest possible date.

Page 3

Subject CT1 (Financial Mathematics Core Technical)

April 2006

Examiners Report

An investor can borrow 10 at the risk-free rate, buy one share for 10, enter into the
forward contract to sell the share in six months time.
The initial cashflow is zero.
After one month the 50p dividend from the share is invested at the risk-free rate. After
six months the share can be sold for 9.70, the dividend proceeds are worth

0.5e

0.03

+ 0.5e

5
12

0.03

and the borrowing is repaid at 10 e0.015 . This gives a net cashflow of 9.7

5
12

10 e0.015 = 0.0552

The investor has made a deal with zero initial cost, no risk of future loss and a riskfree future profit.
Comments on question 5: The majority of candidates were able to calculate the nonarbitrage forward price by use of the appropriate formula. However, marks were lost for not
clearly explaining how a risk-free profit could thus be made.

(a)

Expected accumulated value


800 0.25s10 0.02 0.55s10 0.04 0.2 s10 0.07
800 0.25 s11 0.02 1

0.55 s11 0.04 1

0.2 s11 0.07 1

800 0.25 11.1687

0.55 12.4864

0.2 14.7836

0.25 8934.96

0.55 9989.12

0.2 11826.88

10, 093.13

(b)

Accumulation is only over 10,000 if the interest rate is 7% p.a. which has
probability 0.2

Comments on question 6: The most poorly answered question on the paper. This model of
interest rates had not been examined recently and the majority of candidates assumed instead
that the interest rate changed each year (in line with previous examination questions on this
topic).

Page 4

Subject CT1 (Financial Mathematics Core Technical)

(a)

April 2006

Examiners Report

The counterparty faces market risk which is the risk that market conditions
will change so that the present value of the net outgo under the agreement
increases.
The counterparty also faces credit risk which is the risk that the other
counterparty will default on its payments.

(b)

The company still faces the market risk since the interest rates could fall
further which will make the value of the swap even more negative to the
company.
The company does not currently face a credit risk since the value of the swap
is positive to the other counterparty.

Comments on question 7: Part (a) was answered well but many candidates failed to
recognise in (b) that the company would not currently face credit risk in this example.

(i)

Main characteristics of ordinary shares:


Issued by commercial undertakings and other bodies.
Entitle holders to receive all net profits of the company in the form of
dividends after interest on loans and other fixed interest stocks has been
paid.
Higher expected returns than for most other asset classes
but risk of capital losses
and returns can be variable.
Lowest ranking form of finance.
Low initial running yield but dividends should increase with inflation.
Marketability varies according to size of company.
Voting rights in proportion to number of shares held.

(ii)

Present value of future dividends


100 0.25 1.02v 1.02 1.04v 2 1.02 1.04 1.06v3 1.02 1.04 1.062 v 4

25 1.02v 25 1.02 1.04v 2 1 1.06v 1.062 v 2

25 1.02v 25 1.02 1.04v 2

1.09
0.03

23.3945 811.0092 834.4037

834.40

Page 5

Subject CT1 (Financial Mathematics Core Technical)

(iii)

April 2006

Examiners Report

Real rate of return is i such that:

820 100 0.25 1.02

100
100 100 2
v 100 0.25 1.02 1.04
v
103
103 103.5

100 100 2
v
103 103.5

900

24.7573v 869.1150v 2

24.75732 4 869.1150 820


v
2 869.1150
(taking positive root)
24.7573

0.95719

Hence i = 4.47%
Comments on question 8: Despite being a bookwork question, part (i) was answered patchily
with few students getting all of the required points. Part (ii) was answered well. In part (iii),
it was expected that students would solve the quadratic equation. However, full credit was
given to students who used interpolation methods.

(i)

A(0,5) e 0

A(5,10) e

0.04 dt

0.04t

10
0.008tdt
5

A(10,12) e

12
10

5
0

e0.2 1.22140

0.004t 2

0.005t 0.0003t 2 dt

10

e0.3 1.34986

0.0025t 2 0.0001t 3

12
10

e0.1828 1.20057

Required present value


1
A 0,5 A 5,10 A 10,12

1
1.22140 1.34986 1.20057

1
1.97941

= 0.50520
(ii)

Page 6

Equivalent effective annual rate is i where 1 i

12

1.97941

5.855%

Subject CT1 (Financial Mathematics Core Technical)

(iii)

April 2006

Examiners Report

Present Value at time t = 0


5

0.05t

t
0.04 ds
0

dt

0.05t

0.04t

dt

0.09t

dt

0.09t 5

0.09

0.18

0.45

e
0.09

2.1960

Comments on question 9: Well answered.

10

(i)

Net present value of costs

5, 000, 000 3,500, 000a2

i
5, 000, 000 3,500, 000 a2

5, 000, 000 3,500, 000 1.073254 1.6257 11,106, 762

Net present value of benefits


4
n 2

450, 000v 2 a

450, 000v 2

i
d

(4)

50, 000v 2 Ia

an

(4)
n 2

50, 000v 2

i
d

(4)

Sn v n

Ia

n 2

Sn v n

where n is the year of sale and Sn are the sale proceeds if the sale is made in
year n.
If n = 3 the NPV of benefits
450, 000 0.75614 1.092113 0.86957
50, 000 0.75614 1.092113 0.86957
16,500, 000 0.65752
323,137 35,904 10,849, 080 11, 208,121
Hence net present value of the project is 11,208,121 11,106,762 = 101,359
Note that if n = 4 the extra benefits in year 4 consist of an extra 1.5 million
on the sale proceeds and an extra 650,000 rental income. This is clearly less
than the amount that could have been obtained if the sale had been made at the
end of year 3 and the proceeds invested at 15% per annum. Hence selling in
year 4 is not an optimum strategy.

Page 7

Subject CT1 (Financial Mathematics Core Technical)

April 2006

Examiners Report

If n = 5 the NPV of benefits


450, 000 0.75614 1.092113 2.2832
50, 000 0.75614 1.092113 4.3544
20,500, 000 0.49718
848, 450 179, 791 10,192,190 11, 220, 431
Hence net present value of the project is 11,220,431 11,106,762 = 113,669
Hence the optimum strategy if net present value is used as the criterion is to
sell the housing after 5 years.
(ii)

If the discounted payback period is used as the criterion, the optimum strategy
is that which minimises the first time when the net present value is positive.
By inspection, this is when the housing is sold after 3 years.

(iii)

We require

i
5, 000, 000 3,500, 000 a2

4
n 2

450, 000v 2 a

LHS 5, 000, 000 3,500, 000

2
1 v0.175

50, 000v 2 Ia

(4)
n 2

Sn v n at 17.5%

5, 000, 000 3,500, 000

0.175

1 0.72431
0.16127

10,983, 227
2
450, 000v0.175

RHS

d 0.175

a4

41 v

1 v4
d

4
1 v0.175

2
50, 000v0.175

a4

4
4v0.175

6
S6v0.175

0.15806

3.1918

Therefore we have on the RHS


450, 000 0.72431 3.0076 50, 000 0.72431

980, 296 250,502 0.37999 S6

Page 8

3.1918 2.0985
0.15806

0.37999 S6

Subject CT1 (Financial Mathematics Core Technical)

10,983, 227 1, 230, 798


0.37999

For equality S6
(iv)

April 2006

Examiners Report

25, 665, 000

Reasons investor may not achieve the internal rate of return:


Allowance for expenses when buying/selling which may be significant.
There may be periods when the property is unoccupied and no rental
income is received.
Rental income may be reduced by maintenance expenses.
Tax on rental income and/or sale proceeds

Comments on question 10: A significant number of candidates assumed that the development
costs amounted to 7 million per annum and subsequently found that no strategy would lead
to a profit. Otherwise the calculations were performed well. In part (iv), credit was given for
other valid answers. Despite this, few students scored full marks on this part.

11

(i)

Let X A , X B be the monthly repayments under Loans A and B respectively.


For loan A:
Flat rate of interest = 10.715%
60 X A LA 60 X A 10000
5 LA
50000

XA

255.96

For loan B:

LB

15000 12 X B a (12)

3 10%

1, 250

XB

i
i

12

2
v12%

a2
12%

1.053875 1.6901
XB

2
v12%
a (12)

2 12%

i
i

12

a3
10%

1, 250
0.79719 1.045045 2.4869

324.43

Hence student s overall surplus = 600

XA

X B = 19.61

Page 9

Subject CT1 (Financial Mathematics Core Technical)

(ii)

April 2006

Examiners Report

Effective rate of interest under loan A is i % where


12
5

10000

12
5

= 3.2557

12 255.96a

Try i = 20%: a

12
5

3.2557

12
3

So capital outstanding after 24 months is 12 255.96 a

at 20%

12 255.96 1.088651 2.1065 7043.74


12
3

Capital outstanding under B is 12 324.43 a

at 10%

12 324.43 1.045045 2.4869 10118.02

So interest paid in month 25 under loans A and B


12

7043.74

12

i 20%

10118.02

12

i10%
12

107.84 80.68 188.52

and capital repaid


255.96 107.84

(iii)

324.43 80.68

148.12 243.75 391.87

Under the new loan the capital outstanding is the same as under the original
arrangement = 17161.76.
The monthly repayment

255.96 324.43
2

290.20

The effective rate of interest on the new loan A is i where


12
10

17161.76

12
10

4.5642

12
10

5.3551

12 290.20a

Try i = 20%: a

Try i = 15%: a

By interpolation i 15%

Page 10

12
10

4.9281

5.3551 4.9281
5.3551 4.5642

20% 15%

17.7%

Subject CT1 (Financial Mathematics Core Technical)

April 2006

Examiners Report

Hence interest paid in month 25


12

17161.76

i17.7%

234.66

12

and capital repaid is 290.20


(iv)

234.66 = 55.54

The new strategy reduces the monthly payments but repays the capital more
slowly. The student could consider the following options:
Keeping loan B and taking out a smaller new loan to repay loan A
(which has the highest effective interest rate).
Taking out the new loan for a shorter term to repay the capital more
quickly.

Comments on question 11: In part (i) some candidates struggled to deal with the flat rate of
Loan A whilst others failed to deal with the change in interest rate of Loan B. Part (ii) was
answered well. In part (iii), different answers for the effective rate of interest (and hence the
interest paid) for the new loan could be obtained according to the actual interpolation used
and full credit was given for a range of answers. If calculated exactly, the effective rate of
interest is actually 17.5%. In part (iv), credit was again given for any valid strategy suitably
explained.

12

(i)

We will consider three conditions necessary for immunisation


(1)

VA

VL (all expressions in terms of m)

VA

a5

Rv n at 8%

3.9927 Rv n
VL

3v 3 5v 5 9v 9 11v11 at 8%
15.0044

Rv n
(2)

VA'
VA'

11.0117

VL' where VA'


Ia

VA

& VL'

VL

nRv n

11.3651 nRv n

VL'

9v3 25v5 81v9 121v11


116.5741

Page 11

Subject CT1 (Financial Mathematics Core Technical)

April 2006

nRv n 105.2090
105.2090
n
9.5543
11.0117
R 11.0117

9.5543

1.08

22.9720m

Alternatively:

VA'

VA
& VL'
i

VL' where VA'

VA'

v Ia

nRv n

11.3651v nRv n
10.5233 nRv n

VL'

VL
i

9v 4 25v 6 81v10 121v12


107.9389

nRv n

97.4156

97.4156
11.0117v

R 11.0117

9.5543
1.08

9.5543

22.9720m

(3)

VA''

VL'' (where VA''

VA
2

& VL''

VL
2

VA''

t 2vt

n 2 Rv n

t 1

40.275

9.5543

22.9720 v 9.5543

1045.483

VL''

27v 3 125v 5 729v 9 1331v11


1042.031

Page 12

Examiners Report

Subject CT1 (Financial Mathematics Core Technical)

April 2006

Examiners Report

Alternatively (differentiating with respect to i):


5

VA''

t t 1 vt

n n 1 Rv n

t 1
5

v2

t 2vt

v 2 Ia

n n 1 Rv n

t 1

0.85734 40.275 0.85734 11.3651 9.5543 10.5543 22.9720 v11.5543


34.53 9.74 952.00 996.27

VL''

3 3 4 v5 5 5 6 v 7 9 9 10 v11 11 11 12 v13
993.32

Thus n 9.5543, R 22.9720m will satisfy all three conditions and so will
achieve immunisation.
(ii)

(a)

Value of assets at 3%

a5

Rv n
3

4.5797 22.9720v 9.5543


5

21.900m

11

Value of liabilities at 3% = 3v 5v 9v 11v


21.903m
Hence fund has a deficit of approximately 3,000.
(b)

Immunisation will only enable to be a fund to be protected against a


small change in interest rates. It will not be necessarily protected
against sudden large changes as in this case.

Comments on question 12: Part (i) was answered surprisingly poorly, given that it required
the same techniques as those required in previous examination questions on the same topic.
Full credit was given to students who observed directly that the spread of the assets around
the mean term was greater than the spread of the liabilities. Few students answered part (ii)
fully and the examiners felt that students should have recognised that immunisation would
not protect the fund against such a large change in interest rates even if they had not
answered part (i) correctly.

END OF EXAMINERS REPORT

Page 13

Faculty of Actuaries

Institute of Actuaries

EXAMINATION
12 September 2006 (am)

Subject CT1
Financial Mathematics
Core Technical
Time allowed: Three hours
INSTRUCTIONS TO THE CANDIDATE
1.

Enter all the candidate and examination details as requested on the front of your answer
booklet.

2.

You must not start writing your answers in the booklet until instructed to do so by the
supervisor.

3.

Mark allocations are shown in brackets.

4.

Attempt all 12 questions, beginning your answer to each question on a separate sheet.

5.

Candidates should show calculations where this is appropriate.

Graph paper is not required for this paper.

AT THE END OF THE EXAMINATION


Hand in BOTH your answer booklet, with any additional sheets firmly attached, and this
question paper.
In addition to this paper you should have available the 2002 edition of the
Formulae and Tables and your own electronic calculator.

CT1 S2006

Faculty of Actuaries
Institute of Actuaries

(a)
(b)

Distinguish between a future and an option.


Explain why convertibles have option-like characteristics.
[3]

An individual makes an investment of 4m per annum in the first year, 6m per


annum in the second year and 8m per annum in the third year. The investments are
made continuously throughout each year. Calculate the accumulated value of the
investments at the end of the third year at a rate of interest of 4% per annum effective.
[3]

An individual has invested a sum of 10m. Exactly one year later, the investment is
worth 11.1m. An index of prices has a value of 112 at the beginning of the
investment and 120 at the end of the investment. The investor pays tax at 40% on all
money returns from investment. Calculate:
(a)
(b)
(c)

The money rate of return per annum before tax.


The rate of inflation.
The real rate of return per annum after tax.
[4]

An investor is able to purchase or sell two specially designed risk-free securities, A


and B. Short sales of both securities are possible. Security A has a market price of
20p. In the event that a particular stock market index goes up over the next year, it
will pay 25p and, in the event that the stock market index goes down, it will pay 15p.
Security B has a market price of 15p. In the event that the stock market index goes up
over the next year, it will pay 20p and, in the event that the stock market index goes
down, it will pay 12p.
(i)

Explain what is meant by the assumption of no arbitrage used in the pricing


of derivative contracts.
[2]

(ii)

Find the market price of B, such that there are no arbitrage opportunities and
assuming the price of A remains fixed. Explain your reasoning.
[2]
[Total 4]

(i)

Calculate the time in days for 3,600 to accumulate to 4,000 at:


(a)
(b)
(c)

a simple rate of interest of 6% per annum


a compound rate of interest of 6% per annum convertible quarterly
a compound rate of interest of 6% per annum convertible monthly
[4]

(ii)

CT1 S2006

Explain why the amount takes longest to accumulate in (i)(a)

[1]
[Total 5]

The rate of interest is a random variable that is distributed with mean 0.07 and
variance 0.016 in each of the next 10 years. The value taken by the rate of interest in
any one year is independent of its value in any other year. Deriving all necessary
formulae calculate:
(i)

The expected accumulation at the end of ten years, if one unit is invested at the
beginning of ten years.
[3]

(ii)

The variance of the accumulation at the end of ten years, if one unit is invested
at the beginning of ten years.
[5]

(iii)

Explain how your answers in (i) and (ii) would differ if 1,000 units had been
invested.
[1]
[Total 9]

A life insurance fund had assets totalling 600m on 1 January 2003. It received net
income of 40m on 1 January 2004 and 100m on 1 July 2004. The value of the fund
was:
450m on 31 December 2003;
500m on 30 June 2004;
800m on 31 December 2004.
(i)

Calculate, for the period 1 January 2003 to 31 December 2004, to three


decimal places:
(a)

The time weighted rate of return per annum.

(b)

The linked internal rate of return, using sub intervals of a calendar


year.
[8]

(ii)

Explain why the linked internal rate of return is higher than the time weighted
rate of return.
[2]
[Total 10]

The force of interest (t ) at time t is at bt 2 where a and b are constants. An amount


of 100 invested at time t = 0 accumulates to 150 at time t = 5 and 230 at time
t = 10.
(i)

Calculate the values of a and b.

[5]

(ii)

Calculate the constant force of interest that would give rise to the same
accumulation from time t = 0 to time t = 10.

[2]

(iii)

CT1 S2006

At the force of interest calculated in (ii), calculate the present value of a


continuous payment stream of 20e0.05t paid between from time t = 0 to time
t = 10.
[4]
[Total 11]

PLEASE TURN OVER

10

An individual took out a loan of 100,000 to purchase a house on 1 January 1980.


The loan is due to be repaid on 1 January 2010 but the borrower can repay the loan
early if he wishes. The borrower pays interest on the loan at a rate of 6% per annum
convertible monthly, paid in arrears. The loan instalments only cover the interest on
the loan. At the same time, the borrower took out a thirty-year investment policy,
which was expected to repay the loan, and into which monthly premiums were paid,
in advance, at a rate of 1,060 per annum. The individual was told that premiums in
the investment policy were expected to earn a rate of return of 7% per annum
effective. After twenty years, the individual was informed that the premiums had
only earned a rate of return of 4% per annum effective and that they would continue
to do so for the final ten years of the policy. The borrower agrees to increase his
monthly payments into the investment policy to 5,000 per annum for the final ten
years.
(a)

Calculate the amount to which the investment policy was expected to


accumulate at the time it was taken out.

(b)

Calculate the amount by which the investment policy would have fallen short
of repaying the loan had extra premiums not been paid for the final ten years.

(c)

Calculate the amount of money the individual will have, after using the
proceeds of the investment policy to repay the loan, after allowing for the
increase in premiums.

(d)

Suggest another course of action the borrower could have taken which would
have been of higher value to him, explaining why this higher value arises.

(e)

Calculate the level annual instalment that the investor would have had to pay
from outset if he had repaid the loan in equal instalments of interest and
capital.
[11]

A financial regulator has brought in a new set of regulations and wishes to assess the
cost of them. It intends to conduct an analysis of the costs and benefits of the new
regulations in their first twenty years.
The costs are estimated to be as follows:
The cost to companies who will need to devise new policy terms and computer
systems is expected to be incurred at a rate of 50m in the first year increasing by
3% per annum over the twenty year period.
The cost to financial advisers who will have to set up new computer systems and
spend more time filling in paperwork is expected to be incurred at a rate of 60m
in the first year, 19m in the second year, 18m in the third year, reducing by 1m
every year until the last year, when the cost incurred will be at a rate of 1m.
The cost to consumers who will have to spend more time filling in paperwork and
talking to their financial advisers is expected to be incurred at a rate of 10m in
the first year, increasing by 3% per annum over the twenty year period.

CT1 S2006

The benefits are estimated as follows:


The benefit to consumers who are less likely to buy inappropriate policies is
estimated to be received at a rate of 30m in the first year, 33m in the second
year, 36m in the third year and so on, rising by 3m per year until the end of
twenty years.
The benefit to companies who will spend less time dealing with complaints from
customers is estimated to be received at a rate of 12m per annum for twenty
years.
Calculate the net present value of the benefit or cost of the regulations in their first
twenty years at a rate of interest of 4% per annum effective. Assume that all costs and
benefits occur continuously throughout the year.
[12]

11

(i)

Describe the characteristics of an index-linked government bond.

[3]

(ii)

On 1 July 2002, the government of a country issued an index-linked bond of


term seven years. Coupons are paid half-yearly in arrears on 1 January and 1
July each year. The annual nominal coupon is 2%. Interest and capital
payments are indexed by reference to the value of an inflation index with a
time lag of eight months.
You are given the following values of the inflation index.
Date

Inflation index

November 2001
May 2002
November 2002
May 2003

110.0
112.3
113.2
113.8

The inflation index is assumed to increase continuously at the rate of 2% per


annum effective from its value in May 2003.
An investor, paying tax at the rate of 20% on coupons only, purchased the
stock on 1 July 2003, just after a coupon payment had been made.
Calculate the price to this investor such that a real net yield of 3% per annum
convertible half yearly is obtained and assuming that the investor holds the
bond to maturity.
[10]
[Total 13]

CT1 S2006

PLEASE TURN OVER

12

A pension fund has the following liabilities: annuity payments of 160,000 per annum
to be paid annually in arrears for the next 15 years and a lump sum of 200,000 to be
paid in ten years. It wishes to invest in two fixed-interest securities in order to
immunise its liabilities. Security A has a coupon rate of 8% per annum and a term to
redemption of eight years. Security B has a coupon rate of 3% per annum and a term
to redemption of 25 years. Both securities are redeemable at par and pay coupons
annually in arrear.
(i)

Calculate the present value of the liabilities at a rate of interest of 7% per


annum effective.

[2]

(ii)

Calculate the discounted mean term of the liabilities at a rate of interest of 7%


per annum effective.
[4]

(iii)

Calculate the nominal amount of each security that should be purchased so


that both the present value and discounted mean terms of assets and liabilities
are equal.
[7]

(iv)

Without further calculation, comment on whether, if the conditions in (iii) are


fulfilled, the pension fund is likely to be immunised against small, uniform
changes in the rate of interest.
[2]
[Total 15]

END OF PAPER

CT1 S2006

Faculty of Actuaries

Institute of Actuaries

EXAMINATION
September 2006

Subject CT1 Financial Mathematics


Core Technical
EXAMINERS REPORT
Introduction
The attached subject report has been written by the Principal Examiner with the aim of
helping candidates. The questions and comments are based around Core Reading as the
interpretation of the syllabus to which the examiners are working. They have however given
credit for any alternative approach or interpretation which they consider to be reasonable.
M A Stocker
Chairman of the Board of Examiners
November 2006

Faculty of Actuaries
Institute of Actuaries

Subject CT1 (Financial Mathematics Core Technical) September 2006 ExaminersReport

Comments
As in many recent diets, the questions requiring verbal reasoning (e.g. Question 4(i)) tended
not to be well answered with candidates producing vague statements which did not
demonstrate that they understood the relevant points
Please note that differing answers may be obtained from those shown in these solutions
depending on whether figures obtained from tables or from calculators are used in the
calculations but candidates are not penalised for this.
However, candidates may be penalised where excessive rounding has been used or where
insufficient working is shown.
Comments on solutions presented to individual questions for this September 2006 paper are
given below.
Question 1
Generally well answered. To gain full marks candidates were required to specify the
difference between futures and options rather than just defining each contract separately.
Question 2
Well answered. This was a question where some candidates were penalised if answers had
been rounded excessively.
Question 3
Generally well answered. Another possible solution is to use 1 + j =

1 + 0.6i 1 + 0.6 0.11


=
1+ f
1.07143

which leads to the same answer.


Question 4
For full marks in part (i), an answer should have included a description of the risk-free
concept (rather than just saying arbitrage profits are impossible). Many students had
difficulty with part (ii).
Question 5
Full marks were given if either 365 or 365.25 days were used in the calculation. Most
students scored well on this question.
Question 6
This question was well answered. For full marks, candidates were required to show detailed
steps in deriving the result required including a definition of the initial terms used and a
correct explanation of the relevance of the independence assumption.

Page 2

Subject CT1 (Financial Mathematics Core Technical) September 2006 Examiners Report

Question 7
This question was poorly answered to the surprise of the examiners. Many candidates
struggled to deal with the linked internal rate of return.
Question 8
Well answered.
Question 9
This question appeared to reward candidates who had a good understanding of the topic.
Whilst the best candidates usually scored close to full marks on this question, weaker or lessprepared candidates often scored very badly.
Whilst the question did state that payments were made monthly, the examiners recognised
that there was some potential for misinterpretation as to the frequency of the loan repayments
in part (e) and took this into account. Thus students who used the formula Xa30 = 100, 000
with

(12 )

= 6% & i =6.168% to get an answer of 7,396 in this part were awarded full marks.

Question 10
Generally well answered.
Question 11
This was the worst answered question on the paper by some margin with very few candidates
scoring close to full marks. This may be because this type of question has not appeared in
recent diets. Candidates needed to show that they could derive logically the amounts that will
be paid, the real values of those amounts and their present values in real terms. Appropriate
formulae then needed to be developed.
Question 12
Many candidates answered this question well although a minority scored very badly (possibly
due to time pressure).

Page 3

Subject CT1 (Financial Mathematics Core Technical) September 2006 ExaminersReport

(i)

A future is a contract binding buyer and seller to deliver or take delivery of an


asset at a given price at a given time in the future. An option is a contract that
gives the buyer the option to deliver or take delivery of the asset at the given
price. The seller of the option must deliver/take delivery if the buyer of the
option wishes to exercise the option.

(ii)

Convertibles have option-like characteristics because they give the holder the
option to purchase equity in a company on pre-arranged terms.

The accumulated value is


4 s3 + 2 s2 + 2 s1

i
4s + 2s2 + 2 s1
3

0.04
( 4 3.1216 + 2 2.0400 + 2 )
0.039221

= 18.9352

Page 4

(a)

The money rate of return is i where (1+i) = 11.1/10


i = 0.11 or 11%

(b)

The rate of inflation is f where (1+f) = 120/112


f = 0.07143 or 7.143%

(c)

The net real rate of return per annum is j


0.6i f 0.6 0.11 0.07143
=
= 0.005068 or 0.5068%
where j =
1+ f
1.07143

(i)

The no arbitrage assumption means that it is assumed that an investor is unable


to make a risk-free trading profit.

(ii)

In all states of the world, security B pays 80% of A. Therefore its price must
be 80% of As price, or the investor could obtain a better payoff by only
purchasing one security and make risk-free profits by selling one security short
and buying the other. The price of B must therefore be 16p.

Subject CT1 (Financial Mathematics Core Technical) September 2006 Examiners Report

(i)

(a)

Let the answer be t days


3,600(1 + 0.06 t/365) = 4,000
t = 675.9 days

(b)

Let the answer be t days

3,600 1 + 0.06
4

4t

365

= 4,000

(4t/365) ln(1.015) = ln (4,000/3,600)


t = 645.7 days

(c)

Let the answer be t days

3,600 1 + 0.06
12

12 t

365

= 4,000

(12t/365) ln(1.005) = ln (4,000/3,600)


t = 642.5 days

(ii)

(i)(a) takes longest because, under conditions of simple interest, interest does
not earn interest.

(i)

Let it be the (random) rate of interest in year t . Let S10 be the accumulation of
the unit investment after 10 years:

E ( S10 ) = E (1 + i1 )(1 + i2 ) (1 + i10 )


E ( S10 ) = E [1 + i1 ] E [1 + i2 ] E [1 + i10 ] as {it } are independent
E [it ] = j
E ( S10 ) = (1 + j )

10

(ii)

= 1.0710 = 1.96715

( )

2
2
E S10
= E (1 + i1 )(1 + i2 ) (1 + i10 )

= E (1 + i1 ) E (1 + i2 ) E (1 + i10 ) (using independence)


2

) (

) (

2
= E 1 + 2i1 + i12 E 1 + 2i2 + i22 E 1 + 2i10 + i10

)
Page 5

Subject CT1 (Financial Mathematics Core Technical) September 2006 ExaminersReport

10

= E 1 + 2it + it2

= 1 + 2 j + s2 + j 2

10

as E ii2 = V [it ] + E [it ] = s 2 + j 2



2

(
) (1 + j )
= (1 + 2 0.07 + 0.016 + 0.07 )

Var [ S n ] = 1 + 2 j + s 2 + j 2

10

20

10

(1.07 )

20

= 0.5761

(iii)

If 1,000 units had been invested, the expected accumulation would have been
1,000 times bigger. The variance would have been 1,000,000 times bigger.

(i)

(a)

(b)

(1 + i )2 =

450 500
800
i = 1.015%
600 450 + 40 500 + 100

First sub-interval is first year. Money weighted rate of return is i1


450
where (1 + i1 ) =
i1 = 25%
600
Second sub-interval is second year. Money weighted rate of return is i2
where 490 (1 + i2 ) + 100 (1 + i2 )
Then (1 + i2 )

= 800

100 1002 4 490 (800) 100 1256.1847


=
2 490
980

= 1.17978 (taking positive root)

(1 + i2 ) = 1.39188 i2 = 39.188%
Linked internal rate of return is i
where (1 + i ) = 0.75 1.39188 i = 2.1719%
2

(ii)

Page 6

The linked IRR is higher because it relies on two money weighted rates of
return. With the calculation of the second money weighted rate of return, there
is more money in the fund when the fund is performing well (in the second
half of the year).

Subject CT1 (Financial Mathematics Core Technical) September 2006 Examiners Report

(i)

5
150 = 100 exp at + bt 2 dt = 100 exp 12 at 2 + 13 bt 3 = 100 exp [12.5a + 41.667b ]

0
0

10

10
230 = 100 exp at + bt 2 dt = 100 exp 12 at 2 + 13 bt 3 = 100 exp [50a + 333.333b ]

0
0

ln(1.5) = 12.5a + 41.667b


ln(2.3) = 50a + 333.333b
The second expression less four times the first expression gives:

ln(2.3) 4 ln(1.5) = 166.667b b = 0.0047337


a=

ln(2.3) 333.333 0.0047337


= 0.0482162
50

(ii)

100e10 = 230 10 = ln 2.3 = 0.08329

(iii)

Present Value =

10

20e

0.05t 0.08329t

dt

0
10

20e

0.03329t

dt

0
10

e 0.03329t
= 20

0.03329 0

= 20 8.5058 = 170.116

(a)

Premiums were expected to accumulate to


(12 )
30

1, 060s

(b)

at 7% = 1, 060

i
s = 1, 060 1.037525 94.4608 = 103,885.77
12 30
(
d )

Premiums would have accumulated to


(12 )
30

1, 060s

at 4% = 1, 060

i
s = 1, 060 1.021537 56.0849 = 60, 730.37
12 ) 30
(
d

The shortfall is 100,000 60,730.37 = 39,269.63

Page 7

Subject CT1 (Financial Mathematics Core Technical) September 2006 ExaminersReport

(c)

Accumulation will be
(12 )
20 4%

1, 060 s

= 1, 060

)
(1.04 )10 + 5, 000s10(124%

i
i
10
s20 (1.04 ) + 5, 000
s
12
12 10
d( )
d( )

= 1, 060 1.021537 29.77811.48024 + 5, 000 1.021537 12.0061


= 109, 053.12
Therefore the excess is 9,053.12
(d)

The investor has earned a return of 4 % by investing extra premiums in the


investment policy. The investor could have obtained a lower present value of
total payments on the loan by paying off part of the loan instead. This is
because the interest being paid on the loan was greater than the interest he was
earning on his premiums.

(e)

If he had repaid the loan by a level annuity, the annual instalment would have
been X where
X
(12 )
a360 = 100, 000 at 0.5% (or Xa
= 100, 000 with
30
12
X=

10

(12 )

= 6% & i = 6.168%)

12 100, 000 1, 200, 000


=
= 7,194.61
a360
166.7916

Present value of companies and consumers costs is (in million)

i
( 50 + 10 ) v + 1.03v 2 + 1.032 v3 + + 1.0319 v 20

i
2
19
= 60v 1 + 1.03v + (1.03v ) + + (1.03v )

1 (1.03v )
i
= 60v

1 1.03v

20

) = 1.019869 60 0.96154 1 1.80611 0.45639

= 1.019869 60 0.96154 18.27680 = 1075.383

Page 8

1 1.03 0.96154

Subject CT1 (Financial Mathematics Core Technical) September 2006 Examiners Report

Present value of costs to financial advisors (in million)

i
60v + 19v 2 + 18v3 + + v 20

= 40

iv i
+ 20v + 19v 2 + 18v3 + + v 20

= 40

iv i
i
+ 21a20 Ia20 = 40v + 21a20 Ia20

= 1.019869 ( 40 0.96154 + 21 13.5903 125.1550 )


= 1.019869 198.7029 = 202.651

Total PV of all costs = 1278.034 million


Present value of benefits (in million)

i
i
30v + 33v 2 + 36v3 + + 87v 20 + 12a20

i
27 a20 + 3v + 6v 2 + 9v3 + + 60v 20 + 12a20

i
3 ( Ia )20 + 39a20

= 1.019869 ( 3 125.1550 + 39 13.5903)


= 1.019869 905.4867
= 923.478
Net present value of costs = PV(costs) PV(benefits)
= 1278.034 923.478 = 354.556 million

Page 9

Subject CT1 (Financial Mathematics Core Technical) September 2006 ExaminersReport

11

(i)

(ii)

Payments guaranteed by government.


Can be various different indexation provisions but, in general, protection is
given against a fall in the purchasing power of money.
Fairly liquid (i.e. large issue size and ability to deal in large quantities)
compared with corporate issues, but not compared with conventional
issues.
Normally coupon and capital payments both indexed to increases in a
given price index with a lag.
Low volatility of return and low expected real return.
More or less guaranteed real return if held to maturity (can vary due to
indexation lag).
Nominal return is not guaranteed.

The first coupon the investor will receive will be on 31st December 2003. The
net coupon per 100 nominal will be:
0.8 1 (Index May 2003/Index November 2001) = 0.8 1

In real present value terms, this is 0.8

113.8
110

113.8
v
110 (1 + r )0.5

where r = 2.5% per annum and v is calculated at 1.5% (per half year)

The second coupon on 30th June 2004 per 100 nominal will be
113.8
0.8 1
(1 + r )0.5
110
2
0.5 113.8 v
In real present value terms, this is 0.8 (1 + r )
110 (1 + r )
The third coupon on 31st December 2004 per 100 nominal will be
113.8
0.8 1
(1 + r )
110
113.8
v3
In real present value terms, this is 0.8 (1 + r )
110 (1 + r )1.5

Continuing in this way, the last coupon payment on 30 June 2009 per 100
113.8
nominal will be 0.8 1
(1 + r )5.5
110
5.5 113.8

In real present value terms, this is 0.8 (1 + r )

Page 10

v12

110 (1 + r )6

Subject CT1 (Financial Mathematics Core Technical) September 2006 Examiners Report

By similar reasoning, the real present value of the redemption payment is


5.5 113.8

100 (1 + r )

v12

110 (1 + r )6

The present value of the succession of coupon payments and the capital
payment can be written as:
P=

( (

113.8
0.8 v + v 2 + + v12 + 100v12
0.5 110
(1 + r )

1
113.8
12
0.8a12 1.5% + 100v1.5%
1.0124224 110

= 1.02185 ( 0.8 10.9075 + 100 0.83639 )


= 94.3833

12

(i)

Present value of liabilities is 160, 000a15 + 200, 000v10 at 7%


= 160, 000 9.1079 + 200, 000 0.50835
= 1,558,934

(ii)

Discounted mean term (DMT) of liabilities is

(1160, 000 v + 2 160, 000 v


=

+ + 15 160, 000 v15 + 200, 000 10 v10

160, 000a15 + 200, 000v

10

160, 000 Ia15 + 200, 000 10 v10


160, 000a15 + 200, 000v10

160, 000 61.5540 + 200, 000 10 0.50835


1,558,934

10,865,340
= 6.9697 years ( mark deducted for no units)
1,558,934

Page 11

Subject CT1 (Financial Mathematics Core Technical) September 2006 ExaminersReport

(iii)

Let the nominal amounts in each security equal A and B respectively.


If the present values of assets and liabilities are to be equal then:

) (

A 0.08a8 + v8 + B 0.03a25 + v 25 = 1,558,934 (1)

If the DMTs of the assets and liabilities are equal, then:

) (

A 0.08 ( Ia )8 + 8v8 + B 0.03 ( Ia )25 + 25v 25


1,558,934

) (

) = 6.9697
)

or A 0.08 ( Ia )8 + 8v8 + B 0.03 ( Ia )25 + 25v 25 = 10,865,340 (2)


From (1)
A ( 0.08 5.9713 + 0.58201) + B ( 0.03 11.6536 + 0.18425 ) = 1,558,934
1.059714 A + 0.533858 B = 1,558,934
From (2)
A ( 0.08 24.7602 + 8 0.58201) + B ( 0.03 112.3301 + 25 0.18425 ) = 10,865,340
6.636896 A + 7.976153B = 10,865,340
Therefore

1,558,934 0.533858 B
6.636896
+ 7.976153B = 10,865,340
1.059714

6.636896 0.533858
6.636896 1,558,934

B 7.976153
= 10,865,340
1.059714
1.059714

B=

1,101,872.85
= 237,850
4.632647

1,558,934 0.533858 B
A=
= 1,351, 266
1.059714

Page 12

Subject CT1 (Financial Mathematics Core Technical) September 2006 Examiners Report

(iv)

It appears that the asset payments are more spread out than the liability
payments. The third condition for immunisation is that that convexity of the
assets is greater than that of the liabilities, or that the asset times are more
spread around the discounted mean term than the liability times. From
observation is appears likely that this condition is met.

END OF EXAMINERS REPORT

Page 13

Faculty of Actuaries

Institute of Actuaries

EXAMINATION
12 April 2007 (am)

Subject CT1 Financial Mathematics


Core Technical
Time allowed: Three hours
INSTRUCTIONS TO THE CANDIDATE
1.

Enter all the candidate and examination details as requested on the front of your answer
booklet.

2.

You must not start writing your answers in the booklet until instructed to do so by the
supervisor.

3.

Mark allocations are shown in brackets.

4.

Attempt all 11 questions, beginning your answer to each question on a separate sheet.

5.

Candidates should show calculations where this is appropriate.

Graph paper is not required for this paper.

AT THE END OF THE EXAMINATION


Hand in BOTH your answer booklet, with any additional sheets firmly attached, and this
question paper.
In addition to this paper you should have available the 2002 edition of the
Formulae and Tables and your own electronic calculator.

CT1 A2007

Faculty of Actuaries
Institute of Actuaries

An investor pays 400 every half-year in advance into a 25-year savings plan.
Calculate the accumulated fund at the end of the term if the interest rate is 6% per
annum convertible monthly for the first 15 years and 6% per annum convertible halfyearly for the final 10 years.
[5]

The force of interest ( t ) is a function of time and at any time, measured in years, is
given by the formula:
( t ) = 0.04 + 0.01t

0t4

( t ) = 0.12 0.01t

4<t 8

( t ) = 0.06

8<t

Calculate the present value at time t = 0 of a payment stream, paid continuously from
time t = 9 to t = 12, under which the rate of payment at time t is 50e0.01t .
[6]

An ordinary share pays annual dividends. The next dividend is due in exactly eight
months time. This dividend is expected to be 1.10 per share. Dividends are expected
to grow at a rate of 5% per annum compound from this level and are expected to
continue in perpetuity. Inflation is expected to be 3% per annum. The price of the
share is 21.50.
Calculate the expected effective annual real rate of return for an investor who
purchases the share.

[7]

An investor entered into a long forward contract for a security five years ago and the
contract is due to mature in seven years time. The price of the security was 95 five
years ago and is now 145. The risk-free rate of interest can be assumed to be 3% per
annum throughout the 12-year period.
Assuming no arbitrage, calculate the value of the contract now if:
(i)

The security will pay dividends of 5 in two years time and 6 in four years
time.
[3]

(ii)

The security has paid and will continue to pay annually in arrear a dividend of
2% per annum of the market price of the security at the time of payment. [3]
[Total 6]

CT1 A20072

In a particular bond market, n-year spot rates per annum can be approximated by the
function 0.08 0.04e 0.1n .
Calculate:

(i)

The price per unit nominal of a zero coupon bond with term nine years.

[2]

(ii)

The four-year forward rate at time 7 years.

[3]

(iii)

The three-year par yield.

[3]
[Total 8]

A fund had a value of 21,000 on 1 July 2003. A net cash flow of 5,000 was
received on 1 July 2004 and a further net cash flow of 8,000 was received on 1 July
2005. Immediately before receipt of the first net cash flow, the fund had a value of
24,000, and immediately before receipt of the second net cash flow the fund had a
value of 32,000. The value of the fund on 1 July 2006 was 38,000.
(i)

Calculate the annual effective money weighted rate of return earned on the
fund over the period 1 July 2003 to 1 July 2006.
[3]

(ii)

Calculate the annual effective time weighted rate of return earned on the fund
over the period 1 July 2003 to 1 July 2006.
[3]

(iii)

Explain why the values in (i) and (ii) differ.

[2]
[Total 8]

An insurance company has liabilities of 87,500 due in 8 years time and 157,500
due in 19 years time. Its assets consist of two zero coupon bonds, one paying
66,850 in four years time and the other paying X in n years time. The current
interest rate is 7% per annum effective.
(i)

Calculate the discounted mean term and convexity of the liabilities.

(ii)

Determine whether values of X and n can be found which ensure that the
company is immunised against small changes in the interest rate.
[5]
[Total 10]

CT1 A20073

[5]

PLEASE TURN OVER

A company has borrowed 800,000 from a bank. The loan is to be repaid by level
instalments, payable annually in arrear for 10 years from the date the loan is made.
The annual repayments are calculated at an effective rate of interest of 8% per annum.
(i)

Calculate the amount of the level annual payment and the total amount of
interest which will be paid over the 10 year term.
[3]

(ii)

At the beginning of the eighth year, immediately after the seventh payment
has been made, the company asks for the term of the loan to be extended by
two years. The bank agrees to do this on condition that the rate of interest is
increased to an effective rate of 12% per annum for the remainder of the term
and that payments are made quarterly in arrear.
(a)

Calculate the amount of the new quarterly payment.

(b)

Calculate the capital and interest components of the first quarterly


instalment of the revised loan repayments.
[6]
[Total 9]

A property developer is constructing a block of offices. It is anticipated that the


offices will take six months to build. The developer incurs costs of 40 million at the
beginning of the project followed by 3 million at the end of each month for the
following six months during the building period. It is expected that rental income
from the offices will be 1 million per month, which will be received at the start of
each month beginning with the seventh month. Maintenance and management costs
paid by the developer are expected to be 2 million per annum payable monthly in
arrear with the first payment at the end of the seventh month. The block of offices is
expected to be sold 25 years after the start of the project for 60 million.
(i)

Calculate the discounted payback period using an effective rate of interest of


10% per annum.
[7]

(ii)

Without doing any further calculations, explain whether your answer to (i)
would change if the effective rate of interest were less than 10% per annum.
[3]
[Total 10]

CT1 A20074

10

11

A loan is issued bearing interest at a rate of 9% per annum and payable half-yearly in
arrear. The loan is to be redeemed at 110 per 100 nominal in 13 years time.
(i)

The loan is issued at a price such that an investor, subject to income tax at
25%, and capital gains tax at 30%, would obtain a net redemption yield of 6%
per annum effective. Calculate the issue price per 100 nominal of the stock.
[5]

(ii)

Two years after the date of issue, immediately after a coupon payment has
been made, the investor decides to sell the stock and finds a potential buyer,
who is subject to income tax at 10% and capital gains tax at 35%. The
potential buyer is prepared to buy the stock provided she will obtain a net
redemption yield of at least 8% per annum effective.
(a)

Calculate the maximum price (per 100 nominal) which the original
investor can expect to obtain from the potential buyer.

(b)

Calculate the net effective annual redemption yield (to the nearest 1%
per annum effective) that will be obtained by the original investor if
the loan is sold to the buyer at the price determined in (ii) (a).
[10]
[Total 15]

80,000 is invested in a bank account which pays interest at the end of each year.
Interest is always reinvested in the account. The rate of interest is determined at the
beginning of each year and remains unchanged until the beginning of the next year.
The rate of interest applicable in any one year is independent of the rate applicable in
any other year.
During the first year, the annual effective rate of interest will be one of 4%, 6% or 8%
with equal probability.
During the second year, the annual effective rate of interest will be either 7% with
probability 0.75 or 5% with probability 0.25.
During the third year, the annual effective rate of interest will be either 6% with
probability 0.7 or 4% with probability 0.3.
(i)

Derive the expected accumulated amount in the bank account at the end of
three years.
[5]

(ii)

Derive the variance of the accumulated amount in the bank account at the end
of three years.
[8]

(iii)

Calculate the probability that the accumulated amount in the bank account is
more than 97,000 at the end of three years.
[3]
[Total 16]

END OF PAPER

CT1 A20075

Faculty of Actuaries

Institute of Actuaries

EXAMINATION
April 2007

Subject CT1 Financial Mathematics


Core Technical
EXAMINERS REPORT
Introduction
The attached subject report has been written by the Principal Examiner with the aim of
helping candidates. The questions and comments are based around Core Reading as the
interpretation of the syllabus to which the examiners are working. They have however given
credit for any alternative approach or interpretation which they consider to be reasonable.

M A Stocker
Chairman of the Board of Examiners
June 2007

Faculty of Actuaries
Institute of Actuaries

Subject CT1 (Financial Mathematics Core Technical) April 2007 Examiners Report

Comments
Please note that different answers may be obtained to those shown in these solutions
depending on whether figures obtained from tables or from calculators are used in the
calculations but candidates are not penalised for this.
However, candidates may be penalised where excessive rounding has been used or where
insufficient working is shown.
Q1.
Whilst most candidates made a good attempt at this question on basic compound interest
accumulation, comparatively few students completed the question without error.
Q2.
Well answered.
Q3.
Most students answered this question well although candidates were expected to note that the
sum of the geometric progression would only converge if the rate of return was below the
dividend growth rate. Depending on the interpolation used, the final answer can justifiably
vary from that given.
Q4.
This proved to be the most difficult question on the paper. Other related methods to
determine the answers were available e.g. calculating the forward price of each contract and
working out the present value of the difference in these prices.
Q5.
Well answered.
Q6.
The calculations in parts (i) and (ii) were generally well done. Again, depending on the
interpolation used, the final answer can justifiably vary from that given although the
examiners penalised the use of too wide a range of interpolation.
The explanation in part (iii) was very poorly handled. In such cases, the examiners are not
simply looking for a statement lifted directly from the Core Reading. Instead, candidates are
expected to apply the relevant theory to the actual situation described in the question.

Page 2

Subject CT1 (Financial Mathematics Core Technical) April 2007 Examiners Report

Q7.
Generally well answered.
Q8.
This was the best answered question on the paper.
Q9.
Many candidates struggled with this question, firstly in determining when the various
costs/payments would be made and then in manipulating the resulting equation(s). A common
error was not to recognise that the DPP should be expressed as a whole number of months
since payments at the relevant time were being made at monthly intervals. In part (ii) little
credit was given for a correct conclusion without any accompanying explanation.
Q10.
This question seemed to provide a significant differentiation between candidates with many
scoring well and a sizeable minority scoring very badly. This seemed surprising given that
this topic is regularly examined. A common omission on part (ii)(b) was not to state whether
a capital gain had been made.
Q11.
The workings for parts (i) and (ii) were often too brief (the questions said Derive). Note
that the final answer in part (ii) can justifiably vary significantly according to the rounding
used in intermediate calculations. Part (iii) was poorly done with many candidates assuming
a lognormal distribution for this discrete example.

Page 3

Subject CT1 (Financial Mathematics Core Technical) April 2007 Examiners Report

Fund after 25 years =

400S30i (1.03)
*%

where 1 + i* = (1.005 )

20

3%
+ 400S20

i = 3.03775% per -year

s30

(1.0303775 )30 1

@ 3.03775% = 1.0303775
0.0303775

= 49.3215
(1.03)20 1

= 27.6765
S20 @ 3% = 1.03
0.03
Hence fund =
400 49.3215 (1.03)

20

+ 400 27.6765

= 35632.06 + 11070.60
= 46,702.66

(i)

12

PV = 50 e

0.01t

. e

( t )dt
t
0

dt

where
t

( t ) dt =

( 0.04 + 0.01t ) dt + 4 ( 0.12 0.01t ) dt + 8 0.06dt


0
4

t
= 0.04t + 0.005t 2 + 0.12t 0.005t 2 + [ 0.06t ] 8

0
4

= [ 0.24] + [ 0.64 0.40] + [ 0.06t 0.48]

= 0.06t

Page 4

Subject CT1 (Financial Mathematics Core Technical) April 2007 Examiners Report

Hence
12

PV = 50e0.01t . e0.06t dt
9

12

= 50 e

0.05t

dt

12

50 0.05t
=
e

0.05
9

= 548.812 + 637.628
= 88.816

Let i = money rate of return


i = real rate of return
1 + i = (1 + i )(1.03) here

21.50 = (1 + i )

= (1 + i )

= 1.10

12

12

( )

1 1.05
1+i
1.10v
1 1.05

1+i

(1 + i )

19.5455 =

8
12

(1 1.05
1+i )

(1 + i )

1.10v + 1.05 1.10v 2 + (1.05 ) 1.10v3 + ""

8
12

assuming i > 0.05

1
1 1.05
1+i

Try i = 10% RHS = 20.6456


11% RHS = 17.2566
i = 0.10 +

20.6456 19.5455
0.01 = 0.10325
20.6456 17.2566

i comes from 1 + i =

1.10325
i = 7.1% p.a.
1.03

Page 5

Subject CT1 (Financial Mathematics Core Technical) April 2007 Examiners Report

(i)

The current value of the forward price of the old contract is:
95 (1.03) 5 (1.03)
5

6 (1.03)

whereas the current value of the forward price of a new contract is:
145 5 (1.03)

6 (1.03)

Hence, current value of old forward contract is:


145 95 (1.03) = 34.87
5

(ii)

The current value of the forward price of the old contract is:
95 (1.02 )

12

(1.03)5 = 86.8376

whereas the current value of the forward price of a new contract is:
145 (1.02 )

= 126.2312

current value of old forward contract is:


126.23 - 86.84 = 39.39

(i)

Let Yk = spot rate for k year term


Pk = Price per unit nominal for k year term
Y9 = 0.063737
9

1
P9 =
= 0.57344
1 + Y9

Page 6

Subject CT1 (Financial Mathematics Core Technical) April 2007 Examiners Report

(ii)

Y7 = 0.08 0.04 e

0.1( 7 )

Y11 = 0.08 0.04e

(1 + f7,4 )

0.1(11)

= 0.060137

= 0.066685

11
11
1 + Y11 )
1.066685 )
(
(
=
=
(1 + Y7 )7 (1.060137 )7

= 1.35165
4-year forward rate is 7.824% at time 7.
(iii)

Y1 = 0.04381, Y2 = 0.04725, Y3 = 0.05037

( ) ( vY1 + vY2 + vY3 ) + vY3

1 = Yc3

Yc3 = 0.05016 i.e. 5.016% p.a.

(i)

Work in 000s
MWRR is i such that:
21(1 + i ) + 5 (1 + i ) + 8 (1 + i ) = 38
3

Try

i = 5%, LHS = 38.223


i = 4%, LHS = 37.350

By interpolation i = 4.74% p.a.


(ii)

TWRR is i such that:

(1 + i )3 =
(iii)

24 32 38
i = 6.21% p.a.
21 29 40

MWRR is lower than TWRR because of the large cash flow on 1/7/05; the
overall return in the final year is much lower than in the first 2 years, and the
payment at 1/7/05 gives this final year more weight in the MWRR, but does
not affect the TWRR.

Page 7

Subject CT1 (Financial Mathematics Core Technical) April 2007 Examiners Report

Let PVL be PV of liabilities, DMTL be DMT of liabilities, CL be convexity of


liabilities.
(i)

PVL = 87,500v8 + 157,500v19 at 7%


= 94,475.86
DMTL =
=

87,500 8v8 + 157,500 19v19


at 7%
94, 475.86

1, 234,857.56
94, 475.86

= 13.070615 years

CL =

87,500 8 9v10 + 157,500 19 20v 21


at 7%
94, 475.86

17, 657,158.78
94, 475.86

= 186.895985

(ii)

Firstly, PVs should be equal:


66,850v 4 + Xv n = 94, 475.86 at 7%
Xv n = 43, 476.31507

Secondly, DMTs should be equal


66,850 4v 4 + Xnv n = 1, 234,857.56
Xnv n = 1, 030,859.38
n = 23.710827 years
X = 43, 476.31507 1.07 n
= 216,255.12

Page 8

Subject CT1 (Financial Mathematics Core Technical) April 2007 Examiners Report

Lastly, verify 3rd condition

C A = 66,850 4 5v 6 + 216, 255.12n ( n + 1) v(

n+ 2)

) / 94, 475.86

= 23,140,343.20/94,475.86
= 244.93393
> CL
Hence, immunisation is achieved.

(i)

8%
800, 000 = P a10
= P 6.7101

P = 119, 223.26
Total amount of interest = 10 119,223.26 800,000
= 392,232.60
(ii)

(a)

Capital o/s at start of 8th year


= 119,223.26 a38% = 119, 223.26 2.5771 = 307, 250.26
Let new payment be P per annum, then
Pa (4) = P 1.043938 3.6048 = 307, 250.26
5
12%

P = 81, 646.28
q'ly payment = 20, 411.57
(b)

Capital o/s after 7 years = 307,250.26


1
Interest in 1st q'ly payment = 30, 7250.26 (1.12 ) 4 1 = 8,829.56

capital component = 20,411.57 8,829.56 = 11,582.01

Page 9

Subject CT1 (Financial Mathematics Core Technical) April 2007 Examiners Report

(i)

The discounted payback period is the first point at which the present value of
the income exceeds the present value of the outgoings. The present value of
all payments and income up to time t is given by (working in m)
1
1
(12 )
(12 )
(12 )
PV = 40 36a 1 2v 2 a 1 + 12v 2 a 1
2

t 2 + 112

1
1
1
(12 )
(12 )
(12 )
= 40 36a 1 2v 2 a 1 + 12v 2 12
+a 1
t 2
t 2
2

1
1
t 0.5
(12 )
= 40 36a 1 + v 2 + 10v 2 1v(12)
2

(12 )

a1

1 v 2
1 0.9534626
=
at 10% =
= 0.48634
12
0.0956897
i( )

0.56758 = 1 - vt-0.5
vt-0.5
= 0.43242
log( 0.43242 )
t
= log 0.90909 + 0.5
(
)
t
9.296
Hence, the discounted pay back period is 9 years and 4 months.
(ii)

10

(i)

If the effective rate of interest were less than 10% p.a. then the present values
of the income and outgo would both increase. However, the bigger impact
would be on the present value of the income since the bulk of the outgo occurs
in the early years when discounting has less effect. Hence, the DPP would
decrease.
2
i ( ) = 0.059126

g (1 t1 ) =

0.09
0.75=0.06136
1.10

2
i ( ) < (1 t1 ) g

No capital gain

Page 10

Subject CT1 (Financial Mathematics Core Technical) April 2007 Examiners Report

Price of 100 nominal stock


( 2)
+110v13
13

= 0.75 9 a

at 6%

= 0.75 9 1.014782 8.8527 + 110 0.46884


= 60.639 + 51.572
= 112.21
(ii)

(a)

2
i ( ) = 0.078461

g (1 t1 ) =

0.09
0.90= 0.073636
1.10

2
i ( ) > (1 t1 ) g

Capital gain
Price, P = 0.90 9 a

( 2)

11

P=

=
(b)

+ (110 (110 P ) 0.35 ) v11 at 8%

0.90 9 1.019615 7.1390 + 0.65 110 0.42888


1 0.35 0.42888

89.62508
= 105.455
0.849892

No capital gain made


( 2)
2

112.21 = 0.75 9 a

+ 105.455v 2

Try i = 3%, RHS = 112.41


i = 4%, RHS = 110.36
yield = 3% p.a. to nearest 1%

Page 11

Subject CT1 (Financial Mathematics Core Technical) April 2007 Examiners Report

11

(i)

Let S3 = Accumulated fund after 3 years of investment of 1 at time 0


it = Interest rate for year t
Then, fund after 3 years
= 80,000 S3 = 80000 (1 + i1 )(1 + i2 ) (1 + i3 )
E ( i1 ) = 13 ( 0.04+0.06+0.08 ) =0.06
E ( i2 ) = 0.75 0.07+0.25 0.05=0.065
E ( i3 ) = 0.7 0.06 + 0.3 0.04 = 0.054

Then:
E [80000 S3 ] = 80, 000 E [ S3 ]

= E 80,000 (1 + i1 )(1 + i2 ) (1 + i3 )
= 80,000 E (1 + i1 ) . E (1 + i2 ) . E (1 + i3 )

since it ' s are independent


= 80,000 1.06 1.065 1.054 = 95,188.85
(ii)

Var [80000S3 ] = 80,0002 Var [ S3 ]


where Var [ S3 ] = E S32 ( E [ S3 ])

2
2
2
E S32 = E (1 + i1 ) (1 + i2 ) (1 + i3 )

2
2
2
= E (1 + i1 ) . E (1 + i2 ) . E (1 + i3 )

using independence

)(

)(

= 1 + 2 E [i1 ] + E i12 . 1 + 2 E [i2 ] + E i22 1 + 2 E [i3 ] + E i32




Page 12

Subject CT1 (Financial Mathematics Core Technical) April 2007 Examiners Report

Now,

( ) (

E i12 = 13 0.042 + 0.062 + 0.082 = 0.0038667

( )

E i22 = 0.75 0.07 2 + 0.25 0.052 = 0.0043

( )

E i32 = 0.7 0.062 + 0.3 0.042 = 0.0030

Hence, E S32

= (1 + 2 0.06 + 0.0038667 ) (1 + 2 0.065 + 0.0043) (1 + 2 0.054 + 0.003)

=1.41631
Hence Var [80, 000 S3 ]
= 80, 0002 Var [ S3 ]

= 80, 0002 1.41631 (1.18986 )

= 3,476,355
(iii)

Note: 80,000 1.08 1.07 1.06 = 97,995 > 97,000


But, if in any year, the highest interest rate for the year is not achieved then the
fund after 3 years falls below 97,000.
Hence, answer is probability that highest interest rate is achieved in each year
1
= 0.75 0.7 = 0.175
3

END OF EXAMINERS REPORT

Page 13

Faculty of Actuaries

Institute of Actuaries

EXAMINATION
25 September 2007 (am)

Subject CT1 Financial Mathematics


Core Technical
Time allowed: Three hours
INSTRUCTIONS TO THE CANDIDATE
1.

Enter all the candidate and examination details as requested on the front of your answer
booklet.

2.

You must not start writing your answers in the booklet until instructed to do so by the
supervisor.

3.

Mark allocations are shown in brackets.

4.

Attempt all 11 questions, beginning your answer to each question on a separate sheet.

5.

Candidates should show calculations where this is appropriate.

Graph paper is not required for this paper.

AT THE END OF THE EXAMINATION


Hand in BOTH your answer booklet, with any additional sheets firmly attached, and this
question paper.
In addition to this paper you should have available the 2002 edition of the
Formulae and Tables and your own electronic calculator.

CT1 S2007

Faculty of Actuaries
Institute of Actuaries

A 90-day government bill is purchased for 96 at the time of issue and is sold after 45
days to another investor for 97.90. The second investor holds the bill until maturity
and receives 100.
Determine which investor receives the higher rate of return.

[2]

An investor purchases a share for 769p at the beginning of the year. Halfway through
the year he receives a dividend, net of tax, of 4p and immediately sells the share for
800p. Capital gains tax of 30% is paid on the difference between the sale and the
purchase price.
Calculate the net annual effective rate of return the investor obtains on the investment.
[4]

An insurance company offers a customer two payment options in respect of an


invoice for 456. The first option involves 24 payments of 20 paid at the beginning
of each month starting immediately. The second option involves 24 payments of
20.50 paid at the end of each month starting immediately. The customer is willing to
accept a monthly payment schedule if the annual effective interest rate per annum he
pays is less than 5%.
Determine which, if any, of the payment options the customer will accept.

[4]

State the characteristics of an equity investment.

[4]

A one-year forward contract is issued on 1 April 2007 on a share with a price of 900p
at that date. Dividends of 50p per share are expected on 30 September 2007 and 31
March 2008. The 6-month and 12-month spot, risk-free rates of interest are 5% and
6% per annum effective respectively on 1 April 2007.
Calculate the forward price at issue, stating any assumptions.

[4]

The annual effective forward rate applicable over the period t to t + r is defined as
ft ,r where t and r are measured in years. f 0,1 = 4%, f1,1 = 4.25% f 2,1 = 4.5%,

f 2,2 = 5%. Calculate the following:


f3,1

(i)

[1]

(ii)

All possible zero coupon (spot) yields that the above information allows you
to calculate.
[4]

(iii)

The gross redemption yield of a four-year bond, redeemable at par, with a 3%


coupon payable annually in arrears.
[6]

(iv)

Explain why the gross redemption yield from the four-year bond is lower than
[2]
the one-year forward rate up to time 4, f3,1
[Total 13]

CT1 S20072

The force of interest, (t ) , is a function of time and at any time t (measured in years)
is given by

0.04 + 0.01t
(t ) =
0.05

for 0 t 10
for t > 10

(i)

Derive, and simplify as far as possible, expressions for v (t ) where v(t ) is the
present value of a unit sum of money due at time t.
[5]

(ii)

(a)

Calculate the present value of 1,000 due at the end of 15 years.

(b)

(iii)

Calculate the annual effective rate of discount implied by the


transaction in (a).
[4]
0.01t
A continuous payment stream is received at a rate of 20e
units per
annum between t = 10 and t = 15. Calculate the present value of the payment
stream.
[4]
[Total 13]

A pension fund makes the following investments (m):


1 January 2004
12.5

1 July 2004

1 January 2005

6.6

1 January 2006

7.0

8.0

The rates of return earned on money invested in the fund were as follows:
1 January 2004
to 30 June 2004

1 July 2004 to
31 December 2004

1 January 2005 to
31 December 2005

1 January 2006 to
31 December 2006

5%

6%

6.5%

3%

You may assume that 1 January to 30 June and 1 July to 31 December are precise half
year periods.
(i)

Calculate the linked internal rate of return per annum over the three years from
1 January 2004 to 31 December 2006, using semi-annual sub-intervals.
[3]

(ii)

Calculate the time weighted rate of return per annum over the three years from
1 January 2004 to 31 December 2006.
[3]

(iii)

Calculate the money weighted rate of return per annum over the three years
from 1 January 2004 to 31 December 2006.
[4]

(iv)

Explain the relationship between your answers to (i), (ii) and (iii) above.

[2]

[Total 12]

CT1 S20073

PLEASE TURN OVER

The expected effective annual rate of return from a banks investment portfolio is 6%
and the standard deviation of annual effective returns is 8%. The annual effective
returns are independent and (1+ it ) is lognormally distributed, where it is the return in
year t.
Deriving any necessary formulae:

10

(i)

calculate the expected value of an investment of 2 million after ten years. [6]

(ii)

calculate the probability that the accumulation of the investment will be less
than 80% of the expected value.
[3]
[Total 9]

A government is holding an inquiry into the provision of loans by banks to consumers


at high rates of interest. The loans are typically of short duration and to high risk
consumers. Repayments are collected in person by representatives of the bank making
the loan. Campaigners on behalf of the consumers and campaigners on behalf of the
banks granting the loans are disputing one particular type of loan. The initial loans are
for 2,000. Repayments are made at an annual rate of 2,400 payable monthly in
advance for two years.
The consumers association case

The consumers association asserts that, on this particular type of loan, consumers
who make all their repayments pay interest at an annual effective rate of over 200%.
The banks case

The banks state that, on the same loans, 40% of the consumers default on all their
remaining payments after exactly 12 payments have been made. Furthermore half of
the consumers who have not defaulted after 12 payments default on all their
remaining payments after exactly 18 payments have been made. The banks also argue
that it costs 30% of each monthly repayment to collect the payment. These costs are
still incurred even if the payment is not made by the consumer. Furthermore, with
inflation of 2.5% per annum, the banks therefore assert that the real rate of interest
that the lender obtains on the loan is less than 1.463% per annum effective.
(i)

(ii)

(a)

Calculate the flat rate of interest paid by the consumer on the loan
described above.

(b)

State why the flat rate of interest is not a good measure of the cost of
borrowing to the consumer.
[4]

Determine, for each of the cases above, whether the assertion is correct. [10]
[Total 14]

CT1 S20074

11

A pension fund has liabilities to pay pensions each year for the next 60 years. The
pensions paid will be 100m at the end of the first year, 105m at the end of the
second year, 110.25m at the end of the third year and so on, increasing by 5% each
year. The fund holds government bonds to meet its pension liabilities. The bonds
mature in 20 years time and pay an annual coupon of 4% in arrears.
(i)

Calculate the present value of the pension funds liabilities at a rate of interest
of 3% per annum effective.
[4]

(ii)

Calculate the nominal amount of the bond that the fund needs to hold so that
the present value of the assets is equal to the present value of the liabilities. [3]

(iii)

Calculate the duration of the liabilities.

[6]

(iv)

Calculate the duration of the assets.

[4]

(v)

Using your calculations in (iii) and (iv), estimate by how much more the value
of the liabilities would increase than the value of the assets if there were a
reduction in the rate of interest to 1.5% per annum effective.
[4]
[Total 21]

END OF PAPER

CT1 S20075

Faculty of Actuaries

Institute of Actuaries

EXAMINATION
September 2007

Subject CT1 Financial Mathematics


Core Technical
MARKING SCHEDULE
Introduction
The attached subject report has been written by the Principal Examiner with the aim of
helping candidates. The questions and comments are based around Core Reading as the
interpretation of the syllabus to which the examiners are working. They have however given
credit for any alternative approach or interpretation which they consider to be reasonable.
M A Stocker
Chairman of the Board of Examiners
December 2007

Faculty of Actuaries
Institute of Actuaries

Subject CT1 (Financial Mathematics Core Technical) September 2007 Examiners Report

Comments
Please note that different answers may be obtained from those shown in these solutions
depending on whether figures obtained from tables or from calculators are used in the
calculations but candidates are not penalised for this.
However, candidates may be penalised where excessive rounding has been used or where
insufficient working is shown.
It should be noted that the rubric of the examination paper does ask for candidates to show
their calculations where this is appropriate. Candidates often failed to show sufficient clarity
and detail in their working and lost marks as a result.
Q1.
Well answered.
Q2.
Well answered.
Q3.
Whilst this question was generally answered well, some candidates lost marks by not stating
the conclusions that arose from their calculations i.e. that neither deal was acceptable.
Q4.
This question was very poorly answered which was disappointing given that this was a
bookwork question.
Q5.
Reasonably well answered but some candidates failed to obtain full marks by not stating the
required assumption.
Q6.
Parts (i) and (ii) were well answered but part (iii) was a good differentiator with weaker
candidates failing to recognise the correct method for calculating the gross redemption yield.
As with many previous diets, many candidates in part (iv) had great difficulty in giving a
clear explanation of their calculations.

Page 2

Subject CT1 (Financial Mathematics Core Technical) September 2007 Examiners Report

Q7.
Generally well answered. Some candidates lost marks by not giving an explicit formula for
v(t) when t 10.
Q8.
This question was very poorly answered to the surprise of the examiners who felt that the
question should have been relatively straightforward.
Q9.
Part (i) can be done much more simply than by using the method given in this report but the
calculations given would still need to be done for part (ii).
Q10.
This question was the worst answered on the paper. Part (ii) did successfully differentiate
between candidates with weaker candidates appearing to struggle to apply the theory to a
real-life situation.
Q11.
The first three parts were generally answered well by the candidates who attempted the
question. Many struggled to complete part (iv) although it is possible that this was due to
time pressure. When calculating DMTs, candidates were expected to give the answer in terms
of the correct units.

Page 3

Subject CT1 (Financial Mathematics Core Technical) September 2007 Examiners Report

The first investor receives the higher rate of return if:

97.9 100
>
96 97.9
This inequality does not hold, therefore the second investor receives the higher rate of
return.

Start by working in half years. The half yearly effective return is i such that:
769 = 4v + 800v 0.3(800 769)v
769 = (804 - 240 + 230.7)v
v=

769
= 0.967661 therefore i = 3.3420%
794.7

Annual effective rate is (1.033422 1) = 6.7957%

The annual rate of payment for the first deal is 240.


This deal is acceptable if:
(12 )
2

< 456 at a rate of interest of 5%

(12 )
2

= 240 1.8594 1.026881 = 458.252

240 a
240 a

Therefore first deal is not acceptable


The annual rate of payment on the second deal is 246.
This deal is acceptable if:
(12 )
2

246 a

= 246 1.8594 1.022715 = 467.803

Therefore second deal is also not acceptable

Main characteristics of equity investments:

Page 4

Issued by commercial undertakings and other bodies.


Entitle holders to receive all net profits of the company in the form of
dividends after interest on loans and other fixed interest stocks has been paid.
Higher expected returns than for most other asset classes

Subject CT1 (Financial Mathematics Core Technical) September 2007 Examiners Report

but risk of capital losses


and returns can be variable.
Lowest ranking form of finance.
Low initial running yield
but dividends should increase with inflation.
Marketability varies according to size of company.
Voting rights in proportion to number of shares held.

Assuming no arbitrage:
Present value of dividends is (in):
0.5v1/2 (at 5%) + 0.5v (at 6%) = 0.5(0.97590+0.94340) = 0.95965
Hence forward price is: F = (9-0.95965) 1.06 = 8.5228

(i)

f3,1 is such that 1.045 f3,1 = 1.052. Therefore f3,1 = 5.5024%

(ii)

One-year spot rate is same as one-year forward rate = 4%


Two-year spot rate is i2 such that (1+ i2 )2 = 1.04 1.0425.
Therefore i2 = 4.1249%
Three-year spot rate is i3 such that (1+ i3 )3 = 1.04 1.0425 1.045.
Therefore i3 = 4.2498%
Four year spot rate is such that (1+ i4 )4 = 1.04 1.0425 1.045 1.055024
Therefore i4 = 4.5615%

(iii)

Present value of the payments from the bond is:


P = 3(1.04-1 + 1.041249-2 + 1.042498-3 + 1.045615-4)
+ 100 1.045615-4

Therefore P = 3(0.96154 + 0.92234 + 0.88262 + 0.83659)


+ 100 0.83659 = 94.468
Equation of value to find the gross redemption yield from the bond is such
that:
94.468 = 3 a4 + 100v4
Try i = 4.5%
v4 = 0.83856, a4 = 3.58753, RHS = 94.619

Page 5

Subject CT1 (Financial Mathematics Core Technical) September 2007 Examiners Report

Try i = 5%
v4 = 0.82270, a4 = 3.5460, RHS = 92.908

Interpolation:
Yield = 0.045 + 0.005 (94.619 94.468) /(94.619 92.908)
= 4.544%

(iv)

The yield from the bond is lower than the one-year forward rate up to time 4
because the bond can be seen to be a series of zero coupon bonds (1 year, 2
years etc.) each with lower yields than the forward rate. The gross redemption
yield from the bond is, in effect, an average of spot rates that are themselves a
weighted average of earlier forward rates.

(i)

For t 10
v (t ) = e

t
0

0.04+ 0.01sds

=e

0.04 s + 0.005 s 2

= e 0.04t 0.005t

For t > 10
v ( t ) = v (10 ) e
(ii)

(iii)

t
10

0.05ds

= e0.9 e

[ 0.05 s ]10
t

0.05( t 10 )

0.4+ 0.05t )
=e (

(a)

0.4+ 0.0515 )
Present value = 1000e (
= 1000e 1.15 = 316.637

(b)

1000(1 d )15 = 316.637 d = 7.380%


15 (0.4+ 0.05t )
e
20e0.01t dt
10
15 0.4 0.06t

Present value =

= 20

= 20e

0.4

10

dt

15

Page 6

= e0.9 e

e 0.06t
0.4
( 6.77616 + 9.14686 ) = 31.783

= 20e
0.06 10

Subject CT1 (Financial Mathematics Core Technical) September 2007 Examiners Report

(i)

Linked internal rate of return is found by linking the money weighted rate of
return from the sub-periods.
(LIRR)3 = 1.05 1.06 1.065 1.03
Therefore LIRR = 0.06879 or 6.879%

(ii)

The TWRR requires the value of the fund every time a payment is made.
Size of the fund after six months is: 12.5 (1.05) = 13.125
Size of the fund after one year is: (13.125 + 6.6) 1.06 = 20.909
Size of the fund after two years is: (20.909 + 7) 1.065 = 29.723
Size of the fund after three years is: (29.723 + 8) 1.03 = 38.855
The TWRR is i where i is the solution to:
(1+i)3 = (13.125/12.5) [20.909/(13.125+6.6)] [29.723/(20.909+7)]
[38.855/(29.723+8)]
or just use the rates of return given to give:
(1+i)3 = 1.05 1.06 1.065 1.03
giving i = 6.879%

(iii)

For MWRR, we need to know the size of the fund at the end of the period. We
can use the values above to give:
MWRR is solution to: 12.5(1+i)3 + 6.6(1+i)2.5 + 7(1+i)2 + 8(1+i) = 38.855
Solve by iteration and interpolation, starting with i = 7%.
i = 7% gives LHS = 39.704
i = 6% gives LHS = 38.868
i = 5.5% gives LHS = 38.454

Interpolate between 5.5% and 6%.


i = 0.055 + 0.005 (38.855-38.454)/(38.868-38.454) = 5.98%

(iv)

(i) and (ii) are the same because there are no cash flows within sub-periods to
distort the LIRR away from the TWRR. The MWRR is lower because the
fund has a smaller amount of money in it at the beginning when rates of return
are higher.

Page 7

Subject CT1 (Financial Mathematics Core Technical) September 2007 Examiners Report

(i)

(1 + it ) ~ Lognormal ( , 2 )

ln (1 + it ) ~ N , 2
ln (1 + it )

10

= ln (1 + it ) + ln (1 + it ) + + ln (1 + it ) ~ N 10,10 2

since it ' s are independent

(1 + it )10 ~ Lognormal (10,102 )


[] for correct use of independence assumption

2
E (1 + it ) = exp +
= 1.06

( )

Var (1 + it ) = exp 2 + 2 exp 2 1 = 0.082

0.082

( )

= exp 2 1 2 = 0.0056798

1.06
2

0.0056798
0.0056798

exp +
= 0.055429
= 1.06 = ln1.06
2
2

10 = 0.55429 , 102 = 0.056798


Let S10 be the accumulation of one unit after 10 years:
0.056798

E ( S10 ) = exp 0.55429 +


= 1.790848
2

Expected value of investment = 2, 000, 000 E ( S10 ) = 3.5817 m


(ii)

We require P [ S10 < 0.8 1.790848 = 1.4327 ]


P [ ln S10 < ln1.4327 ] where ln S10 ~N(0.55429,0.056798)

ln1.4327 0.55429

P N ( 0,1) <

0.056798

P N ( 0,1) < 0.8171 = 0.207 21%

Page 8

Subject CT1 (Financial Mathematics Core Technical) September 2007 Examiners Report

10

(i)

(ii)

(a)

The flat rate of interest is: (2 2,400 2,000)/(2 2,000) = 70%

(b)

The flat rate of interest is not a good measure of the cost of borrowing
because it takes no account of the timing of payments and the timing of
repayment of capital.

If the consumers association is correct, then the present value of the


repayments is greater than the loan at 200%
i.e. 2, 000 < 2, 400

i
a2
12
d( )

12
i =2; a2 = 0.44444; d ( ) = 1.04982 gives RHS = 2,032

The consumers association is correct.


If the banks are correct, then the present value of the payments received by the
bank, after expenses, is less than the amount of the loan at a nominal (before
inflation) rate of interest of (1.01463 1.025 -1) per annum effective = 0.04.
i
i
i
i
i.e. 2, 000 > 720
a + 720
a + 960
a 0.3 2, 400
a
12 ) 2
12 ) 1.5
12 ) 1
12 ) 2
(
(
(
(
d
d
d
d

i
1 1.041.5
= 1.4283
= 1.021529; a2 = 1.8861; a1 = 0.9615; a1.5 =
12
0.04
d( )

So RHS = 720 1.021529 1.8861+ 720 1.021529 1.4283 +


960 1.021529 0.9615 0.3 2,400 1.021529 1.8861
= 1,387.23+ 1,050.52 + 942.91 1,387.23 = 1,993.43
Therefore, the banks are also correct.

11

(i)

Present value of the funds liabilities (in m) is:

100 v + 1.05v 2 + 1.052 v3 + + 1.0559 v60

= 100v 1 + 1.05v + (1.05v ) + + (1.05v )


2

59

( )
( )

1- 1.05

= 100 0.97087 1.03


1- 1.05

1.03

= 97.087 111.7795 = 10,852m


1 (1.05v )60
= 100v
1 1.05v

(ii)

60

Let the nominal holding of bonds = N in m


The present value of the bonds must equal 10,852m

Page 9

Subject CT1 (Financial Mathematics Core Technical) September 2007 Examiners Report

Therefore 0.04 Na20 + Nv 20 = 10,852 at 3%


a20 = 14.8775, v20 = 0.55368

So 10,852 = 0.04N 14.8775 + N 0.55368

N = 10,852 /(0.04 14.8775 + 0.55368) = 9,446.54m


(iii)

The numerator for the duration of the liabilities can be expressed as follows:
100v (1 1 + 1.05v 2 + 1.052v2 3++1.0559v59 60)
1.03
100 v (1.05v 1 + 1.052v2 2 + 1.053v3 3++1.0560v60 60)
1.05
The part inside the brackets can be regarded as ( Ia )60 evaluated at a rate of
=

interest i such that v = 1.05/1.03; the discount factor outside the brackets
should be evaluated at 3%
1.03
100
100 v =
= 95.2381
1.05
1.05
For the ( Ia )60 function, v = 1.019417; i = -0.019048; (1 + i ) a60 = 111.7727

( Ia )60

111.7727 60 1.01941760
= 4118.567
0.019048

Therefore numerator for duration is: 95.2381 4118.567 = 392,244


Therefore the duration is: 392,244/10,852 = 36.1 years.
(iv)

The duration of the assets can be expressed as the sum of payments times time
of receipt times present value factors divided by total present value.
The equation for the numerator is
0.04 9,446.54 ( Ia )20 + 9,446.54 20 v20 at 3%

( Ia )20 = 141.6761, v20 = 0.55368


Numerator is: 158,141
Therefore the duration is: 158,141/10,852 = 14.6 years.
(v)

Page 10

Duration of the liabilities is 36.1 years. Therefore volatility of the liabilities is:
36.1/1.03 = 35. If there were a reduction in interest rates to 1.5%, the liabilities
would increase in value by approximately 35 1.5 = 52.5%

Subject CT1 (Financial Mathematics Core Technical) September 2007 Examiners Report

Duration of the assets is 14.6 years. Therefore volatility of the assets is:
14.6/1.03 = 14.2. If there were a reduction in interest rates to 1.5%, the assets
would increase in value by approximately 14.2 1.5 = 21.3%.
The liabilities would increase in value by an additional 31.2% of their original
value i.e. by 3,386 more than the value of the assets.

END OF EXAMINERS REPORT

Page 11

Faculty of Actuaries

Institute of Actuaries

EXAMINATION
15 April 2008 (am)

Subject CT1 Financial Mathematics


Core Technical
Time allowed: Three hours
INSTRUCTIONS TO THE CANDIDATE
1.

Enter all the candidate and examination details as requested on the front of your answer
booklet.

2.

You must not start writing your answers in the booklet until instructed to do so by the
supervisor.

3.

Mark allocations are shown in brackets.

4.

Attempt all 10 questions, beginning your answer to each question on a separate sheet.

5.

Candidates should show calculations where this is appropriate.

Graph paper is not required for this paper.

AT THE END OF THE EXAMINATION


Hand in BOTH your answer booklet, with any additional sheets firmly attached, and this
question paper.
In addition to this paper you should have available the 2002 edition of the Formulae
and Tables and your own electronic calculator from the approved list.

CT1 A2008

Faculty of Actuaries
Institute of Actuaries

An eleven month forward contract is issued on 1 March 2008 on a stock with a price
of 10 per share at that date. Dividends of 50 pence per share are expected to be paid
on 1 April and 1 October 2008.
Calculate the forward price at issue, assuming a risk-free rate of interest of 5% per
annum effective and no arbitrage.
[4]

Describe the characteristics of the following investments:


(a)
(b)

Eurobonds
Certificates of deposit

[4]

A mortgage company offers the following two deals to customers for twenty-five year
mortgages.
Product A
A mortgage of 100,000 is offered with level repayments of 7,095.25 made annually
in arrear. There are no arrangement or exit fees.
Product B
A mortgage of 100,000 is offered whereby a monthly payment in advance is
calculated such that the customer pays an effective rate of return of 4% per annum
ignoring arrangement and exit fees. In addition the customer also has to pay an
arrangement fee of 6,000 at the beginning of the mortgage and an exit fee of 5,000
at the end of the twenty-five year term of the mortgage.
Compare the annual effective rates of return paid by customers on the two products.
[8]

A loan of nominal amount 100,000 is to be issued bearing coupons payable quarterly


in arrear at a rate of 7% per annum. Capital is to be redeemed at 108% on a coupon
date between 15 and 20 years after the date of issue, inclusive. The date of redemption
is at the option of the borrower.
An investor who is liable to income tax at 25% and capital gains tax at 35% wishes to
purchase the entire loan at the date of issue.
Calculate the price which the investor should pay to ensure a net effective yield of at
least 5% per annum.
[8]

CT1 A20082

The n year spot rate of interest, in , is given by:


in = a bn
for n = 1, 2 and 3, and where a and b are constants.
The one-year forward rates applicable at time 0 and at time 1 are 6.1% per annum
effective and 6.5% per annum effective respectively. The 4year par yield is 7% per
annum.
Stating any assumptions:

(i)

calculate the values of a and b.

(ii)

calculate the price per 1 nominal at time 0 of a bond which pays annual
coupons of 5% in arrear and is redeemed at 103% after 4 years.
[5]
[Total 9]

(i)

An investor is considering the purchase of an annuity, payable annually in


arrear for 20 years. The first payment is 500. Using a rate of interest of 8%
per annum effective, calculate the duration of the annuity when:
(a)
(b)

(ii)

[4]

the payments remain level over the term.


the payments increase at a rate of 8% per annum compound.

Explain why the answer in (i)(b) is higher than the answer in (i)(a).

[6]
[2]
[Total 8]

The shares of a company currently trade at 2.60 each, and the company has just paid
a dividend of 12p per share. An investor assumes that dividends will be paid annually
in perpetuity and will grow in line with a constant rate of inflation. The investor
estimates the assumed inflation rate from equating the price of the share with the
present value of all estimated future gross dividend payments using an effective
interest rate of 6% per annum.
(i)

(ii)

Calculate the investors estimation of the effective inflation rate per


annum based on the above assumptions.

[4]

Suppose that the actual inflation rate turns out to be 3% per annum effective
over the following twelve years, but that all the investors other assumptions
are correct.
Calculate the investors real rate of return per annum from purchase to sale, if
she sold the shares after twelve years for 5 each immediately after a dividend
has been paid. You may assume that the investor pays no tax.
[6]
[Total 10]

CT1 A20083

PLEASE TURN OVER

An investor is considering investing in a capital project.


The project requires an outlay of 500,000 at outset and further payments at the end
of each of the first 5 years, the first payment being 100,000 and each successive
payment increasing by 10,000.
The project is expected to provide a continuous income at a rate of 80,000 in the first
year, 83,200 in the second year and so on, with income increasing each year by 4%
per annum compound. The income is received for 25 years.
It is assumed that, at the end of 15 years, a further investment of 300,000 will be
required and that the project can be sold to another investor for 700,000 at the end of
25 years.

(i)

Calculate the net present value of the project at a rate of interest of 11% per
annum effective.
[9]

(ii)

Without doing any further calculations, explain how the net present value
would alter if the interest rate had been greater than 11% per annum effective.
[3]
[Total 12]

The force of interest, ( t ) , is a function of time and at any time t, measured in years,
is given by the formula:
0.06

( t ) = 0.10 0.01t
0.01t 0.04

0t 4
4<t 7
7<t

(i)

Calculate the value at time t = 5 of 1,000 due for payment at time t = 10. [5]

(ii)

Calculate the constant rate of interest per annum convertible monthly which
leads to the same result as in (i) being obtained.
[2]

(iii)

Calculate the accumulated amount at time t = 12 of a payment stream, paid


continuously from time t = 0 to t = 4, under which the rate of payment at time t
is ( t ) = 100e0.02t .
[6]

[Total 13]

CT1 A20084

10

An insurance company holds a large amount of capital and wishes to distribute some
of it to policyholders by way of two possible options.
Option A
100 for each policyholder will be put into a fund from which the expected annual
effective rate of return from the investments will be 5.5% and the standard deviation
of annual returns 7%. The annual effective rates of return will be independent and
(1+ it ) is lognormally distributed, where it is the rate of return in year t. The
policyholder will receive the accumulated investment at the end of ten years.
Option B
100 will be invested for each policyholder for five years at a rate of return of 6% per
annum effective. After five years, the accumulated sum will be invested for a further
five years at the prevailing five-year spot rate. This spot rate will be 1% per annum
effective with probability 0.2, 3% per annum effective with probability 0.3, 6% per
annum effective with probability 0.2, and 8% per annum effective with probability
0.3. The policyholder will receive the accumulated investment at the end of ten years.
Deriving any necessary formulae:
(i)

Calculate the expected value and the standard deviation of the sum the
policyholders will receive at the end of the ten years for each of options A and
B.
[17]

(ii)

Determine the probability that the sum the policyholders will receive at the
end of ten years will be less than 115 for each of options A and B.
[5]

(iii)

Comment on the relative risk of the two options from the policyholders
perspective.
[2]
[Total 24]

END OF PAPER

CT1 A20085

Faculty of Actuaries

Institute of Actuaries

Subject CT1 Financial Mathematics


Core Technical
EXAMINERS REPORT
April 2008

Introduction
The attached subject report has been written by the Principal Examiner with the aim of
helping candidates. The questions and comments are based around Core Reading as the
interpretation of the syllabus to which the examiners are working. They have however given
credit for any alternative approach or interpretation which they consider to be reasonable.

M A Stocker
Chairman of the Board of Examiners
June 2008

Faculty of Actuaries
Institute of Actuaries

Subject CT1 (Financial Mathematics Core Technical) April 2008 Examiners Report

Comments
Comments on solutions presented to individual questions for this April 2008 paper are given
below.
Please note that different answers may be obtained to those shown in these solutions
depending on whether figures obtained from tables or from calculators are used in the
calculations but candidates are not penalised for this. However, candidates may be penalised
where excessive rounding has been used or where insufficient working is shown.
Question 1

Well answered.

Question 2

As has often been the case when words rather than numbers have been
required, this bookwork question was answered poorly.

Question 3

Generally well answered, although some students treated the fees on Product
B paid by the customer as a cost to the mortgage company.

Question 4

Well answered although many candidates working was unclear when


performing the CGT test.

Question 5

Part (i) was answered well but in part (ii) many candidates failed to recognise
the need to calculate the 4-year spot rate before calculating the bond price.

Question 6

Part (i) of this question did appear to differentiate between stronger


candidates who often scored very well and weaker candidates who often failed
to score at all. As with many previous diets, many candidates in part (ii) had
difficulty in giving a clear explanation of their results.

Question 7

This question was answered relatively poorly with, particularly in part (ii),
candidates often appearing confused between real and money rates of interest.

Question 8

Most candidates managed to make a reasonable attempt at this question


although marks were often lost in part (i) through a combination of
calculation errors and insufficient working being shown. Candidates generally
made a better attempt at the explanation required in part (ii) when compared
to similar questions both on this paper and in previous diets.

Question 9

Well answered.

Question 10

Part (i) (for Option A) can be done much more simply than by using the
method given in this report but the calculations given would still need to be
done for part (ii). It was disappointing to see many candidates incorrectly
calculate the mean accumulated value for Option B by using the mean rate of
interest. Few candidates brought together the answers from (i) and (ii) to fully
answer part (iii).

Page 2

Subject CT1 (Financial Mathematics Core Technical) April 2008 Examiners Report

The present value of the dividends, I, is:


I = 0.5v

1
12

+ 0.5v

12

= 0.5 ( 0.99594 + 0.97194 ) = 0.98394 calculated at i = 5%

Hence forward price is (again calculated at i = 5%):


F = (10 0.98394 )(1 + i )

11
12

= 9.42845

= 9.43

(a)

Eurobonds

form of unsecured medium or long-term borrowing

issued in a currency other than the issuer's home currency outside the
issuer's home country

pay regular interest payments and a final capital repayment at par.

issued by large companies, governments and supra-national organisations.

yields depend upon the issuer and issue size but will typically be slightly
lower than for the conventional unsecured loan stocks of the same issuer.

issuers have been free to add novel features to their issues in order to
make them appeal to different investors.

usually issued in bearer form


Certificates of Deposit

a certificate stating that some money has been deposited

issued by banks and building societies

terms to maturity are usually in the range 28 days to 6 months.

interest is payable on maturity

security and marketability will depend on the issuing bank

active secondary market

(b)

For Product A, the annual rate of return satisfies the equation:


7, 095.25a25 = 100, 000
a25 = 14.0939
This equates to the value of a25 at 5%. Hence the annual effective rate of return is
5%.
For Product B, the annual rate of payment is X such that:
Xa

(12 )

25

(12 )

25

= 100, 000 at 4%

X=

i
a = 1.021537 15.6221 = 15.95855
12 25
d( )
100,000
= 6, 266.23
15.95855

Page 3

Subject CT1 (Financial Mathematics Core Technical) April 2008 Examiners Report

The equation of value to calculate the rate of return from Product B is:
i
6,000 + 5,000v 25 + 6, 266.23
a = 100, 000
12 25
(
d )

Clearly the rate of return must be greater than 4%. Try 5%.
LHS = 6, 000 + 5, 000 0.29530 + 6,266.2335 1.02688114.0939 = 98,166
At 5% the present value of the payments is less than the amount of the loan at 5% so
the rate of return must be less than 5%. Try 4%:
LHS = 6, 000 + 5, 000 0.37512 + 100, 000 = 107,876

Interpolate between 4% and 5% to get the effective rate of return, i:


107,876 100,000
i = 0.04 + 0.01
4.81% (actual answer is 4.80%)
107,876 98,166
Therefore Product B charges a lower effective annual return than Product A.

i ( 4)
( 4)
1 +
= 1.05 i = 0.049089
4

0.07
g (1 t1 ) =
0.75 = 0.04861
1.08
4

i ( 4) > (1 t1 ) g
Capital gain on contract and we assume loan is redeemed as late as possible (i.e.
after 20 years) to obtain minimum yield.
Let Price of stock = P
( 4)
P = 0.07 100, 000 0.75 a20

+ (108, 000 0.35 (108, 000 P ) ) v 20 at 5%


( )
5250a20
+ 70, 200v 20
4

P=

5250 1.018559 12.4622 + 70, 200 0.37689


1 0.35 0.37689

= 107,245.38

Page 4

1 0.35v 20

Subject CT1 (Financial Mathematics Core Technical) April 2008 Examiners Report

Assuming no arbitrage.
(i)

i1 = f 0 and (1 + i2 ) = (1 + i1 )(1 + f1 ) .
2

Hence a b = 0.061
a = b + 0.061

(1 + a 2b )

= 1.061 1.065

1 + a 2b = 1.061 1.065

b = 0.002
a = 0.059
(ii)

Firstly, find the 4-year spot rate. Consider 1 nominal:

(v

1 = 0.07

i1

+ vi22 + vi33 + vi44 + vi44

= 0.07 (1.0611 + 1.0632 + 1.0653 ) + 1.07 vi44


(1 + i4 ) = 1.31429212
4

i4 = 7.0713% p.a
Let bond price per 1 nominal be P. Then

P = 0.05 vi1 + vi22 + vi33 + vi44 + 1.03vi44

= 0.05 1.0611 + 1.063 + 1.0653 + 1.08 1.0707134


= 0.9545
i.e. 95.45 pence per 1 nominal

(i)

(a)

The duration is:

500 v + 2v 2 + 3v3 + + 20v 20


500(v + v 2 + v3 + + v 20 )
=

( Ia )20
a20

) at 8%

78.9079
= 8.037 years
9.8181

Page 5

Subject CT1 (Financial Mathematics Core Technical) April 2008 Examiners Report

(b)

(ii)

(i)

The duration is:


500 v + 1.08 2v 2 + 1.082 3v3 + + 1.0819 20v 20

at 8%
2
2
3
19
20
500 v + 1.08v + 1.08 v + + 1.08 v

1
v (1 + 2 + 3 + + 20 ) 2 ( 20 21)
=
=
= 10.5 years
20v
20

) (
( ) (

The duration in (i)(b) is higher because the payments increase over time so
that the weighting of the payments is further towards the end of the series.

260 = 12 v (1 + e ) + v 2 (1 + e ) + v 3 (1 + e ) + ......
2

where v =

1
and e denotes inflations rate.
1.06

Then,
260 = 12a

at j % where

12
j
j = 0.046153846
e = 0.01324

1
1+ e
0.06 e
i.e. j =
=
1+ j 1+ i
1+ e

260 =

(ii)

i.e 1.324% pa

260 = 12 (1.03v + 1.032 v 2 + ..... + 1.0312 v12 ) +500v12


= 12 a12 + 500 v12 where j =
j%

i%

i 0.03
1.03

Try i = 10%, RHS = 255.67


i = 9%, RHS = 279.35
279.35 260
0.01
Hence, i = 0.09 +
279.35 255.67
= 0.098
Let i = real return
Then (1 + i )(1 + e ) = 1 + i

1 + i =

Page 6

1.0982
i = 6.62% pa
1.03

Subject CT1 (Financial Mathematics Core Technical) April 2008 Examiners Report

(i)

Working in 000s
Outlay
Pv = 500 + 90a5 + 10 ( Ia )5 @11%

a5 =

( Ia )5

1 v5
= 3.695897
0.11
=

a5 5v5
0.11

1.11 3.695897 5v5


0.11

= 10.319900
PV = 500 + 90 3.695897 + 10 10.3199

= 935.8297
Income

PV = 80 a1 + 1.04v a1 + (1.04 ) v 2 a1 + " " + (1.04 ) v 24 a1


2

24

1 (1.04v )25
= 80a1

1 (1.04v )
where a1 =

.v =

0.11 1
= 0.949589
.
ln1.11 1.11

PV = 80 0.949589 12.74554 = 968.2421

PV of cost of further investment


= 300v15 = 62.7013
PV of sale = 700v 25 = 51.5257
Hence NPV = 968.2421 + 51.5257 - 935.8297 - 62.7013
= 21.2368 (21,237)
(ii)

If interest > 11% then

1
decreases.
1+ i

PV of both income and outgo

Page 7

Subject CT1 (Financial Mathematics Core Technical) April 2008 Examiners Report

However, PV of outgo is dominated by initial outlay of 500k at time 0 which


is unaffected.
PV of income decreases by more than decrease in PV of outgo

NPV = PV of income PV of outgo


would reduce (and possibly become negative)

(i)

10
7
pv = 1, 000 exp ( 0.01t 0.04 ) dt exp ( 0.10 0.01t ) dt
7

10
7
0.01t 2


0.01t 2
= 1000 exp
0.04t exp 0.10t

2


2 5

0.01 51

0.01 24

= 1000 exp
0.04 3 exp 0.10 2

2
2

= 1000 exp ( 0.255 + 0.12 0.20 + 0.12 )


= 1000 exp ( 0.215 )

= 806.54
(ii)

Required interest rate p.a. convertible monthly is given by


125

i (12)
806.54 1 +
= 1, 000
12

i (12) = 4.3077% p.a. convertible monthly

(iii)

Accumulated amount =

100e

0.02 t

12

= 100 e

0.02 t

= 100 e0.02t e(
4

[0.06 r ]t4

0.24 0.06 t )

e(

0.10 r 0.01 r 2
2
4

0.30 0.165)

= 100e0.24 e0.135e0.275 e 0.04t dt


0

e 0.04t
= 100e

0.04 0
= 2,500e0.65 (1 e0.16 )
0.65

Page 8

e(

12

0.06 dr
( 0.10 0.01r ) dr
e t
e 4
e 7

0.01 r 2 0.04 r
2
7

0.475 0.200 )

dt

dt

( 0.01r 0.04 ) dr

dt

Subject CT1 (Financial Mathematics Core Technical) April 2008 Examiners Report

= 2,500 * 1.915540829 * 0.1478562


= 708.06

10

(i)

Option A:

(1 + it ) ~ Lognormal ( , 2 )

ln (1 + it ) ~ N , 2
ln (1 + it )

10

= ln (1 + it ) + ln (1 + it ) + + ln (1 + it ) ~ N 10,102

since it ' s are independent

(1 + it )10 ~ Lognormal (10,102 )

2
E (1 + it ) = exp +
= 1.055

Var (1 + it ) = exp 2 + 2 exp 2 1 = 0.07 2

0.07 2

( )

( )

= exp 2 1 2 = 0.0043928

1.055
2

0.0043928
0.0043928

exp +
= 0.051344
= 1.055 = ln1.055
2
2

10 = 0.51344 , 102 = 0.043928


Let S10 be the accumulation of one unit after 10 years:
0.043928

E ( S10 ) = exp 0.51344 +


= 1.70814
2

Accumulated sum is 100 E ( S10 ) = 170.81


Option B:
The accumulated sum at the end of five years is:
100 1.065 = 100 1.33823 = 133.823

Page 9

Subject CT1 (Financial Mathematics Core Technical) April 2008 Examiners Report

The expected value of the accumulated sum at the end of ten years is:

133.823 0.2 1.015 + 0.3 1.035 + 0.2 1.065 + 0.3 1.085

= 133.823 ( 0.2 1.05101 + 0.3 1.15927 + 0.2 1.33823 + 0.3 1.46933)


= 169.48
Option A:
Var ( S10 ) = exp ( 2 0.51344 + 0.043928 ) exp ( 0.043928 ) 1
= 2.91776 0.04491 = 0.13103
Therefore standard deviation of 100 is 100 0.13103 = 36.20
Option B:
Here we need to find the expected value of the square of the accumulation as
follows:

133.8232 0.2 1.051012 +0.3 1.15927 2 +0.2 1.338232 +0.3 1.469332

= 29,189.86
The variance of the accumulation is therefore:
29,189.86 169.482 = 2 467.54
and the standard deviation is 21.62
(ii)

For option A we require P [ S10 < 1.15]


P [ ln S10 < ln1.15] where ln S10 ~N(0.51344,0.043928)

ln1.15 0.51344

P N ( 0,1) <

0.043928

P N ( 0,1) < 1.7829 = 0.0373 4%


For option B we first examine the lowest payout possible.
There is a probability of 0.2 that the amount will be 100 1.065 1.015 or less
which equals 133.823 1.05101 = 140.65 . Therefore the probability of a
payment of less than 115 is zero.
(iii)

Option A is riskier both from the perspective of having a higher standard


deviation of return and also a higher probability of a very low value.

END OF EXAMINERS REPORT

Page 10

Faculty of Actuaries

Institute of Actuaries

EXAMINATION
23 September 2008 (am)

Subject CT1 Financial Mathematics


Core Technical
Time allowed: Three hours
INSTRUCTIONS TO THE CANDIDATE
1.

Enter all the candidate and examination details as requested on the front of your answer
booklet.

2.

You must not start writing your answers in the booklet until instructed to do so by the
supervisor.

3.

Mark allocations are shown in brackets.

4.

Attempt all 12 questions, beginning your answer to each question on a separate sheet.

5.

Candidates should show calculations where this is appropriate.

Graph paper is not required for this paper.

AT THE END OF THE EXAMINATION


Hand in BOTH your answer booklet, with any additional sheets firmly attached, and this
question paper.
In addition to this paper you should have available the 2002 edition of the Formulae
and Tables and your own electronic calculator from the approved list.

CT1 S2008

Faculty of Actuaries
Institute of Actuaries

A 91-day government bill is purchased for 95 at the time of issue and is redeemed at
the maturity date for 100. Over the 91 days, an index of consumer prices rises from
220 to 222.
Calculate the effective real rate of return per annum.

[3]

(i)

State the strengths and weaknesses of using the money-weighted rate of return
as opposed to the time-weighted rate of return as a measure of an investment
managers skill.
[3]

(ii)

An investor had savings totalling 41,000 in an account on 1 January 2006.


He invested a further 12,000 in this account on 1 August 2006. The total
value of the account was 45,000 on 31 July 2006 and was 72,000 on 31
December 2007.
Assuming that the investor made no further deposits or withdrawals in relation
to this account, calculate the annual effective time-weighted rate of return for
the period 1 January 2006 to 31 December 2007.
[2]
[Total 5]

(i)

A forward contract with a settlement date at time T is issued based on an


underlying asset with a current market price of B.
The annualised risk-free force of interest applying over the term of the forward
contract is and the underlying asset pays no income. Show that the
[3]
theoretical forward price is given by K = Be T , assuming no arbitrage.

(ii)

An asset has a current market price of 200p, and will pay an income of 10p in
exactly three months time.
Calculate the price of a forward contract to be settled in exactly six months,
assuming a risk-free rate of interest of 8% per annum convertible quarterly. [3]
[Total 6]

Describe the characteristics of commercial property (i.e. commercial real estate) as an


investment.
[5]

A bank offers two repayment alternatives for a loan that is to be repaid over ten years.
The first requires the borrower to pay 1,200 per annum quarterly in advance and the
second requires the borrower to make payments at an annual rate of 1,260 every
second year in arrears.
Determine which terms would provide the best deal for the borrower at a rate of
interest of 4% per annum effective.

CT1 S20082

[5]

A pension fund holds an asset with current value 1 million. The investment return
on the asset in a given year is independent of returns in all other years. The annual
investment return in the next year will be 7% with probability 0.5 and 3% with
probability 0.5. In the second and subsequent years, annual investment returns will be
2%, 4% or 6% with probability 0.3, 0.4 and 0.3, respectively.
(i)

Calculate the expected accumulated value of the asset after 10 years, showing
all steps in your calculations.
[3]

(ii)

Calculate the standard deviation of the accumulated value of the asset after 10
years, showing all steps in your calculations.
[4]

(iii)

Without doing any further calculations explain how the mean and variance of
the accumulation would be affected if the returns in years 2 to 10 were 1%,
4%, or 7%, with probability 0.3, 0.4 and 0.3 respectively.
[2]
[Total 9]

The force of interest, (t ) , is a function of time and at any time t (measured in years)
is given by
0.05 + 0.02t
(t ) =
0.15

for 0 t 5
for t > 5

(i)

Calculate the present value of 1,000 due at the end of 12 years.

[5]

(ii)

Calculate the annual effective rate of discount implied by the transaction in (i).
[2]
[Total 7]

A tax advisor is assisting a client in choosing between three types of investment. The
client pays tax at 40% on income and 40% on capital gains.
Investment A requires the investment of 1m and provides an income of 0.1m per
year in arrears for ten years. Income tax is deducted at source. At the end of the ten
years, the investment of 1m is returned.
In Investment B, the initial sum of 1m accumulates at the rate of 10% per annum
compound for ten years. At the end of the ten years, the accumulated value of the
investment is returned to the investor after deduction of capital gains tax.
Investment C is identical to Investment B except that the initial sum is deemed, for tax
purposes, to have increased in line with the index of consumer prices between the date
of the investment and the end of the ten-year period. The index of consumer prices is
expected to increase by 4% per annum compound over the period.
(i)

Calculate the net rate of return expected from each of the investments.

(ii)

Explain why the expected rate of return is higher for Investment C than for
Investment B and is higher for Investment B than for Investment A.
[3]
[Total 10]

CT1 S20083

[7]

PLEASE TURN OVER

Three bonds, paying annual coupons in arrears of 6%, are redeemable at 105 per
100 nominal and reach their redemption dates in exactly one, two and three years
time respectively. The price of each of the bonds is 103 per 100 nominal.
(i)

Calculate the gross redemption yield of the three-year bond.

[3]

(ii)

Calculate to three decimal places all possible spot rates, implied by the
information given, as annual effective rates of interest.

[4]

(iii)

10

Calculate to three decimal places all possible forward rates, implied by the
information given, as annual effective rates of interest.
[4]
[Total 11]

An insurance company is considering two possible investment options.


The first investment option involves setting up a branch in a foreign country. This will
involve an immediate outlay of 0.25m, followed by investments of 0.1m at the end
of one year, 0.2m at the end of two years, 0.3m at the end of three years and so on
until a final investment is made of 1m in ten years time. The investment will
provide annual payments of 0.5m for twenty years with the first payment at the end
of the eighth year. There will be an additional incoming cash flow of 5m at the end
of the 27th year.
The second investment option involves the purchase of 1 million shares in a bank at a
price of 4.20 per share. The shares are expected to provide a dividend of 21p per
share in exactly one year, 22.05p per share in two years and so on, increasing by 5%
per annum compound. The shares are expected to be sold at the end of ten years, just
after a dividend has been paid, for 5.64 per share.
(i)

Determine which of the options has the higher net present value at a rate of
interest of 7% per annum effective.
[9]

(ii)

Without doing any further calculations, determine which option has the higher
discounted mean term at a rate of interest of 7% per annum effective.
[2]
[Total 11]

CT1 S20084

11

A company has a liability of 400,000 due in ten years time.


The company has exactly enough funds to cover the liability on the basis of an
effective interest rate of 8% per annum. This is also the interest rate on which current
market prices are calculated and the interest rate earned on cash.
The company wishes to hold 10% of its funds in cash, and to invest the balance in the
following securities:

a zero-coupon bond redeemable at par in twelve years time

a fixed-interest stock which is redeemable at 110% in sixteen years time bearing


interest at 8% per annum payable annually in arrear

(i)

Calculate the nominal amounts of the zero-coupon bond and the fixed-interest
stock which should be purchased to satisfy Redingtons first two conditions
for immunisation.
[10]

(ii)

Calculate the amount which should be invested in each of the assets mentioned
in (i).
[2]

(iii)

Explain whether the company would be immunised against small changes in


the rate of interest if the quantities of stock in part (i) are purchased.
[2]
[Total 14]

CT1 S20085

PLEASE TURN OVER

12

An individual takes out a 25-year bank loan of 300,000 to purchase a house.


The individual agrees to pay only the interest payments, monthly in arrear, for the first
15 years whereupon he repays half of the capital as a lump sum. He then pays only
the interest for the remaining 10 years, quarterly in arrear, and repays the other half of
the capital as a lump sum at the end of the term.
(i)

(ii)

Calculate the total amount of interest paid by the individual, assuming an


effective rate of interest of 8% p.a.

The individual believes that he can earn a nominal rate of interest convertible
half-yearly of 9% p.a. from a separate savings account.
Calculate the level contribution he must make monthly in advance to the
savings account in order to repay half the capital after 15 years.

(iii)

[5]

[4]

The individual made the monthly contributions calculated in (ii) to the savings
account. However, over the first 15 years, the effective rate of return earned
on the savings account was 10% per annum.
The individual used the proceeds at that time to repay as much of the loan as
possible and then decided to repay the remainder of the loan by level
instalments of interest and capital. After the first 15 years, the effective rate of
interest changed to 7% per annum.
Calculate the level payment he must make, payable monthly in arrear, to repay
the loan over the final 10 years of the loan.
[5]
[Total 14]

END OF PAPER

CT1 S20086

Faculty of Actuaries

Institute of Actuaries

Subject CT1 Financial Mathematics


Core Technical
EXAMINERS REPORT
September 2008

Introduction
The attached subject report has been written by the Principal Examiner with the aim of
helping candidates. The questions and comments are based around Core Reading as the
interpretation of the syllabus to which the examiners are working. They have however given
credit for any alternative approach or interpretation which they consider to be reasonable.

R D Muckart
Chairman of the Board of Examiners
November 2008

Faculty of Actuaries
Institute of Actuaries

Subject CT1 (Financial Mathematics Core Technical) September 2008 Examiners Report

Comments
Please note that different answers may be obtained to those shown in these solutions
depending on whether figures obtained from tables or from calculators are used in the
calculations but candidates are not penalised for this. However, candidates may be penalised
where excessive rounding has been used or where insufficient working is shown.
Candidates appeared to be less well prepared than in previous recent diets. As has often been
the case when words rather than numbers have been required, Q4 was answered relatively
poorly despite only involving bookwork with a wide range of available points that could be
made. Many candidates also struggled with the first part of Q2 where explanation rather
than calculation was required. The remainder of the shorter questions were answered well
with candidates scoring particularly highly on Q7.
The more application styled questions (especially Qs 8, 11 and 12) tended to act as a clear
discriminator between stronger and weaker candidates with a significant minority of
candidates scoring very few marks on these questions. By contrast, Q9 on spot and forward
yields was answered relatively well compared to questions in previous diets on this topic.

Page 2

Subject CT1 (Financial Mathematics Core Technical) September 2008 Examiners Report

If j = real rate of return then equation of value in real terms is:

95 (1 + j )

91/ 365

= 100

220
222

(1 + j )91/ 365 = 1.04315


therefore j = 18.465%

(i)

MWRR
Requires less information compared to TWRR
But
Affected by amount and timing of net cashflows, which may not be in the
managers control and less fair measure than TWRR
More difficult equation to solve than TWRR
Also: equation may not have unique (or any) solution

(ii)

Let TWRR = i
Then
45 72

41 57
= 1.386392811

(1 + i ) 2 =

i = 17.745% p.a.

(i)

Consider two portfolios A and B at time 0.


Portfolio A: - buy forward at price of K
- deposit Ke T in risk-free asset
Portfolio B: - buy asset at price of B
Then, at maturity, both portfolios have the same value (i.e. hold the underlying
asset).
Thus, by the no-arbitrage principle, both portfolios must have same value at
time 0.
Ke T = B K = BeT

Page 3

Subject CT1 (Financial Mathematics Core Technical) September 2008 Examiners Report

i = 2% per quarter

(ii)

K = 200 (1.02 ) 10 1.02 = 197.88


2

( using K = Be

T t
Ce ( 1 )

Main characteristics of commercial property investments:

Many different types of properties available for investment, e.g. offices, shops and
industrial properties.
Return comes from rental income and from the proceeds on sale.
Total expected return higher than for gilts
Rents and capital values are expected to increase broadly with inflation in the long
term
Neither rental income nor capital values are guaranteed capital values in
particular can fluctuate in the short term
but rental income more secure than dividends
Rents and capital values expected to increase when the price level rises (though
the relationship is far from perfect).
Rental terms are specified in lease agreements. Typically, rents increase every
three to five years, Some leases have clauses which specify upward-only
adjustments of rents.
Large unit sizes, leading to less flexibility than investment in shares
Each property is unique
. so can be difficult to value.
Valuation is expensive, because of the need to employ an experienced surveyor
Marketability and liquidity are poor because of uniqueness
and because buying and selling incurs high costs.
Rental income received gross of tax.
Net rental income may be reduced by maintenance expenses
There may be periods when the property is unoccupied, and no income is
received.
The running yield from property investments will normally be higher than that for
ordinary shares.

Present value in first case is


1, 200

i
a10 = 1200 1.024877 8.1109 = 9,975.210
4
d( )

Present value in second case is:

(1 v )

(1 v )
10

2,520 (v

Page 4

+ v

++ v ) = 2,520 v
10

Subject CT1 (Financial Mathematics Core Technical) September 2008 Examiners Report

= 2,520 0.92456

(1 0.67556 ) = 10, 020.01


(1 0.92456 )

Therefore first option is better for the borrower.

(i)

Let it = investment return for year t


Then, the expected value of the accumulation ( S10 ) is given by (in millions):
10

E ( S10 ) = E (1 + it )

t =1

10

= E (1 + it ) using independence
t =1
10

= (1 + E ( it ) )
t =1

Now, E ( i1 ) = 0.5 ( 0.07 + 0.03) = 0.05

and for t 1, E ( it ) = ( 0.3 0.02 + 0.4 0.04 + 0.3 0.06 )


= 0.04
So the expected value of the accumulation is
1.05 1.049 = 1.494477 (i.e. 1,494,477)

(ii)

The variance of the accumulation is

( ( )

2
1, 000, 0002 E S10
E ( S10 )

10
2
2
= E (1 + it )
where E S10

t =1

10

= E 1 + 2it + it2

t =1

( )
(

(1 + 2 E ( it ) + E ( it2 ) ) from independence


10

t =1

Page 5

Subject CT1 (Financial Mathematics Core Technical) September 2008 Examiners Report

( )

Now E i12 = 0.5 0.07 2 + 0.032 = 0.0029

( )

for t 1, E it 2 = 0.3 0.022 + 0.4 0.042 + 0.3 0.062


= 0.00184
Hence,

( )

2
E S10
= (1 + 0.1 + 0.0029 ) (1 + 0.08 + 0.00184 )

= 2.238739
Standard deviation of the accumulation is

1, 000, 000 2.238739 1.494477 2

= 72, 646

(iii)

The mean would remain unchanged as the expected rate of return in years 2-10
is unchanged. The variance of the rate in years 2-10 has increased and this will
lead to an increase in the variance of the 10 year accumulation.

(i)

Discounting from t = 12 to t = 5
12
v (12,5 ) = exp 0.15ds
5

= exp [ 0.15s ]5 = e 1.05 = 0.34994


12

Discounting from t = 5 to t = 0
5
v ( 5, 0 ) = exp 0.05 + 0.02 sds
0

= exp 0.05s 0.01s 2 = e 0.5 = 0.60653

0
Hence present value of 1,000 at time t = 12
= 1, 000v (12,5 ) v ( 5, 0 ) = 1, 000 0.34994 0.60653 = 212.25

Page 6

Subject CT1 (Financial Mathematics Core Technical) September 2008 Examiners Report

(ii)

The annual effective rate of discount is d such that:


1000 (1 d )

12

= 212.25

d = 1 0.21225

(i)

1
12

= 12.117%

Investment A: the gross rate of return per annum effective is clearly 10%. The
net return is therefore (1-0.4 ) 10% = 6% per annum effective.
Investment B: the investment will accumulate to 1m 1.110 = 2.5937 m at the
end of the ten years. The equation of value is:
1 = 2.59374 (1 + i )
= 1.95625 (1 + i )
(1 + i )

10

10

0.4 ( 2.59374 -1)(1 + i )

10

10

= 1.95625

i = 6.94%

Investment C: again the investment will accumulate to 2.5937m at the end of


ten years. However, the indexed purchase price is subtracted from the value of
the investment in this case. Thus the equation of value is:
1 = 2.59374 (1 + i )
= 2.5937 (1 + i )

10

= 2.14834 (1 + i )
(1 + i )

10

10

0.4 2.59374 -1 1.0410 (1 + i )

0.4 2.59374 (1 + i )

10

10

+ 0.4 1.0410 (1 + i )

10

10

= 2.14834

i = 7.95%

(ii)

All investments give a gross return of 10% per annum effective. Investment B
gives a higher return than A because the tax is deferred until the end of the
investment as capital gains tax is paid and not income tax. [However,
candidates might note that tax is paid on the interest earned by deferral of tax].
Investment C gives a higher return than investment B because the tax is only
paid on the real return over the ten year period which is lower than the
nominal return.

Page 7

Subject CT1 (Financial Mathematics Core Technical) September 2008 Examiners Report

(i)

103 = 6a3 + 105v3

try i = 6%: a3 = 2.6730 v3 = 0.83962


RHS = 104.1981
try i = 7%: a3 = 2.6243 v3 = 0.81630
RHS = 101.4573
Using linear interpolation:

i = 0.06 +

(ii)

(104.1981 103)

(104.1981 101.4573)

0.01 = 0.06437 = 6.44%

Let in = spot yield for term n


Then
103 (1 + i1 ) = 111 i1 = 7.767%

(iii)

103 = 6 (1.07767 )

103 = 6 (1.07767 )

+ 111(1 + i2 )

i2 = 6.736%

+ 6 (1.06736 )

+ 111(1 + i3 )

i3 = 6.394%

First year forward rate is 7.767% (same as spot rate).


Forward rate from time one to time two is i such that:
1.07767 (1 + i ) = 1.067362 i = 5.715%

Forward rate from time two to time three is i such that:


1.067362 (1 + i ) = 1.063943 i = 5.713%
Forward rate from time one to time three is i such that:
1.07767 (1 + i ) = 1.063943 i = 5.714%
2

Forward rate from time zero to two and from time zero to three are the same as
the respective spot rates (no additional marks for this point).

10

(i)

NPV of first project in m is:

0.5 a27 a7 + 5v 27 0.1( Ia )10 0.25 at 7%


= 0.5 (11.9867 5.3893) + 5 0.16093 0.1 34.7391 0.25
= 0.379m

Page 8

Subject CT1 (Financial Mathematics Core Technical) September 2008 Examiners Report

The NPV of second project in m is:


0.21v + 0.21(1.05)v 2 + 0.21(1.05) 2 v3 + + 0.21(1.05)9 v10 + 5.64v10 4.2
1 1.0510 v10
= 0.21v
+ 5.64v10 4.2
1 1.05v

v = 0.93458 v10 = 0.50835

Therefore NPV = 1.8055 + 5.64 0.50835 4.2 = 0.473m


The second project has the higher net present value at 7% per annum effective.

11

(ii)

The second project clearly has a discounted mean term of less then ten years.
However, the discounted mean term of the first project must be greater than
ten years because the undiscounted incoming cash flows are less than the
undiscounted outgoing cash flows after ten years.

(i)

Working in 000s
Let X = Nominal amount of Zero Coupon Bond
Y = Nominal amount of 8% bond
VL = 400v10 = 185.2774
VA = 18.52774 + Xv12 + 0.08Ya16 +1.1Yv16

Then, since VA = VL (1st condition)


166.74966 = 0.39711 X + 0.08 8.8514 Y + 0.32108 Y
166.74966 = 0.39711 X + 1.02919 Y ...... (1)

2nd condition is VA' = VL'


VL' = 4000 v10 = 1852.7740
VA' = 12 X v12 + 0.08 ( Ia )16 .Y +1.116 Y v16

= 4.76537 X + 0.08 61.1154 Y + 5.13727 Y


1852.7740 = 4.76537 X + 10.0265 Y ..... ( 2 )

148.2429 = 2.32391Y

Page 9

Subject CT1 (Financial Mathematics Core Technical) September 2008 Examiners Report

4.76537

from ( 2 ) 0.39711 (1)


Hence Y = 63,790
X = 254,583
(ii)

Amount invested in X is 254,583 v12


= 101,098
and amount invested in Y is:
185,277 - 18,528 - 101,098 = 65,651

(iii)

The spread of the assets is clearly greater than the spread of the liability
(which is a single point).
Hence, Redingtons 3rd condition is satisfied and the fund is immunised.

12

(i)

First 15 years:
Interest paid each month
12
12
i(12 )
i( )

=
300, 000 where 1.085 = 1 +

12
12

12
i( )

= 0.0068215
12

monthly interest = 0.0068215 300,000 = 2,046.45


After repayment of 150,000 after 15 years:
Interest paid each quarter
4
4
i( 4)
i( )

=
150, 000 where 1.085 = 1 +

4
4

4
i( )

= 0.020604
4

Quarterly interest = 0.020604 150,000 = 3,090.66

Page 10

Subject CT1 (Financial Mathematics Core Technical) September 2008 Examiners Report

Total interest paid over the 25 years


= (2046.45 12 15) + (3090.66 4 10) = 491,987.40
(ii)

( 6)
@
30

150,000 = X s

4 %

where X = Amount paid in each 6 month period

( 6)

30

30
1.045 ) 1
(
=
6
d( )

6
d ( 6)
1

where
= 1
1.045
6

d (6) = 0.043856
Hence X =

150000

(1.045) 1
0.043856

30

150000
= 2396.23
62.5985

Monthly contribution =
(iii)

2396.23
= 399.37 per month
6

Savings proceeds after 15 years:


(12 )
12 399.37 s15

10%

(12 )
15

where s

i
s15
12
d( )

= 1.0533781 31.7725
= 33.46845

Hence, savings proceeds


= 4792.44 33.46845 = 160,395.56

Loan o/s after 15 years


= 300,000 - 160,395.56 = 139,604.44

Page 11

Subject CT1 (Financial Mathematics Core Technical) September 2008 Examiners Report

Let Y = new monthly payment


(12 )
10
7%

139,604.44 = 12 Y a
= 12Y

0.07
7.02358
0.06785

Y = 1, 605.50 per month

END OF EXAMINERS REPORT

Page 12

Faculty of Actuaries

Institute of Actuaries

EXAMINATION
21 April 2009 (am)

Subject CT1 Financial Mathematics


Core Technical
Time allowed: Three hours
INSTRUCTIONS TO THE CANDIDATE
1.

Enter all the candidate and examination details as requested on the front of your answer
booklet.

2.

You must not start writing your answers in the booklet until instructed to do so by the
supervisor.

3.

Mark allocations are shown in brackets.

4.

Attempt all 11 questions, beginning your answer to each question on a separate sheet.

5.

Candidates should show calculations where this is appropriate.

Graph paper is not required for this paper.

AT THE END OF THE EXAMINATION


Hand in BOTH your answer booklet, with any additional sheets firmly attached, and this
question paper.
In addition to this paper you should have available the 2002 edition of the Formulae
and Tables and your own electronic calculator from the approved list.

CT1 A2009

Faculty of Actuaries
Institute of Actuaries

Describe the characteristics of Government Bills.

Describe the characteristics of:


(a)
(b)

[3]

an interest-only loan (or mortgage); and


a repayment loan (or mortgage).

[4]

A loan is to be repaid by an annuity payable annually in arrear. The annuity starts at a


rate of 300 per annum and increases each year by 30 per annum. The annuity is to
be paid for 20 years.
Repayments are calculated using a rate of interest of 7% per annum effective.
Calculate:

(i)

The amount of the loan.

[3]

(ii)

The capital outstanding immediately after the 5th payment has been made. [2]

(iii)

The capital and interest components of the final payment.

(i)

Explain what is meant by the no arbitrage assumption in financial


mathematics.

[2]
[Total 7]

[2]

An investor entered into a long forward contract for 100 nominal of a security eight
years ago and the contract is due to mature in four years time. The price per 100
nominal of the security was 94.50 eight years ago and is now 143.00. The risk-free
rate of interest can be assumed to be 5% per annum effective throughout the contract.
(ii)

Calculate the value of the contract now if it were known from the outset that
the security will pay coupons of 9 two years from now and 10 three years
from now. You may assume no arbitrage.
[5]
[Total 7]

CT1 A20092

A companys required return for a particular investment project can be expressed as a


force of interest, (t). This force of interest is a function of time and at any time t,
measured in years, is given by the formula:

(t ) = 0.05 + 0.002t 0 t 5
(t ) = 0.06
5<t
The expenditure required for this project is a payment of 100,000 at t = 0 and a
further payment of 80,000 at t = 2.
The income received from the project is a payment stream paid continuously from
t = 8 to t = 12 under which the annual rate of payment at time t is 100, 000e0.001t .
Calculate the discounted payback period for this project.

[8]

A pension fund purchased an office block nine months ago for 5 million.
The pension fund will spend a further 900,000 on refurbishment in two months time.
A company has agreed to occupy the office block six months from now. The lease
agreement states that the company will rent the office block for fifteen years and will
then purchase the property at the end of the fifteen year rental period for 6 million.
It is further agreed that rents will be paid quarterly in advance and will be increased
every three years at the rate of 4% per annum compound. The initial rent has been set
at 800,000 per annum with the first rental payment due immediately on the date of
occupation.
Calculate, as at the date of purchase of the office block, the net present value of the
project to the pension fund assuming an effective rate of interest of 8% per annum.
[8]

A fund had a value of 150,000 on 1 July 2006. A net cash flow of 30,000 was
received on 1 July 2007 and a further net cash flow of 40,000 was received on 1 July
2008. The fund had a value of 175,000 on 30 June 2007 and a value of 225,000 on
30 June 2008. The value of the fund on 1 January 2009 was 280,000.
(i)

Calculate the time-weighted rate of return per annum earned on the fund
between 1 July 2006 and 1 January 2009.

[3]

(ii)

Calculate the money-weighted rate of return per annum earned on the fund
between 1 July 2006 and 1 January 2009.
[4]

(iii)

Explain why the time-weighted rate of return is more appropriate than the
money-weighted rate of return when comparing the performance of two
investment managers over the same period of time.
[2]
[Total 9]

CT1 A20093

PLEASE TURN OVER

An insurance company has liabilities consisting of eleven annual payments of 1


million, with the first payment due to be made in 10 years time and the last payment
due to be made in 20 years time. The rate of interest is 6% per annum effective.
(i)

Show that the discounted mean term of these liabilities, to four significant
figures, is 14.42 years.
[3]

The insurance company holds two zero-coupon bonds, one paying X in 10 years
time and the other paying Y in 20 years time.
(ii)

(iii)

Find values of X and Y such that Redingtons first two conditions for
immunisation from small changes in the rate of interest are satisfied.

[6]

Explain, without making any further calculations, whether you would expect
Redingtons third condition for immunisation to be satisfied for the values of
X and Y calculated in (ii).
[2]
[Total 11]

Two bonds paying annual coupons of 5% in arrear and redeemable at par have terms
to maturity of exactly one year and two years, respectively.
The gross redemption yield from the 1-year bond is 4.5% per annum effective; the
gross redemption yield from the 2-year bond is 5.3% per annum effective. You are
informed that the 3-year par yield is 5.6% per annum.
Calculate all zero-coupon yields and all one-year forward rates implied by the yields
given above.
[12]

10

A loan pays coupons of 11% per annum quarterly on 1 January, 1 April, 1 July and
1 October each year. The loan will be redeemed at 115% on any 1 January from
1 January 2015 to 1 January 2020 inclusive, at the option of the borrower. In addition
to the redemption proceeds, the coupon then due is also paid.
An investor purchased a holding of the loan on 1 January 2005, immediately after the
payment of the coupon then due, at a price which gave him a net redemption yield of
at least 8% per annum effective. The investor pays tax at 30% on income and 25% on
capital gains.
On 1 January 2008 the investor sold the holding, immediately after the payment of the
coupon then due, to a fund which pays no tax. The sale price gave the fund a gross
redemption yield of at least 9% per annum effective.
Calculate the following:
(i)

The price per 100 nominal at which the investor bought the loan.

[6]

(ii)

The price per 100 nominal at which the investor sold the loan.

[4]

(iii)

The net yield per annum convertible quarterly that was actually obtained by
the investor during the period of ownership of the loan.
[5]
[Total 15]

CT1 A20094

11

An individual wishes to receive an annuity which is payable monthly in arrears for 15


years. The annuity is to commence in exactly 10 years at an initial rate of 12,000 per
annum. The payments increase at each anniversary by 3% per annum. The individual
would like to buy the annuity with a single premium 10 years from now.
(i)

Calculate the single premium required in 10 years time to purchase the


annuity assuming an interest rate of 6% per annum effective.

[5]

The individual wishes to invest a lump sum immediately in an investment product


such that, over the next 10 years, it will have accumulated to the premium calculated
in (i). The annual effective returns from the investment product are independent and
(1 + it ) is lognormally distributed, where it is the return in the tth year. The expected
annual effective rate of return is 6% and the standard deviation of annual returns is
15%.
(ii)

Calculate the lump sum which the individual should invest immediately in
order to have a probability of 0.98 that the proceeds will be sufficient to
purchase the annuity in 10 years time.
[9]

(iii)

Comment on your answer to (ii).

END OF PAPER

CT1 A20095

[2]
[Total 16]

Faculty of Actuaries

Institute of Actuaries

Subject CT1 Financial Mathematics


Core Technical
EXAMINERS REPORT
April 2009

Introduction
The attached subject report has been written by the Principal Examiner with the aim of
helping candidates. The questions and comments are based around Core Reading as the
interpretation of the syllabus to which the examiners are working. They have however given
credit for any alternative approach or interpretation which they consider to be reasonable.

R D Muckart
Chairman of the Board of Examiners
June 2009

Faculty of Actuaries
Institute of Actuaries

Subject CT1 (Financial Mathematics Core Technical) April 2009 Examiners Report
Comments
Please note that different answers may be obtained to those shown in these solutions
depending on whether figures obtained from tables or from calculators are used in the
calculations but candidates are not penalised for this. However, candidates may be penalised
where excessive rounding has been used or where insufficient working is shown.
There were some excellent performances and well-prepared candidates scored well across the
whole paper. However, the comments below on each question concentrate on areas where
candidates could have improved their performance.
Q1, Q2.
As has often been the case when words rather than numbers have been required, these
bookwork questions were answered relatively poorly (although Q2 was answered better than
Q1).
Q3.
Well answered.
Q4.
Defining an arbitrage profit correctly was also acceptable as an answer to (i) although a
description of both possible arbitrage scenarios was required for full marks. Many
candidates performed the calculations well although the methodology being used was not
always clear.
Q5.
The question required an ability to bring together two separate elements of the syllabus and
less well-prepared candidates seemed to struggle with this.
Q6.
This was another question where students scored relatively poorly with many candidates
having difficulty with the income calculation. A common error was to assume that the income
rose by 4% every three years.
Q7.
This was answered much better than questions on the same topic in previous exams.
However, some candidates did confuse the money-weighted and time-weighted rates of
return.

Page 2

Subject CT1 (Financial Mathematics Core Technical) April 2009 Examiners Report
Q8.
It was particularly disappointing to see many candidates using the wrong formula for DMT
in part (i) but ending their proof with=14.42 QED in the final line. This suggests a lack of
professionalism, honesty and integrity which are key attributes of the actuarial profession.
Part (ii) was well-answered with various different methods leading to the correct answer.
Q9.
This was the worst-answered question on the paper although it was still possible to score
significant marks by calculating forward rates using the correct formula even if the spot rates
had been calculated incorrectly.
Q10.
Part (i) was answered well but many candidates lost marks in part (ii) by not realising that a
separate test was required to ascertain the worst time to redemption. Many candidates
calculated the annual effective yield rather than the yield per annum convertible quarterly in
part (iii).
Q11.
Many candidates seemed confused as to what to calculate in part (i) and failed to distinguish
between the premium needed in 10 years time and the present value of that premium. Part
(ii) was answered well (although some candidates appeared to be short of time at this stage).
Part (iii) was answered very poorly with many candidates not appreciating the effects of the
high variance.

Page 3

Subject CT1 (Financial Mathematics Core Technical) April 2009 Examiners Report

Characteristics of government bills:

short-dated securities issued by governments to fund their short-term spending


requirements.
issued at a discount and redeemed at par with no coupon.
mostly denominated in the domestic currency, although issues can be made in
other currencies.
yield is typically quoted as a simple rate of discount for the term of the bill
absolutely secure
often highly marketable despite being unquoted.
often used as a benchmark risk-free short-term investment.

(a)

An interest-only loan requires the borrower only to pay interest on the entire
loan in each time period. The loan does not reduce over time so the interest
remains constant. A separate investment or savings account can be established
in which payments are made to extinguish the whole loan at the end of the
term.

(b)

A repayment loan involves level repayments of capital and interest. The first
part of the payment is used to pay interest on any remaining capital. The
remaining part of the payment is then used to repay capital so that the capital
gradually reduces over the term of the loan.

(i)

300a20 + 30v ( a )19 at 7%


1 82.9347 = 5503.47
= 300 (10.594 ) + 30 1.07

(ii)

Capital outstanding after 5 payments:


420a15 + 30 ( a )15

= 420 9.1079 + 30 61.5540 = 5671.94


(iii)

Cap o/s after 19 payments = 870v @ 7% = 813.08


= Capital in the final payment
Interest in the final payment = 870 813.08 = 56.92

Page 4

Subject CT1 (Financial Mathematics Core Technical) April 2009 Examiners Report

(i)

The no arbitrage assumption means that neither of the following applies:


(a)

an investor can make a deal that would give her or him an immediate
profit, with no risk of future loss;

nor
(b)

(ii)

an investor can make a deal that has zero initial cost, no risk of future
loss, and a non-zero probability of a future profit.

The forward price at the outset of the contract was:

(94.5 9v

10
5%

10v11
5% (1.05 )

12

= 149.29

The forward price that should be offered now is:

(143 9v

2
5%

3
10v5%
(1.05 ) = 153.39
4

Hence the value of the contract now is:


4
= 3.37
(153.39 149.29 ) v5%

Note:
This result can also be obtained directly from:
143 94.5 (1.05 ) = 3.38
8

since the coupons are irrelevant in this calculation.

Working in 000s
( 0.05+ 0.002t )dt
PV of outgo = 100 + 80e 0
2

= 100 + 80e

0.05t +0.001t 2

= 100 + 80e0.104 = 172.10


DPP is value of T for which:
PV (income paid up to T) = PV (outgo)

Page 5

Subject CT1 (Financial Mathematics Core Technical) April 2009 Examiners Report
Where
PV (income paid up to T) =

and v(t)

8 100e

0.001t

v ( t ) dt

5
t
( 0.05+0.002t )dt + 0.06 dt
5
0

=e

=e

0.05t + 0.001t 2

0.06t 0.30 )
.e (

= e 0.275 . e 0.06t e0.30

= e0.025 e 0.06t
T

PV ( income paid up to T ) = 100e0.001t e0.025 e0.06t dt


8

= 100e0.025 e 0.059t dt
8

100 0.025 0.059T


e
e
e 0.0598

0.059

= 1737.8222 e0.059T + 1083.97

DPP is T such that


172.10 = 1737.8222e 0.059T + 1083.97

e0.059T = 0.52472
0.059T = Ln(0.52472) T = 10.93 years

Page 6

Subject CT1 (Financial Mathematics Core Technical) April 2009 Examiners Report

Working in 000s
11
12

= 5000 + 900v

PV of costs

at 8%

= 5838.695
1312

PV of income = 800v

1312

= 800v

( a( ) + 1.04 v a( ) +
4
3

( 4)
3

3 3

(1 + (1.04v ) +
3

( 4)
3

+ (1.04 ) v12 a
12

4
3

(1.04v )12

( 1.04
1.08 )
3

( 1.04
1.08 )

1
= 800 0.9082811.049519 2.5771

15

= 1965.3133 4.038121
= 7936.173
16 312

PV of proceeds from sale = 6000v

= 1717.969

NPV of project = 7936.173+1717.969 5838.695


= 3815.447 (i.e. 3,815,447)

Working in 000s
(i)

TWRR is i such that

(1 + i )2

175
225
280

150 175 + 30 225 + 40

175 225 280

= 1.352968
150 205 265

i = 12.85% p.a.

Page 7

Subject CT1 (Financial Mathematics Core Technical) April 2009 Examiners Report
(ii)

MWRR is i such that


150 (1 + i )

2 12

+ 30 (1 + i )

112

+ 40 (1 + i )

= 280

Try: i = 12%, LHS = 277.02


i = 12.5%, LHS = 279.58
i = 13%, LHS = 282.16
i = 12.5% +

( 28 27.958)
(28.216 27.958)

0.5%

= 12.58% p.a.

(iii)

The TWRR is better for comparing 2 investment managers performances as it


is not sensitive to cash flow amounts and timing of payments. The MWRR is
sensitive to both.

(i)

Working in m
Discounted mean term =
10v10 + 11v11 + 12v12 + ............... + 20v 20
v10 + v11 + v12 + ................. + v 20
=

10v + 11v 2 + 12v3 + ............ + 20v11


v + v 2 + v3 + ................ + v11

9a11 + ( a )11
a11

( a )11

=9+

( a )11
a11

at 6%

= 42.7571

42.7571
= 14.42128
7.8869
to 4 significent figures DMT = 14.42
DMT = 9 +

(ii)

First condition: pv assets = pv liabilities

Xv10 + Yv 20 = v9 a11 *1

at 6%.

X 0.55839 + Y 0.31180 = 0.59190*7.8869

= 4.668256

Page 8

.(1)

(using tables)

Subject CT1 (Financial Mathematics Core Technical) April 2009 Examiners Report
2nd condition:

DMT assets = DMT liabilities

X *10v10 + Y * 20v 20
Xv10 + Yv 20

= 14.42128 (use of 14.42 from (i) will be

accepted)

X *5.5839 + Y *6.236 = 14.42128 Xv10 + Yv 20

= 14.42128* 4.668256 from (1)


= 67.3222

(or 67.3163 if DMT of 14.42 is used)(2)

Equ n (2) 10* Equ n (1)


Y *6.236 Y *3.1180 = 67.3222 10* 4.668256

20.639667
= 6.6195 (or 6.6176 if DMT of 14.42 is used)
3.1180
[or VA' = VL' (differentiating with respect to i)
Y =

10 Xv11 + 20Yv 21 = 10v11 + 11v12 + + 20v 21

= v10 9a11 + ( Ia )11

5.2679 X + 5.8831Y = 63.5112


Equ n (2)

.(2)

5.2679
Equ n (1)
5.8831

2.94155Y = 19.4711 Y = 6.6193 ]


Equ n (1) X * 0.55839 = 4.668256 6.6195 * 0.31180
X = 4.6639 (or 4.6650 if DMT of 14.42 is used)
[check, in equ n (2).
(iii)

4.6639 * 5.5839 + 6.6195 * 6.236 = 67.3222]

For the third condition to be satisfied, it is necessary for the spread of the
assets to exceed the spread of the liabilities. This appears to be the case given
that the liabilities occur in equal annual amounts at durations from 10 years to
20 years, whereas the assets are concentrated in two lumps at the two most
extreme durations, 10 years and 20 years.

Page 9

Subject CT1 (Financial Mathematics Core Technical) April 2009 Examiners Report

Let the 1-year and 2-year zero-coupon yields (spot rates) be ii and i2 respectively.

105
= 105v @ 4.5%
1 + i1
i1 = 0.045
For the 2-year spot rate:
5
105
2
+
= 5a2 5.3% + 100v5.3%
2
1 + i1 (1 + i2 )

1 1

2
5
105
1.053 + 100

5
+
=
1.045 (1 + i2 )2
0.053
1.0532

= 9.257681 + 90.186858
= 99.444539
105

(1 + i2 )

= 99.444539

(1 + i2 ) =
2

5
1.045

105
94.659850

i2 = 5.3202% p.a.
For the 3-year spot rate:
The 3-year par yield is 5.6% p.a.

1
1
1
1
+
1 = 0.056
+
+
2
3
1 + i1 (1 + i )
(1 + i3 ) (1 + i3 )3
2

1.056

(1 + i3 )

= 1

(1 + i3 ) =
3

0.056
0.056

1.045 (1.053202 )2

1.056
0.895926

i3 = 5.6324% p.a.

Page 10

Subject CT1 (Financial Mathematics Core Technical) April 2009 Examiners Report
1-year forward rates:

f 0 = i1 = 4.5% p.a.

(1 + i1 )(1 + f1 ) = (1 + i2 )2
1 + f1 =

1.0532022
1.045

f1 = 6.1468% p.a.

(1 + i2 )2 (1 + f 2 ) = (1 + i3 )3
3
1.056324 )
(
1 + f2 =
(1.053202 )2

f 2 = 6.2596% p.a.

10

(i)

check for capital gain:

g (1 t1 ) =

0.11
(1 0.3)
1.15

= 0.06696
4
i = 8% i ( ) = 0.077706
4
i ( ) > g (1 t1 )

Theres a capital gain and thus loan should be assumed to be redeemed at


the latest possible date.
Let P be price at which the investor bought the loan.
Then
( 4)
15

P = 11 0.7 a

P=

+ 115v15 0.25 (115 P ) v15 at 8%

7.7 1.029519 8.5595 + 0.75 115 0.31524


1 0.25 0.31524

= 103.17 per 100 nominal

Page 11

Subject CT1 (Financial Mathematics Core Technical) April 2009 Examiners Report
(ii)

check for capital gain:


0.11
= 0.095652
1.15

g (1 t1 ) =

4
i = 9% i ( ) = 0.087113
4
i( ) < g 1 t

Theres no capital gain and thus loan should be assumed to be redeemed at


the earliest possible date.
Let P ' be the price at which the investor sold the loan. Then
P ' = 11a

( 4)

+ 115v 7 at 9%

= 111.033144 5.033 + 115 0.54703


= 120.1064 per 100 nominal

(iii)

Let j be the yield per quarter. Then


103.17 =

11
0.7a12 + 120.1064v12 0.25 (120.1064 103.17 ) v12 at j %
4

103.17 = 1.925 a12 + 115.8723 v12

Try
j = 3%: RHS = 100.4319638
j = 2.5%: RHS = 105.9042724
Linear interpolation:

j = 0.025 + 0.005

(103.17 105.9042724 )

(100.4319638 105.9042724 )

= 0.02749828
Hence, net yield is 11% p.a. (or 10.99931% p.a.) payable quarterly.

Page 12

Subject CT1 (Financial Mathematics Core Technical) April 2009 Examiners Report

11

(i)

In 10 years time the single premium P is

((

12 )
1

P = 12000 a

(12 )
v+
1

+ 1.03a

(1.03)2 a1(12)v 2 + ... + (1.03)14 v14a1(12)

1.03
(12 ) 1.03 1.03
1 +
+
+ ... +

1
1.06 1.06
1.06

14

= 12000a

1.03 15
1

(12 ) 1.06
= 12000a
1

1.03
1 1.06

(12 )
1

where a

=
i
=

(12 )

1.027211
= 0.969067
1.06

P = 12000 0.969067

0.3499146
0.0283019

= 143,774.45
(ii)

E (1 + it ) = 1.06 = e

2
2

Var (1 + it ) = ( 0.15 ) = e2+ . e 1


2

Then

0.152

(1.06 )

= e 1
2

2 = 0.01982706
= n 1.06

0.01982706
2

= 0.04835538
S10 LN ( 0.4835538, 0.1982706 )

Let X be the amount to be invested at time 0

Page 13

Subject CT1 (Financial Mathematics Core Technical) April 2009 Examiners Report
We want Pr ( X .S10 143, 774.45 ) = 0.98
143, 774.45

so Pr S10
= 0.98
X

Ln 143774.45
10
X
so 1
= 0.02

2
10

Ln 143774.45
10
X
102

So Ln

= 2.0537

143774.45
= 2.0537 0.1982706 + 0.4835538
X

= 0.430909

(iii)

143774.45
= 0.6499179
X

X = 221, 219.41

It might seem odd that the initial investment needs to be substantially higher
than the single premium required in 10 years time to have a 98% probability
of accumulating to the single premium.
This strange result is explained by the fact that the variance of the interest rate
is so high relative to the mean. There is therefore a significant risk that the
investment will decrease in value over the next 10 years.

END OF EXAMINERS REPORT

Page 14

Faculty of Actuaries

Institute of Actuaries

EXAMINATION
30 September 2009 (am)

Subject CT1 Financial Mathematics


Core Technical
Time allowed: Three hours
INSTRUCTIONS TO THE CANDIDATE
1.

Enter all the candidate and examination details as requested on the front of your answer
booklet.

2.

You must not start writing your answers in the booklet until instructed to do so by the
supervisor.

3.

Mark allocations are shown in brackets.

4.

Attempt all 10 questions, beginning your answer to each question on a separate sheet.

5.

Candidates should show calculations where this is appropriate.

Graph paper is not required for this paper.

AT THE END OF THE EXAMINATION


Hand in BOTH your answer booklet, with any additional sheets firmly attached, and this
question paper.
In addition to this paper you should have available the 2002 edition of the Formulae
and Tables and your own electronic calculator from the approved list.

CT1 S2009

Faculty of Actuaries
Institute of Actuaries

A 182-day government bill, redeemable at 100, was purchased for 96 at the time of
issue and was later sold to another investor for 97.89. The rate of return received by
the initial purchaser was 5% per annum effective.
(a)

Calculate the length of time in days for which the initial purchaser held the
bill.

(b)

Calculate the annual simple rate of return achieved by the second investor.
[4]

List the characteristics of an equity investment.

An investor bought a number of shares at 78 pence each on 31 December 2005. She


received dividends on her holding on 31 December 2006, 2007 and 2008. The rate of
dividend per share is given in the table below:

[4]

Date

Rate of dividend per share

Retail price index

31.12.2005
31.12.2006
31.12.2007
31.12.2008

-----4.1 pence
4.6 pence
5.1 pence

147.7
153.4
158.6
165.1

On 31 December 2008, she sold her shares at a price of 93 pence per share.
Calculate, using the retail price index values shown in the table, the effective annual
real rate of return achieved by the investor
[7]

A fixed-interest security has just been issued. The security pays half-yearly coupons
of 5% per annum in arrear and is redeemable at par 20 years after issue.
(i)

Calculate the price to provide an investor with a net redemption yield of 6%


per annum effective. The investor pays tax at a rate of 20% on income and is
not subject to capital gains tax.
[3]

(ii)

Determine the annual effective gross redemption yield of this security


assuming the price calculated in (i) is paid.

(iii)

[5]

Determine the real annual effective gross redemption yield of this security if
the rate of inflation is constant over the twenty years at 3% per annum.
[2]
[Total 10]

CT1 S20092

The force of interest (t ) at time t is a + bt 2 where a and b are constants. An amount


of 100 invested at time t = 0 accumulates to 130 at time t = 5 and 200 at time
t = 10.
(i)

Calculate the values of a and b.

[6]

(ii)

Calculate the constant rate of interest per annum convertible monthly that
would give rise to the same accumulation from time t = 0 to time t = 5.
[2]

(iii)

Calculate the constant force of interest that would give rise to the same
accumulation from time t = 5 to time t = 10.
[2]
[Total 10]

(i)

Distinguish between a future and an option.

[2]

An investor wishes to purchase a one year forward contract on a risk-free bond


which has a current market price of 97 per 100 nominal. The bond will pay
coupons at a rate of 7% per annum half yearly. The next coupon payment is
due in exactly six months and the following coupon payment is due just before
the forward contract matures. The six-month risk-free spot interest rate is 5%
per annum effective and the 12-month risk-free spot interest rate is 6% per
annum effective.
(ii)

Stating all necessary assumptions:


(a)

Calculate the forward price of the bond.

(b)

Calculate the six-month forward rate for an investment made in six


months time.

(c)

Calculate the purchase price of a risk-free bond with exactly one year
to maturity which is redeemed at par and which pays coupons of 4%
per annum half-yearly in arrears.

(d)

Calculate the gross redemption yield from the bond in (c).

(e)

Comment on why your answer in (d) is close to the one-year spot rate.
[10]
[Total 12]

CT1 S20093

PLEASE TURN OVER

A member of a pensions savings scheme invests 1,200 per annum in monthly


instalments, in advance, for 20 years from his 25th birthday. From the age of 45, the
member increases his investment to 2,400 per annum. At each birthday thereafter
the annual rate of investment is further increased by 100 per annum. The
investments continue to be made monthly in advance for 20 years until the
individuals 65th birthday.
(i)

Calculate the accumulation of the investment at the age of 65 using a rate of


interest of 6% per annum effective.
[6]

At the age of 65, the scheme member uses his accumulated investment to purchase an
annuity with a term of 20 years to be paid half-yearly in arrear. At this time the
interest rate is 5% per annum convertible half-yearly.

(ii)

Calculate the annual rate of payment of the annuity.

[3]

(iii)

Calculate the discounted mean term of the annuity, in years, at the time of
purchase.
[3]
[Total 12]

A bank offers a customer two different repayment options on a loan of 50,000 as


follows:
Option 1 level instalments of capital and interest are paid annually in arrear over a
period of 20 years.
Option 2 over the 20-year term the customer pays only interest on the loan, annually
in arrear at a rate of 5.5% per annum with the whole of the capital amount payable at
the end of the term. The customer will take out a separate savings policy which
involves making monthly payments in advance such that the proceeds will be
sufficient to repay the loan at the end of its term. The payments into the savings
policy accumulate at a rate of interest of 4% per annum effective.
(i)

Determine the effective rate of interest per annum that would be paid by the
customer on the loan under Option 1, given that the level annual instalment on
this loan is 4,012.13.
[3]

(ii)

Determine the annual effective rate of interest paid by a customer under


Option 2.
[7]
[Total 10]

CT1 S20094

A life insurance company is issuing a single premium policy which will pay out
20,000 in twenty years time. The interest rate the company will earn on the invested
funds over the first ten years of the policy will be 4% per annum with a probability of
0.3 and 6% per annum with a probability of 0.7. Over the second ten years the
interest rate earned will be 5% per annum with probability 0.5 and 6% per annum
with probability 0.5.
(i)

Calculate the premium that the company would charge if it calculates the
premium using the expected annual rate of interest in each ten year period. [2]

(ii)

Calculate the expected profit to the company if the premium is calculated as in


(i). The rate of interest in the second ten year period is independent of that in
the first ten year period.
[3]

(iii)

Explain why, despite the company using the expected rate of interest to
calculate the premium, there is a positive expected profit.

(iv)

[2]

By considering each possible outcome in (ii):


(a)
(b)

CT1 S20095

Find the range of possible profits.


Calculate the standard deviation of the profit to the company.
[7]
[Total 14]

PLEASE TURN OVER

10

A group of experts is analysing options to try to avert problems caused by climate


change. They agree on the following expected costs and benefits of climate change
over the next 50 years, starting from the current time. All figures are given in 2009
dollars.
Costs of climate change:

Serious events will occur once every three years, in arrear, each giving rise to
costs of $30bn, incurred immediately on the date of the event.

Communities affected by climate change will incur costs of $20bn per annum
incurred continuously, increasing at a continuous rate of 1% per annum.

Other costs, assumed to be $40bn per annum, will be incurred annually in


arrear.

Benefits arising from climate change:

Benefits from higher crop yields and lower heating costs are assumed to be
$10bn per annum, incurred annually in arrear.

The experts are considering whether to recommend investment in a carbon storing


technology which, it is believed, will reduce all the costs and benefits listed above to
zero. The technology requires a one-off investment immediately of $440bn. Costs
are then assumed to be $50bn per annum incurred annually in arrear for 50 years.
The experts do not agree about the appropriate rate of interest at which to evaluate the
options available. One group believes that the net present value of using the carbon
storage technology should be evaluated at a real rate of return of 4% per annum
effective. A second group believe that it should be evaluated at a real rate of return of
1% per annum effective.
(i)

Define what is meant by the discounted payback period of an investment and


indicate its main disadvantage as an investment decision criterion.
[3]

(ii)

Explain why the project must have a discounted payback period when the
interest rate is 1.5% and the internal rate of return is higher than 1.5%.
[2]

(iii)

Calculate the net present value of the carbon storing technology at a real rate
of interest of 1% per annum effective.
[5]

(iv)

Calculate the net present value of the carbon storing technology at a real rate
of interest of 4% per annum effective.
[5]
`
Comment on whether the investment in the carbon storing technology should
go ahead.
[2]
[Total 17]

(v)

END OF PAPER

CT1 S20096

Faculty of Actuaries

Institute of Actuaries

Subject CT1 Financial Mathematics.


Core Technical.
September 2009 examinations
EXAMINERS REPORT

Introduction
The attached subject report has been written by the Principal Examiner with the aim of
helping candidates. The questions and comments are based around Core Reading as the
interpretation of the syllabus to which the examiners are working. They have however given
credit for any alternative approach or interpretation which they consider to be reasonable.
R D Muckart
Chairman of the Board of Examiners
December 2009
Comments for individual questions are given with the solutions that follow.

Faculty of Actuaries
Institute of Actuaries

Subject CT1 (Financial Mathematics. Core Technical) September 2009 Examiners Report

Please note that different answers may be obtained to those shown in these solutions depending
on whether figures obtained from tables or from calculators are used in the calculations but
candidates are not penalised for this. However, candidates may be penalised where excessive
rounding has been used or where insufficient working is shown.
Well-prepared candidates scored well across the whole paper. However, the comments below on
each question concentrate on areas where candidates could have improved their performance.

1
a. 96 1.05

ln

97.89

96

ln 1.05

97.89

1.05

97.89
96

0.400 years or 146 days

b. Second investor held the bill for 36 days. Therefore


97.89 1

36
i
365

100

365 100
1
36 97.89

21.854%

This was answered well except by the very weakest candidates.

2
Issued by corporations.
Holders entitled to a distribution (dividend) declared from profits.
Potential for high returns relative to other asset classes.
Commensurate risk of capital losses.
Lowest ranking finance issued by companies.
Initial running yield low but has potential to increase with dividend growth.
Dividends and capital values have the potential to grow in nominal terms during times of inflation.
Return made up of income return and capital gains.
Marketability depends on the size of the issue.
Ordinary shareholders receive voting rights in proportion to their holding.

This question was not answered as well as the examiners would have expected given that
the topic is standard bookwork.

3
We convert all cash flow to amounts in time 0 values:

Page 2

Subject CT1 (Financial Mathematics. Core Technical) September 2009 Examiners Report

Dividend paid at t 1:10000 0.041

147.7
153.4

394.77

Dividend paid at t

2 :10000 0.046

147.7
158.6

428.39

Dividend paid at t

3:10000 0.051

147.7
165.1

456.25

Sale proceeds at t

3:10000 0.93

147.7
8319.87
165.1

Equation of value involving v where v

1
1 r

and r = real rate of return:


7800 394.77v 428.39v2 8776.17v3..... (1)

[To estimate r:
Approx nominal rate of return is

4.6

93 78
3

/ 78 12.3% p.a.

Average inflation over 3 year period comes from

165.1
147.7

1 3.8 % p.a.

Approx real return:

1.123
1 8.2 % p.a. ]
1.038

Try r 8%, RHS of (1) 7699.61


r

7%, RHS of (1) 7907.09

7%

7907.09 7800
1%
7907.69 7699.61

= 7.52 % p.a.

Page 3

Subject CT1 (Financial Mathematics. Core Technical) September 2009 Examiners Report

Some candidates seemed to struggle to derive the equation of value based on a real rate
of return and multiplied (rather than divided) the payments by the increase in the
inflation index.

4
(i) Let required price = P:

2
20

100v20 at 6%

1 0.2 5a
i

2
20

a20 =

0.06
11.4699 11.6394; v 20
0.059126

0.311805

Therefore

1 0.2 5 11.6394 100 0.311805


46.5576 31.1805 77.7381

(ii) The equation of value for the gross rate of return is:
2
20

77.7381 5a

100v20

If i = 8%
2
20

a20 = 1.019615 9.8181 10.0107; v 20

0.21455

RHS = 50.0534 + 21.4550 = 71.5084


If i = 7%
2
20

a20 = 1.017204 10.5940 10.7763; v 20

0.25842

RHS = 53.8813 + 25.8420 = 79.7233


Interpolating gives i

0.07

79.7233 77.7381
0.01 7.24% 7.2% say
79.7233 71.5084

(iii) If the nominal rate of return is 7.2% per annum effective and inflation is 3% per
annum effective, then the real rate of return is calculated from:
1.072
1 4.1%
1.03
This question was answered very well.

Page 4

Subject CT1 (Financial Mathematics. Core Technical) September 2009 Examiners Report

5
5

a bt 2 dt

(i) 130 100exp

5
1 bt 3
3
0

100exp at

0
10

a bt 2 dt

200 100exp

10
1 bt 3
3
0

100exp at

ln 1.3
ln 2

100exp 5a 41.667b

100exp 10a 333.333b

5a 41.667b
10a 333.333b

The second expression less twice times the first expression gives:
ln(2) 2ln(1.3)

250b

b 0.0006737

ln(2) 333.333 0.0006737


10

(ii) 100 1

(iii) 130e5

12

60

130

12

200

0.04686

ln

12

200
130

12

130
100

60

12

5.259% p.a.

8.616% p.a.

This question was answered very well.

6
(i) A future is a contract which obliges the parties to deliver/take delivery of a
particular quantity of a particular asset at a particular time at a fixed price.
An option is the right to buy or sell a particular quantity of a particular asset at (or
before) a particular time at a given price.
(ii) Assume no arbitrage
a. Buying the forward is exactly the same as buying the bond except that the
forward will not pay coupons and the forward does not require immediate
settlement.
Page 5

Subject CT1 (Financial Mathematics. Core Technical) September 2009 Examiners Report

Let the forward price = F. The equation of value is:


F

97 1.06

1.06

3.5

1.05

3.5
2

102.82 3.62059 3.5 95.6994


1.06

f 0.5,0.5

b. Let six month forward interest rate

1.05

1 3.4454%
2

This does not have to be expressed as a rate of interest per annum


effective, though it could be.
c. P

2 1.05

0.5

102 1.06

1.9518 96.2264 98.1782

d. Gross redemption yield is i such that


98.1782 2 1 i

0.5

102 1 i

Using the formula for solving a quadratic (interpolation will do):


1 i

0.5

0.97133 . Therefore, i 6% (in fact 5.99%).

e. Answer is very close to 6% (the one-year spot rate) because the payments
from the bond are so heavily weighted towards the redemption time in one
year.
This was generally well-answered apart from part (e). A common error in parts (c) and
(d) was to assume that the coupon payments were 4% per half-year.

7 .
(i)

The accumulation is

i
d

12

12
1200s
1.06
20

1200s20 1.06

1.032211

20

20

12
2300s
20

2300 s20

100 Ia

100 Ia

20

Page 6

1.06

1.06

20

1, 200 36.7856 3.20714 2,300 36.7856


100 98.7004 3.20714

1.032211 141,571.88 84, 606.88 31, 654.60


266,138

12
20

20

Subject CT1 (Financial Mathematics. Core Technical) September 2009 Examiners Report

(ii) Let half-yearly payment = X

Xa40
X

266,138 at 2.5%
266,138
10,601.94
25.1028

Therefore, annual rate of payment = 21,203.88

(iii) Work in half-years. Discounted mean term is:

10,601.94 v + 2v2 ++40v40 /266,138


Numerator = 10,601.94 Ia

40

at 2.5% per half year effective.

10,601.94 433.3248 4,584,075


Therefore DMT = 17.26 half years or 8.63 years.

In part (i), many candidates developed the correct formula although calculation errors
were common. In such cases, candidates also lost marks for not showing and explaining
their working fully. Part (ii) was answered well but many candidates surprisingly had
trouble calculating the DMT in part (iii). In this part, candidates often lost marks for not
showing the units properly at the end of the answer; indeed, in many cases, showing the
units may well have alerted candidates to possible mistakes.

2
(i) The equation of value for the borrower is 4, 012.13a20
Therefore

a 20 =

50, 000 .

50,000
= 12.4622
4,012.13

From inspection of tables, i = 5%

(ii) The second customer pays interest of 0.055 50,000 = 2,750 per annum, annually in arrear.
The annual rate of monthly payments in advance from the savings policy is X such that:

Page 7

Subject CT1 (Financial Mathematics. Core Technical) September 2009 Examiners Report

12
Xs =50, 000 at 4%
20

Xs20
X

i
d

12

50, 000

50, 000
29.7781 1.021537

1, 643.69

The equation of value for this borrower is:


12
1, 643.686a

50, 000 2, 750a20

20

2, 750a20

1, 643.686
d

12

a20

Try i = 6%: RHS = 51,002.41


Try i = 7%: RHS = 47,200.14
By interpolation i = 6.3%

Part (i) was well answered but weaker candidates failed to recognise the need to
calculate separately the payments into the savings policy in part (ii).

3
(i) The expected annual interest rate in the first ten years is 0.3 0.04 + 0.7 0.06 =
0.054. The expected interest rate in the second ten years is clearly 5.5%.
If the premium is calculated on the basis of these interest rates, then the premium will be P such
that:

20, 000

P 1.054

20, 000

10

1.055

2.89022 P

10

6,919.89

(ii) The expected accumulation factor in the first ten years is:
0.3 1.0410 0.7 1.0610 1.69767
The expected accumulation factor in the second ten years is:

0.5 1.05

10

1.06

10

1.70987

As they are independent, we can multiply the accumulation factors together and multiply by the
premium to give an expected accumulation of: 6,919.89 1.69767 1.70987 = 20,087.04.

Page 8

Subject CT1 (Financial Mathematics. Core Technical) September 2009 Examiners Report

The expected profit is 87.04.

(iii) There is an expected profit because (in general) the accumulation of a sum of money at the
expected interest rate is not equal to the expected accumulation when the interest rate is a random
variable.

(iv) The highest possible outcome for the accumulation factor is:
1.0610 1.0610 = 3.20714 with probability 0.7 0.5 = 0.35
The lowest possible outcome is:

1.0410 1.0510 = 2.41116 with probability 0.3 0.5 = 0.15.


The range is therefore: 6,919.89 (3.20714 2.41116) = 5,508.05.
The other two possible outcomes are:

1.0610 1.0510 = 2.91710 with probability 0.7 0.5 = 0.35


10

and 1.04

1.0610 = 2.65089 with probability 0.3 0.5 = 0.15

The mean accumulation factor is: 1.69767 1.70987 = 2.90280


The variance of the accumulation from one unit of investment is:
0.35(3.20714-2.90280)2 + 0.15(2.41116-2.90280)2
+ 0.35 (2.91710-2.90280)2 +0.15 (2.65089-2.90280)2
= 0.03241 + 0.03626 + 0.00007 + 0.00952 = 0.07826.
Standard deviation is

0.07826 = 0.27976.

Standard deviation of the accumulation of the whole premium is: 6,919.89 0.27976 = 1,935.88
which is also the standard deviation of the profit.

This was the worst answered question on the paper with many candidates not recognising
that the accumulation of a sum of money at the expected interest rate is not equal to the
expected accumulation when the interest rate is a random variable. The calculation of the
standard deviation of the accumulation was generally only calculated correctly by the
strongest candidates.

4
(i) The discounted payback period is the first time at which the accumulated profit
from/net present value of the cash flows from a project is positive at a given
interest rate.
Page 9

Subject CT1 (Financial Mathematics. Core Technical) September 2009 Examiners Report

It is an inappropriate decision criterion because it does not tell us anything about


the overall profitability of the project.
(ii) If the internal rate of return were greater than 1.5% then the net present value of
the project at 1.5% must be greater than zero. As such, there must be a discounted
payback period as the discounted payback period is the first time at which the net
present value is greater than zero: such a time must exist.
(iii) Returns are real rates of return and figures are in 2009 dollar terms so we are
automatically working with real rather than nominal values. All figures below are
in $bn.
The net benefits from using the technology are the $30 every three years; $20 incurred
continuously increasing at 1% per annum and $30 per annum incurred annually in arrears.
The costs of the technology are $440 incurred immediately and $50 incurred annually in arrears.
The net present value of the project at 1% per annum effective is:

30 v3 v6

v48

50 20 30a50

440

50a50

The 20 does not need to be discounted because the cash flows are growing at the same rate as they
are being discounted.

30v

1 v 48
560 20a50 calculated at 1%

1 v3

30 0.97059

= 375.967
152.045

1 0.62026

560 20 39.1961

1 0.97059

560

783.922

(iv) The net present value of the project at 4% per annum effective is:

30 v3 v6

v48

'
20a50

All are calculated at 4% except

1.04
-1
1.01

30v

1 v 48
1 v3

30 0.88900

Page 10

30a50

440

50a50

'
a50
which is calculated at

2.97%

i '
20 a50
1 0.15219
1 0.88900

440 20a50
20 1.014779 25.8755 440 20 21.4822

Subject CT1 (Financial Mathematics. Core Technical) September 2009 Examiners Report

203.704 525.158
140.790

440

429.644

(v) Whether the investment should go ahead would depend on the choice of the interest rate it is
clearly a crucial assumption (students could make a choice themselves and indicate whether it
should go ahead on the basis of that rate but there must be some justification for the choice).

This question was also poorly answered possibly because project appraisal using real
interest rates has rarely been examined in the past (and also possibly because of time
pressure). Whilst some parts of the question were challenging (e.g. the treatment of the
increasing costs of climate change), it was disappointing that many candidates failed to
recognise that the costs of climate change no longer incurred would be a benefit of the
carbon storing technology project and so failed to score many marks.
END OF EXAMINERS REPORT

Page 11

Anda mungkin juga menyukai